Anda di halaman 1dari 126

The Official SAT Online Course

Help | Profile | My Organizer | My Bookmarks | Logout

1/1

Answers and Explanations

Test Sections

Back to Score Report

Section 1

Essay
Online - Practice Test #6

Section 2

Section 4

Section 5

Section 6

Section 7

These sample essays were originally handwritten by students but are shown typed here for ease of reading. The essays are displayed exactly as students wrote them, without any corrections to spelling, punctuation, or syntax. One handwritten sample essay is provided to illustrate the need for legible and clear handwriting.

Section 8

Section 9

Exemplars:

Section 10

Essay Prompt

Think carefully about the issue presented in the following excerpt and the assignment below.

Adapted from S.I. Hayakawa, Language in Thought and Action

eg

is te

Do people have to be highly competitive in order to succeed? Plan and write an essay in which you develop your point of view on this issue. Support your position with reasoning and examples taken from your reading, studies, experience, or observations.

re

People who like to think of themselves as tough-minded and realistic tend to take it for granted that human nature is selfish and that life is a struggle in which only the fittest may survive. According to this view, the basic law by which people must live is the law of the jungle. The fittest are those people who can bring to the struggle superior force, superior cunning, and superior ruthlessness.

Back to Score Report

nR

Copyright 2006 The College Board. All rights reserved.

Privacy Policy

Terms of Use

Contact Us

file://E:\\f2.htm

2006-11-12

The Official SAT Online Course

1/15

Help | Profile | My Organizer | My Bookmarks | Logout

Answers and Explanations

Test Sections

Back to Score Report

Section 1

View Answers and Explanations


Online - Practice Test #6

Section 2

Section 4

Section 5

If
(A)
(B)
(C)
(D)
(E)

which of the following could be a value of x ?

Section 6

Section 7

Section 8

Section 9

Section 10

ANSWERS

AND EXPLANATIONS
, then

Explanation for Correct Answer A :

Choice (A) is correct. If

equation are

and

. So

could be

Explanation for Incorrect Answer B : could not be Choice (B) is not correct. The value of

is te r

Explanation for Incorrect Answer C : could not be Choice (C) is not correct. The value of

eg

nR

Explanation for Incorrect Answer D : could not be Choice (D) is not correct. The value of

Explanation for Incorrect Answer E : could not be Choice (E) is not correct. The value of

The length of a rectangular rug is 2 feet more than its width. If the length of the rug is 8 feet, what is the area of the rug in square feet?
(A)

16

(B)

48

(C) 66

(D) 80

(E)

96

ANSWERS

AND EXPLANATIONS

Explanation for Correct Answer B : Choice (B) is correct. The area of the rug is equal to the length of the rug times the feet more than its feet, and its length is width of the rug. Since its length is

ed
.

. The solutions to this

, because

, because

, because

, because

file://E:\\f3.htm

2006-11-12

The Official SAT Online Course

2/15

width, the width of the rug is square feet. which is

. So the area of the rug is

feet times

feet,

Explanation for Incorrect Answer A : Choice (A) is not correct. See the explanation for the correct response (B).

Explanation for Incorrect Answer C : Choice (C) is not correct. See the explanation for the correct response (B).

Explanation for Incorrect Answer D : feet more than its length of Choice (D) is not correct. If the width of the rug were feet, and the area of the rug would be feet, then the width would be square feet. However, this answer is incorrect, because the width of the rug is feet less than the length.

Explanation for Incorrect Answer E : Choice (E) is not correct. See the explanation for the correct response (B).

If
(A)

and

what is the value of 2r?

(B)

(C) 6

(D) 8

(E)

12

ANSWERS

AND EXPLANATIONS

follows that

nR

Explanation for Incorrect Answer A : Choice (A) is not correct. The value of

, so

eg

Explanation for Correct Answer E : and Choice (E) is correct. If

is t
.

er ed

, then


. Since

, it

cannot be

, because

, so

Explanation for Incorrect Answer B : Choice (B) is not correct. The value of

cannot be

, because

, so

Explanation for Incorrect Answer C : Choice (C) is not correct. The value of

cannot be

, because

and

. The value of

Explanation for Incorrect Answer D : Choice (D) is not correct. The value of

cannot be

, because

and

, so

Some integers in set X are even.

If the statement above is true, which of the following must also be true?

(A)

If an integer is even, it is in set X.

If an integer is odd, it is in set X.

file://E:\\f3.htm

2006-11-12

The Official SAT Online Course

3/15

(B)
(C) All integers in set X are even.

(D) All integers in set X are odd.

(E)

Not all integers in set X are odd.

ANSWERS

AND EXPLANATIONS

Explanation for Correct Answer E : are even, then those even integers Choice (E) is correct. If some integers in set are odd. that are not odd. So not all integers in set are members of set

Explanation for Incorrect Answer A : are even, Choice (A) is not correct. Although it is true that some integers in set . So there may it is not necessarily true that every even integer belongs to set . be some even integers that are not in set

Explanation for Incorrect Answer B : are even, Choice (B) is not correct. Although it is true that some integers in set contains any odd integers. So there may be odd it is not necessarily true that . integers that are not in set

Explanation for Incorrect Answer C : Choice (C) is not correct. Although it is true that some integers in set are even. it is not necessarily true that all integers in set

ed

are even,

Explanation for Incorrect Answer D : Choice (D) is not correct. Since some integers in set are odd. that all integers in set

te r

are even, it cannot be true

nR eg
(A)

A triangle has a perimeter of 13 and one side of length 3. If the lengths of the other two sides are equal, what is the length of each of them?

(B)

(C) 6

(D) 7

(E)

ANSWERS

AND EXPLANATIONS
and one side has

Explanation for Correct Answer B : Choice (B) is correct. Since the perimeter of the triangle is

is

length

, it follows that the sum of the lengths of the other two sides is

. Since

these other two sides are of equal length and the sum of their lengths is

, it

follows that the length of each of them is

Explanation for Incorrect Answer A : and one side of Choice (A) is not correct. If the triangle had two sides of length . But the perimeter , the perimeter of the triangle would be length

is

, not

. Thus, the two equal sides cannot have length

Explanation for Incorrect Answer C : Choice (C) is not correct. If the triangle had two sides of length

and one side of

length

, the perimeter of the triangle would be

. But the

perimeter is

, not

. Thus, the two equal sides cannot have length

Explanation for Incorrect Answer D : Choice (D) is not correct. If the triangle had two sides of length

and one side of

file://E:\\f3.htm

2006-11-12

The Official SAT Online Course

4/15

, the perimeter of the triangle would be length . Thus, the two equal sides cannot have length , not is

. But the perimeter .

Explanation for Incorrect Answer E : Choice (E) is not correct. If the triangle had two sides of length

and one side of

length

, the perimeter of the triangle would be

. But the perimeter

is

, not

. Thus, the two equal sides cannot have length

The grid above shows the number of hours worked last week by 12 students of various ages at after-school jobs. Which of the following is true, according to this grid?
(A)

Half of the students worked more than 12 hours each.

(C) One 16 year old worked more than 11 hours.

(E)

Most of the students were under 16 years old.

ANSWERS

AND EXPLANATIONS

nR eg

Explanation for Correct Answer D : Choice (D) is correct. According to the information in the grid,

olds worked at least

hours each each. So more hours each.

Explanation for Incorrect Answer A : of the Choice (A) is not correct. According to the information in the grid, only hours each. So it is not true that half of the students worked more than hours each. students worked more than

is

te

year olds than

(D) More 18 year olds than 17 year olds worked at least 13 hours each.

re

of the

(B)

One student worked exactly 15 hours.

of the

year

hours each and

year olds worked at least

year olds worked at least

Explanation for Incorrect Answer B : Choice (B) is not correct. According to the information in the grid, none of the hours. So it is not true that one student worked students worked exactly

exactly

hours.

Explanation for Incorrect Answer C : Choice (C) is not correct. According to the information in the grid, the three

hours or less. So it is not true that one year olds each worked hours. worked more than

year old

Explanation for Incorrect Answer E : Choice (E) is not correct. According to the grid, there was only one student under years old. years old. So most of the students were not under

and 5 gives the same result as squaring the sum of Squaring the product of 5. Which of the following equations could be used to find all possible values of

and

file://E:\\f3.htm

2006-11-12

The Official SAT Online Course

5/15

(A)
(B)
(C)
(D)
(E)

ANSWERS

AND EXPLANATIONS
and

Explanation for Correct Answer D :

Choice (D) is correct. The square of the product of

is

, and the

square of the sum of

and

is

. If squaring the product of

and

gives the same result as squaring the sum of

and

, then the equation

is true. Solving this equation would give all possible values of

ed
and

Explanation for Incorrect Answer A :

Choice (A) is not correct. The square of the sum of

expression

is the product of

and the square of

correct values of

Explanation for Incorrect Answer B :

st er

. Thus, solving

product of

and

. Thus, solving the equation


is

, but the

, not the square of the

will not give the

Choice (B) is not correct. The square of the product of

and

is

, but the

eg i

and

expression

is the sum of the square of

and the square of

, not the

square of the sum of

for

will not give

the correct values of

nR

Explanation for Incorrect Answer C :

Choice (C) is not correct. The expression

is the product of the square of

and

, not the square of the product of

and

. The expression

is the

of

sum of the square of

and the square of

, not the square of the sum of

and

. Thus, solving

will not give the correct values of

Explanation for Incorrect Answer E :

Choice (E) is not correct. The expression

is the product of

and the square

, not the square of the product of

and

. The expression

is the

sum of the square of

and the square of

, not the square of the sum of

and

. Thus, solving

for

will not give the correct values of

If as many 7-inch pieces of wire as possible are cut from a wire that is 3 feet long, what is the total length of the wire that is left over? (12 inches = 1 foot)
(A)

1 inch

(B)

2 inches

(C) 3 inches

(D) 4 inches

(E)

5 inches

ANSWERS

AND EXPLANATIONS

file://E:\\f3.htm

2006-11-12

The Official SAT Online Course

6/15

Explanation for Correct Answer A : foot, a piece of wire that is inches is equal to Choice (A) is correct. Since -inch pieces of wire that can inches long. The greatest number of feet long is pieces pieces. The total length of these -inch-long wire is be cut from the -inch five inches. So the piece of wire left after cutting the of wire is inch long. pieces is

Explanation for Incorrect Answer B : -inch pieces of Choice (B) is not correct. The total length of wire left after cutting inches inches long. If it were -inch piece of wire cannot be wire from a inches. But -inch pieces together would measure long, then the

does not divide

evenly.

Explanation for Incorrect Answer C : -inch pieces of Choice (C) is not correct. The total length of wire left after cutting inches inches long. If it were -inch piece of wire cannot be wire from a inches. But together would measure inch pieces long, then the

does not divide

evenly.

Explanation for Incorrect Answer D : -inch pieces of Choice (D) is not correct. The total length of wire left after cutting inches inches long. If it were -inch piece of wire cannot be wire from a inches. But -inch pieces together would measure long, then the

does not divide

evenly.

nR

eg

is

te r

are

Explanation for Incorrect Answer E : -inch pieces of Choice (E) is not correct. The total length of wire left after cutting inches inches long. If it were -inch piece of wire cannot be wire from a inches. But -inch pieces together would measure long, then the

ed

does not divide

evenly.

Which of the lettered points in the figure above has coordinates

such that

(A)

(B)

(C) C

(D) D

(E)

ANSWERS

AND EXPLANATIONS
. Since

Explanation for Correct Answer B :

Choice (B) is correct. The coordinates of point

, point

has coordinates

such that

file://E:\\f3.htm

2006-11-12

The Official SAT Online Course

7/15

Explanation for Incorrect Answer A :

Choice (A) is not correct. The coordinates of point

are

. Since

, point

does not have coordinates

such that

Explanation for Incorrect Answer C :

Choice (C) is not correct. The coordinates of point

are

. Since

, point

does not have coordinates

such that

Explanation for Incorrect Answer D :

Choice (D) is not correct. The coordinates of point

are

. Since

, point

does not have coordinates

such that

Explanation for Incorrect Answer E :

Choice (E) is not correct. The coordinates of point

are

, point

does not have coordinates

10

III.

nR

If n is a member of both set A and set B above, which of the following must be true?
I.

II.

n is an integer.

4n is an integer.

(A)

None

(B)

II only

(C) I and II only

(D) I and III only

(E)

I, II, and III

ANSWERS

Explanation for Correct Answer A : is a member of both set Choice (A) is correct. If

eg

AND EXPLANATIONS
and set

is t

, then

er ed

is not an integer, so

. Since

such that

, then

must be

, or

. If

is not necessarily true. If

, then

, which is not an integer;

is not necessarily true. If

, so , then statements must be true.

is not necessarily true. Therefore, none of the three

file://E:\\f3.htm

2006-11-12

The Official SAT Online Course

8/15

Explanation for Incorrect Answer B : is a member of both set Choice (B) is not correct. If

and set

, then

must

be

, or

. If

, then

is not an integer, so

is not

necessarily true.

Explanation for Incorrect Answer C : is a member of both set Choice (C) is not correct. If

and set

, then

must

be

, or

. If

, then

is not an integer, so

is not necessarily true.

If

, then

, which is not an integer; so

is not necessarily true.

Neither statement

nor

must be true.

Explanation for Incorrect Answer D : is a member of both set Choice (D) is not correct. If

and set

, then

must be

, or

. If

, then

is not an integer, so

is not necessarily

, then true. If must be true.

, so

is not necessarily true. Neither statement

nor

Explanation for Incorrect Answer E : is a member of both set Choice (E) is not correct. If

If

, then

, which is not an integer; so

11

If y is directly proportional to x and if

nR eg

when

(A)

(B)

(C) 23

(D) 27

(E)

30

is

te r

when

, so , then statements must be true.

is not necessarily true. Therefore, none of the three

ANSWERS

AND EXPLANATIONS
, then there is a constant

Explanation for Correct Answer E : is directly proportional to Choice (E) is correct. If

ed

when
, then

be

, or

. If

, then

is not an integer, so


and set

, then

must

is not necessarily true.

is not necessarily true. If

what is the value of y

such that

. If

, so

and

. When

, the value of

is

Explanation for Incorrect Answer A : is directly proportional to Choice (A) is not correct. If

, then there is a

constant

such that

. If

when

, then

, so

. This is not the answer to the problem, because you must find the value of

when

. This is

Explanation for Incorrect Answer B :

file://E:\\f3.htm

2006-11-12

The Official SAT Online Course

9/15

Choice (B) is not correct. If

is directly proportional to

, then there is a constant

such that

. If

when

, then

, so

and

. When

, the value of

cannot be

, because

Explanation for Incorrect Answer C : is directly proportional to Choice (C) is not correct. If

, then there is a constant

such that

. If

when

, then

, so

and

. When

, the value of

cannot be

, because

Explanation for Incorrect Answer D : is directly proportional to Choice (D) is not correct. If

, then there is a constant

such that

. If

when

, then

, so

and

. When

, the value of

cannot be

(A)

(B)

16

(D) 23

(E)

24

nR

ANSWERS

the

Explanation for Correct Answer C : th term of a sequence is defined to be Choice (C) is correct. If the

eg

(C) 20

AND EXPLANATIONS
, then

is te

. So the

12

The nth term of a sequence is defined to be greater than the 45th term?

re

is

th term of the sequence is

sequence is

th term of the sequence is greater than

the

th term by

Explanation for Incorrect Answer A : Choice (A) is not correct. The number

d
. The


, because

The 50th term is how much

th term of the

greater than the number

, but

the

th term of the sequence,

, is greater than the

th term of the

sequence,

, by

Explanation for Incorrect Answer B : th term of the sequence is Choice (B) is not correct. The

, and the

th

term of the sequence is

. The difference between these numbers is not

Explanation for Incorrect Answer D : th term of the sequence is Choice (D) is not correct. The

, and the

th

term of the sequence is

. The difference between these numbers is not

Explanation for Incorrect Answer E : th term of the sequence is Choice (E) is not correct. The

, and the

th

term of the sequence is

. The difference between these numbers is not

file://E:\\f3.htm

2006-11-12

The Official SAT Online Course

10/15

13

Which of the rectangular solids shown above has a volume closest to the volume of a right circular cylinder with radius 2 and height 4?
(A)

(B)

(C) C

(D) D

(E)

ANSWERS

AND EXPLANATIONS

Explanation for Correct Answer E : Choice (E) is correct. The volume of a right circular cylinder with radius

height

is

. Since

is close to

ed

and

rectangular solid

, which is

, is slightly less than

about

. Solids

and

, with respective volumes of

small; solids

and

, with respective volumes of

the volume of the cylinder is closest to the volume of solid


and

and

, the volume of

, which is

are too

, are too large. So

eg is

Explanation for Incorrect Answer A : Choice (A) is not correct. The volume of the cylinder is about

close to the volume of solid small.

te r

, which is

, which is very

. Solid

, with a volume of

, is too

Explanation for Incorrect Answer B : Choice (B) is not correct. The volume of the cylinder is about

, which is very

nR

close to the volume of solid small.

, which is

. Solid

, with a volume of

, is too

Explanation for Incorrect Answer C : Choice (C) is not correct. The volume of the cylinder is about

close to the volume of solid large.

, which is very

, which is

. Solid

, with a volume of

, is too

Explanation for Incorrect Answer D : Choice (D) is not correct. The volume of the cylinder is about

, which is very

close to the volume of solid large.

, which is

. Solid

, with a volume of

, is too

14

If x is a negative integer, what is the ordering of j, k, and m from least to greatest?


(A)
(B)
(C)
(D)
(E)

file://E:\\f3.htm

2006-11-12

The Official SAT Online Course

11/15

ANSWERS

AND EXPLANATIONS
is a negative integer, then

Explanation for Correct Answer D :

Choice (D) is correct. If

is negative

because

is negative. Since

and

are positive,

it follows that

and

. Since

is negative, it follows that

, so

. Therefore,

. Putting the inequalities

and

together gives

Explanation for Incorrect Answer A : is a negative integer, then Choice (A) is not correct. If

, since

is

negative and

is positive. So the inequality

cannot be true.

Explanation for Incorrect Answer B :

Choice (B) is not correct. If

is a negative integer, then

, since

is

negative and

is positive. So the inequality

ed

, and

Explanation for Incorrect Answer C : is a negative integer, then Choice (C) is not correct. If

negative and

is positive. So the inequality

er

Explanation for Incorrect Answer E :

cannot be true.

, since

is

cannot be true.

Choice (E) is not correct. If

is a negative integer, then

, so

. Therefore,

. So the inequality

15

Flour, water, and salt are mixed by weight in the ratio of 5:4:1, respectively, to produce a certain type of dough. In order to make 5 pounds of this dough, what weight of salt, in pounds, is required?

nR
(A)
(B)

(C)

(D) 1

(E)

ANSWERS

Explanation for Correct Answer B : Choice (B) is correct. The flour, water, and salt are mixed by weight in the ratio pounds of this pounds of salt to make , respectively. So if it requires

eg is t
cannot be true.

AND EXPLANATIONS

dough, then

; so

Explanation for Incorrect Answer A : Choice (A) is not correct. See the explanation for the correct response (B).

Explanation for Incorrect Answer C : Choice (C) is not correct. See the explanation for the correct response (B).

Explanation for Incorrect Answer D :

file://E:\\f3.htm

2006-11-12

The Official SAT Online Course

12/15

Choice (D) is not correct. See the explanation for the correct response (B).

Explanation for Incorrect Answer E :

Choice (E) is not correct. As explained for the correct response (B),

pounds of the dough. But the question pounds of water are required to make asks for the number of pounds of salt, not of water, needed to make the dough.

16

In rectangle ABCD above, the area of the shaded region is given by

If the area

what is the total area, to the nearest whole number, of of the shaded region is the unshaded regions of rectangle ABCD?
(A)

(B)

(C) 8

(D) 9

(E)

10

ANSWERS

AND EXPLANATIONS

eg

Explanation for Correct Answer B : is Choice (B) is correct. The total area of the unshaded regions of rectangle equal to the area of the entire rectangle minus the area of the shaded region. Since

nR

the area of the shaded region is

is

te

re

is too large.

d
, it follows that

. So the area

of the rectangle is

area of the unshaded regions is

, and the area of the shaded region is

. Thus, the total

, which is approximately

Explanation for Incorrect Answer A : Choice (A) is not correct. The total area of the unshaded regions is

, which

is approximately

. The answer of

is too small.

Explanation for Incorrect Answer C : Choice (C) is not correct. The total area of the unshaded regions is

, which

is approximately

. The answer of

is too large.

Explanation for Incorrect Answer D : Choice (D) is not correct. The total area of the unshaded regions is

, which

is approximately

. The answer of

is too large.

Explanation for Incorrect Answer E : Choice (E) is not correct. The total area of the unshaded regions is

, which

is approximately

. The answer of

17

The city library donated some childrens books to Mr. Clarks first-grade class. If each student takes 4 books, there will be 20 books left. If 3 students do not take a book and the rest of the students take 5 books each, there will be no books left. How many books were donated to the class?

file://E:\\f3.htm

2006-11-12

The Official SAT Online Course

13/15

(A)

120

(B)

140

(C) 160

(D) 175

(E)

185

ANSWERS

AND EXPLANATIONS

Explanation for Correct Answer C : books will be students in the class and if Choice (C) is correct. If there are . If books, then the number of books is left after each student takes

students do not take a book and the rest take

books each, there will be no books

left. So the number of books is also given by

. So the number of books is

given by both

and

. Solving the equation

for

gives

. So the number of books is

Explanation for Incorrect Answer A : Choice (A) is not correct. See the explanation for the correct response (C).

Explanation for Incorrect Answer E : Choice (E) is not correct. See the explanation for the correct response (C).

18

nR eg

is
is

In the figure above, if line

has a slope of

(A)

(B)

(C) 9

(D) 10

(E)

12

ANSWERS

AND EXPLANATIONS
lies on line

Explanation for Correct Answer D :

Choice (D) is correct. From the figure, you can see that the point

te r

lies on

Explanation for Incorrect Answer D : Choice (D) is not correct. See the explanation for the correct response (C).

ed

is

Explanation for Incorrect Answer B : Choice (B) is not correct. See the explanation for the correct response (C).

what is the y-intercept of

. The slope of line

, so the equation of line

, where

is the

-intercept of

. Since

, it follows that

Solving this equation for

gives

file://E:\\f3.htm

2006-11-12

The Official SAT Online Course

14/15

Explanation for Incorrect Answer A : has slope Choice (A) is not correct. Line

; if the

-intercept of

were

then

would have the equation

. But then the point

would not

be on

Explanation for Incorrect Answer B : has slope Choice (B) is not correct. Line

; if the

-intercept of

were

then

would have the equation

. But then the point

would not

be on

Explanation for Incorrect Answer C : has slope Choice (C) is not correct. Line

; if the

-intercept of

were

then

would have the equation

. But then the point

would not

be on

Explanation for Incorrect Answer E : has slope Choice (E) is not correct. Line

; if the

-intercept of

were

then

would have the equation

. But then the point

would

not be on

re d

. So

19

For all positive integers w and y, where

let the operation

For how many positive integers w is


(A)

be defined by

equal to 4 ?

None

(B)

One

(C) Two

(D) Four

ANSWERS

nR

Explanation for Correct Answer E : is defined for all positive integers Choice (E) is correct. If

eg

(E)

More than four

AND EXPLANATIONS
and

is
, then

te

, where

, to be

for all positive

integers integers

greater than . Therefore, . , which is more than

for infinitely many positive

Explanation for Incorrect Answer A : Choice (A) is not correct. See the explanation for the correct response (E).

Explanation for Incorrect Answer B : Choice (B) is not correct. See the explanation for the correct response (E).

Explanation for Incorrect Answer C : Choice (C) is not correct. See the explanation for the correct response (E).

Explanation for Incorrect Answer D : Choice (D) is not correct. See the explanation for the correct response (E).

20

file://E:\\f3.htm

2006-11-12

The Official SAT Online Course

15/15

The figure above represents four offices that will be assigned randomly to four employees, one employee per office. If Karen and Tina are two of the four employees, what is the probability that each will be assigned an office indicated with an X ?

(A)
(B)
(C)

(D)

(E)

ANSWERS

AND EXPLANATIONS

ways. So the way to assign the fourth; this is third, leaving probability that Karen and Tina will each be assigned an office with an X is

nR eg

Explanation for Incorrect Answer A : Choice (A) is not correct. See the explanation for the correct response (C).

Explanation for Incorrect Answer B : Choice (B) is not correct. This is the probability that Karen will be assigned the leftmost office and Tina will be assigned the rightmost office. But Karen could be assigned the rightmost office and Tina the leftmost office. See the explanation for the correct response (C).

Explanation for Incorrect Answer D : Choice (D) is not correct. See the explanation for the correct response (C).

Explanation for Incorrect Answer E : Choice (E) is not correct. See the explanation for the correct response (C).

is t

er

Explanation for Correct Answer C : Choice (C) is correct. The probability that Karen and Tina each will be assigned an office indicated with an X is the number of ways this can happen divided by the total number of ways of assigning the four offices to the four employees. If Karen ways to has the leftmost office and Tina has the rightmost office, there are assign the other two offices. Similarly, if Tina has the leftmost office and Karen has ways to assign the other two offices. Thus, there the rightmost office, there are ways to assign Karen and Tina offices marked with an X. Now count are ways to assign the total number of ways of assigning the four offices: there are ways to assign the ways to assign the second, leaving the first office, leaving

ed

Privacy Policy

Back to Score Report

Copyright 2006 The College Board. All rights reserved.

Terms of Use

Contact Us

file://E:\\f3.htm

2006-11-12

The Official SAT Online Course

1/20

Help | Profile | My Organizer | My Bookmarks | Logout

Answers and Explanations

Test Sections

Back to Score Report

Section 1

View Answers and Explanations


Online - Practice Test #6

Section 2

Section 4

Section 5

Edmund White is a ------- author: he has written novels, essays, short stories, a travel book, and a biography.
(A)

Section 6

Section 7

demonstrative

Section 8

(B)

nebulous

Section 9

(C) meticulous

Section 10

(D) versatile

(E)

metaphoric

ANSWERS

AND EXPLANATIONS

nR

Explanation for Incorrect Answer A : Choice (A) is incorrect. Someone who is "demonstrative" is given to open expressions of emotion. If one were to insert this term into the text, the sentence would read "Edmund White is a demonstrative author: he has written novels, essays, short stories, a travel book, and a biography." This term does not fit within the context of the sentence, because an emotional author will not necessarily write many different kinds of works.

Explanation for Incorrect Answer B : Choice (B) is incorrect. "Nebulous" means vague or indistinct. If one were to insert this term into the text, the sentence would read "Edmund White is a nebulous author: he has written novels, essays, short stories, a travel book, and a biography." A list of the different genres in which an author writes offers no support for a claim that the author is vague.

Explanation for Incorrect Answer C : Choice (C) is incorrect. "Meticulous" means extremely careful or precise. If one were to insert this term into the text, the sentence would read "Edmund White is meticulous author: he has written novels, essays, short stories, a travel book, and a biography." While a meticulous author may write a number of different things, there is no inherent connection between these two ideas.

Explanation for Incorrect Answer E : Choice (D) is incorrect. "Metaphoric" means relating to metaphors or figures of speech that make an implicit comparison. If one were to insert this term into the text, the sentence would read "Edmund White is metaphoric author: he has written novels, essays, short stories, a travel book, and a biography." The fact that White has written several different types of literature is unrelated to whether or not he writes metaphorically. All of the genres listed could be written perflectly well with or without the use of metaphors.

The archaeologist believed the coin she unearthed was ------- evidence, unquestionable proof that the site dated to the fourth century.

eg is te r

Explanation for Correct Answer D : Choice (D) is correct. Someone who is "versatile" embraces a variety of subjects, fields, or skills. If one were to insert this term into the text, the sentence would read "Edmund White is a versatile author: he has written novels, essays, short stories, a travel book, and a biography." The colon indicates that the information listed in the second part of the sentence will support the claim made in the first part. A writer who dabbles in so many genres can certainly be described as "versatile."

ed

file://E:\\f4.htm

2006-11-12

The Official SAT Online Course

2/20

(A)

immaterial

(B)

potential

(C) incriminating

(D) nominal

(E)

indisputable

ANSWERS

AND EXPLANATIONS

Explanation for Correct Answer E : Choice (E) is correct. "Indisputable" means unquestionable. If one were to insert this term into the text, the sentence would read "The archaeologist believed the coin she unearthed was indisputable evidence, unquestionable proof that the site dated to the fourth century." The comma in the sentence separates two synonymous phrases. Only "indisputable evidence" has the same meaning as "unquestionable proof."

nR

(A)

Explanation for Incorrect Answer C : Choice (C) is incorrect. Something that is "incriminating" shows evidence of a crime or fault. If one were to insert this term into the text, the sentence would read "The archaeologist believed the coin she unearthed was incriminating evidence, unquestionable proof that the site dated to the fourth century." The term "incriminating" is inappropriate in this sentence because the coin does not prove anyone guilty of wrongdoing.

Explanation for Incorrect Answer D : Choice (D) is incorrect. "Nominal" means insignificant. If one were to insert this term into the text, the sentence would read "The archaeologist believed the coin she unearthed was nominal evidence, unquestionable proof that the site dated to the fourth century." "Nominal," or skimpy, evidence would never be described as "unquestionable proof."

Although the rigors of ballet dancing are primarily -------, this art is also emotionally and spiritually -------.

illusory . . taxing

(B)

exaggerated . . balanced

(C) physical . . demanding

(D) appealing . . indulgent

(E)

strenuous . . dubious

ANSWERS

Explanation for Correct Answer C : Choice (C) is correct. "Physical" means something involving or characterized by vigorous bodily activity. "Demanding" in this context means requiring much effort or attention. If one were to insert these terms into the text, the sentence would read "Although the rigors of ballet dancing are primarily physical, this art is also emotionally and spiritually demanding." The words "although" and "also" indicate that the sentence will compare two related ideas. In this case, they set up a logical

eg

AND EXPLANATIONS

is

te

Explanation for Incorrect Answer B : Choice (B) is incorrect. "Potential" means possible. If one were to insert this term into the text, the sentence would read "The archaeologist believed the coin she unearthed was potential evidence, unquestionable proof that the site dated to the fourth century." A discovery that could potentially provide evidence about an archaeological site is a far cry from "unquestionable proof."

re

Explanation for Incorrect Answer A : Choice (A) is incorrect. "Immaterial" means unimportant. If one were to insert this term into the text, the sentence would read "The archaeologist believed the coin she unearthed was immaterial evidence, unquestionable proof that the site dated to the fourth century." The sentence implies that the archaeologist's discovery is significant, so it would be illogical to call the evidence "immaterial."

file://E:\\f4.htm

2006-11-12

The Official SAT Online Course

3/20

contrast between ballet's physical and non-physical demands.

Explanation for Incorrect Answer A : Choice (A) is incorrect. "Illusory" refers to something that is based on illusion or that is deceptive. "Taxing" means burdensome or difficult. If one were to insert these terms into the text, the sentence would read "Although the rigors of ballet dancing are primarily illusory, this art is also emotionally and spiritually taxing." The first clause of this sentence implies that ballet is not as physically challenging as it appears, so it does not make sense to then claim that its emotional and spiritual elements are "also . . . taxing," or difficult.

Explanation for Incorrect Answer B : Choice (B) is incorrect. "Exaggerated" means overstated. "Balanced" means equal or equivalent. If one were to insert these terms into the text, the sentence would read "Although the rigors of ballet dancing are primarily exaggerated, this art is also emotionally and spiritually balanced." The claim that ballet is emotionally and spiritually balanced is in no way related to the claim that its rigors are exaggerated.

Explanation for Incorrect Answer D : Choice (D) is incorrect. "Appealing" means attractive or inviting, and "indulgent" means lenient. If one were to insert these terms into the text, the sentence would read "Although the rigors of ballet dancing are primarily appealing, this art is also emotionally and spiritually indulgent." It makes little sense to describe the rigors of ballet as attractive, especially in connection to a claim about ballet's emotional and spiritual "indulgence."

nR eg
(A)

Studies of ------- among turtles are sometimes ------- by the fact that the subjects live so long that researchers retire before the studies can be completed.

(B)

(C) behavior. . belied

(D) mortality. . bolstered

extinction. . enhanced

longevity. . hampered

(E)

reproduction. . confirmed

ANSWERS

AND EXPLANATIONS

Explanation for Correct Answer B : Choice (B) is correct. "Longevity" means of great age or great span of life. "Hampered" means to impede in motion or progress. If one were to insert these terms into the text, the sentence would read "Studies of longevity among turtles are sometimes hampered by the fact that the subjects live so long that researchers retire before the studies can be completed." The sentence explains that the lengthy lifespan of turtles can cause problems for researchers. "Longevity" fits the description "lives so long." That turtles outlive the careers of the scientists studying them would certainly hamper, or impede, the study.

Explanation for Incorrect Answer A : Choice (A) is incorrect. "Extinction" means the process of becoming extinct. "Enhanced" means improved, or advanced in qualities. If one were to insert these terms into the text, the sentence would read "Studies of extinction among turtles are sometimes enhanced by the fact that the subjects live so long that researchers retire before the studies can be completed." It is illogical to claim that a study could be enhanced, or improved, by the inability of researchers to complete it.

is

te r

Explanation for Incorrect Answer E : Choice (E) is incorrect. "Strenuous" means requiring great effort, energy, and exertion. "Dubious" means fraught with uncertainty or doubt. If one were to insert these terms into the text, the sentence would read "Although the rigors of ballet dancing are primarily strenuous, this art is also emotionally and spiritually dubious." It is somewhat redundant to describe rigors as strenuous, and this claim is unrelated to the assertion that ballet is emotionally and spiritually "dubious," or doubtful.

ed

file://E:\\f4.htm

2006-11-12

The Official SAT Online Course

4/20

Explanation for Incorrect Answer C : Choice (C) is incorrect. "Behavior" means the manner in which something functions. "Belied" means pictured falsely or misrepresented. If one were to insert these terms into the text, the sentence would read "Studies of behavior among turtles are sometimes belied by the fact that the subjects live so long that researchers retire before the studies can be completed." It makes some sense to say that a study could be belied, or misrepresented, by the problem of researchers retiring, but it is unclear why a behavior study would be affected by this problem.

Explanation for Incorrect Answer D : Choice (D) is incorrect. "Mortality" means death rate. "Bolstered" means supported or propped up with. If one were to insert these terms into the text, the sentence would read "Studies of mortality among turtles are sometimes bolstered by the fact that the subjects live so long that researchers retire before the studies can be completed." A study on turtle mortality would not be bolstered, or propped up by, the retirement of the researcher.

Explanation for Incorrect Answer E : Choice (E) is incorrect. "Reproduction" means the act of reproducing. "Confirmed" means strengthened or made valid. If one were to insert these terms into the text, the sentence would read "Studies of reproduction among turtles are sometimes confirmed by the fact that the subjects live so long that researchers retire before the studies can be completed." The fact of researchers retiring would do nothing to confirm, or prove, anything about reproduction.

(A)

temperate

(B)

laconic

(C) duplicitous

(E)

voluble

ANSWERS

AND EXPLANATIONS

nR

Explanation for Correct Answer A : Choice (A) is correct. "Temperate" in this context refers to moderation in the indulgence of appetite. If one were to insert this term into the text, the sentence would read "A model of temperate behavior, Cunningham never ate or drank to excess." The comma indicates that the second half of the sentence will illustrate Cunningham's behavior. Someone who never consumes too much does indeed exhibit "temperate behavior."

Explanation for Incorrect Answer B : Choice (B) is incorrect. Someone who is "laconic" speaks using a minimum of words. If one were to insert this term into the text, the sentence would read "A model of laconic behavior, Cunningham never ate or drank to excess." A tendency for reserved speech has nothing to do with a moderate appetite.

Explanation for Incorrect Answer C : Choice (C) is incorrect. "Duplicitous" behavior is marked by deceptiveness. If one were to insert this term into the text, the sentence would read "A model of duplicitous behavior, Cunningham never ate or drank to excess." Deceptiveness is not necessarily related to eating and drinking habits.

Explanation for Incorrect Answer D : Choice (D) is incorrect. "Aesthetic" means appreciative of beauty. If one were to insert this term into the text, the sentence would read "A model of aesthetic behavior, Cunningham never ate or drank to excess." One's appreciation of beauty does not necessarily affect one's appetite.

Explanation for Incorrect Answer E : Choice (E) is incorrect. Someone who is "voluble" is marked by a ready flow of speech. If one were to insert this term into the text, the sentence would read "A model of voluble behavior, Cunningham never ate or drank to excess." It is illogical to claim that a talkative person always eats and drinks in moderation.

eg

is te

(D) aesthetic

re d

A model of ------- behavior, Cunningham never ate or drank to excess.

file://E:\\f4.htm

2006-11-12

The Official SAT Online Course

5/20

The entrepreneur had a well-deserved reputation for -------, having accurately anticipated many changes unforeseen by established business leaders.
(A)

prescience

(B)

sincerity

(C) avarice

(D) complicity

(E)

mendacity

ANSWERS

AND EXPLANATIONS

Explanation for Correct Answer A : Choice (A) is correct. "Prescience" means foresight. If one were to insert this term into the text, the sentence would read "The entrepeneur had a well-deserved reputation for prescience, having accurately anticipated many changes unforeseen by established business leaders." The part of the sentence following the comma describes the missing term. It is logical for a person who can frequently predict "unforeseen" events to develop a reputation for "prescience."

nR

(A)

Explanation for Incorrect Answer D : Choice (D) is incorrect. "Complicity" refers to a person's participation in a questionable act or a crime. If one were to insert this term into the text, the sentence would read "The entrepeneur had a well-deserved reputation for complicity, having accurately anticipated many changes unforeseen by established business leaders." This sentence offers no evidence that the entrepreneur has a reputation for conniving behavior

Explanation for Incorrect Answer E : Choice (E) is incorrect. "Mendacity" means untruthfulness. If one were to insert this term into the text, the sentence would read "The entrepeneur had a well-deserved reputation for mendacity, having accurately anticipated many changes unforeseen by established business leaders." The description in the second half of this sentence does not support the claim that the entrepreneur was dishonest.

Scientists require observable data, not -------, to support a hypothesis; sound science is grounded in ------- results rather than speculation.

induction . . diminutive

(B)

experimentation . . pragmatic

(C) intuition . . fiscal

(D) bombast . . theoretical

(E)

conjecture . . empirical

ANSWERS

Explanation for Correct Answer E : Choice (E) is correct. "Conjecture" means guesswork or predicting from incomplete or uncertain evidence. "Empirical" means by experiment or according to experience. If one were to insert these terms into the text, the sentence would read "Scientists

eg

Explanation for Incorrect Answer C : Choice (C) is incorrect. "Avarice" means greed. If one were to insert this term into the text, the sentence would read "The entrepeneur had a well-deserved reputation for avarice, having accurately anticipated many changes unforeseen by established business leaders." The entrepreneur may be a greedy person, but this sentence describes a different trait.

AND EXPLANATIONS

is

te

re

Explanation for Incorrect Answer B : Choice (B) is incorrect. "Sincerity" means genuineness and honesty. If one were to insert this term into the text, the sentence would read "The entrepeneur had a welldeserved reputation for sincerity, having accurately anticipated many changes unforeseen by established business leaders." Sincerity is not linked to one's ability to anticipate changes.

file://E:\\f4.htm

2006-11-12

The Official SAT Online Course

6/20

require observable data, not conjecture, to support a hypothesis; sound science is grounded in empirical results rather than speculation." It makes sense to contrast "observable data" with "conjecture" and "empirical" with "speculation." Scientific criteria usually require hypotheses to be grounded in "empirical results" and "observable data," not "conjecture" and "speculation."

Explanation for Incorrect Answer A : Choice (A) is incorrect. "Induction" means the process of deriving general principles from particular facts or instances. "Diminutive" means tiny. If one were to insert these terms into the text, the sentence would read "Scientists require observable data, not induction, to support a hypothesis; sound science is grounded in diminutive results rather than speculation." It makes no sense to say that sound science is grounded in "diminutive," or tiny, results. Nor does it make sense to contrast tiny results with "speculation."

Explanation for Incorrect Answer B : Choice (B) is incorrect. "Experimentation" means the act of testing under controlled conditions. "Pragmatic" means practical. If one were to insert these terms into the text, the sentence would read "Scientists require observable data, not experimentation, to support a hypothesis; sound science is grounded in pragmatic results rather than speculation." "Experimentation" can provide "observable data"; the two are not opposites.

nR eg
(A)

Explanation for Incorrect Answer D : Choice (D) is incorrect. "Bombast" means pompous speech or writing. "Theoretical" means based on theory. If one were to insert these terms into the text, the sentence would read "Scientists require observable data, not bombast, to support a hypothesis; sound science is grounded in theoretical results rather than speculation." "Theoretical results" can be described as speculative; however, theoretical results and speculation are not mutually exclusive.

The director complained that the sitcoms theme song was downright -------, having no more pep and vigor than a -------.

tedious . . jingle

(B)

inchoate . . lullaby

(C) lugubrious . . dirge

(D) facetious . . ballad

(E)

sprightly . . eulogy

ANSWERS

AND EXPLANATIONS

Explanation for Correct Answer C : Choice (C) is correct. "Lugubrious" means mournful, and a "dirge" is a song of grief. If one were to insert these terms into the text, the sentence would read "The director complained that the sitcom's theme song was downright lugubrious, having no more pep and vigor than a dirge." In this sentence, the information following the comma supports the claim preceding the comma. The director's complaint that the song was "lugubrious" is perfectly supported by calling it a "dirge."

Explanation for Incorrect Answer A : Choice (A) is incorrect. "Tedious" means boring. "Jingle" is a piece of light singsong verse or rhyme. If one were to insert these terms into the text, the sentence would read "The director complained that the sitcom's theme song was downright tedious, having no more pep and vigor than a jingle." A "jingle" is a light or catchy song,

is te r

Explanation for Incorrect Answer C : Choice (C) is incorrect. "Intuition" means immediate cognition. "Fiscal" means relating to finances. If one were to insert these terms into the text, the sentence would read "Scientists require observable data, not intuition, to support a hypothesis; sound science is grounded in fiscal results rather than speculation." While "intuition" is clearly distinguished from "observable data," science is supposed to rely on the physical results of experimentation regardless of the "fiscal," or financial, results the experimentation may involve.

ed

file://E:\\f4.htm

2006-11-12

The Official SAT Online Course

7/20

thus it makes no sense to complain that a song has no more pep than a jingle.

Explanation for Incorrect Answer B : Choice (B) is incorrect. "Inchoate" means in an initial or early stage. "Lullaby" is a soothing song with which to lull a child to sleep. If one were to insert these terms into the text, the sentence would read "The director complained that the sitcom's theme song was downright inchoate, having no more pep and vigor than a lullaby." It does not make complete sense to complain that a song lacks "pep and vigor" because it is in its early stages of development.

Explanation for Incorrect Answer D : Choice (D) is incorrect. "Facetious" means joking or humorous. "Ballad" means a song of a romantic or sentimental nature. If one were to insert these terms into the text, the sentence would read "The director complained that the sitcom's theme song was downright facetious, having no more pep and vigor than a ballad." A facetious song would most likely be witty or joking, and would not likely be lacking pep. Furthermore, many ballads could be described as vigorous.

Explanation for Incorrect Answer E : Choice (E) is incorrect. "Sprightly" means full of spirit and vitality. "Eulogy" is a speech praising someone who has died. If one were to insert these terms into the text, the sentence would read "The director complained that the sitcom's theme song was downright sprightly, having no more pep and vigor than a eulogy." A sprightly, or lively, song would certainly have more pep than a eulogy, or funeral speech.

In between school days, we gathered hazelnuts, fished, had long deer-hunting weekends, went to powwows, beaded on looms, and made quilts. I did not Line question the necessity or value of our school education, but somehow I grew up knowing it 5 wasnt the only education I would need. Im thankful for those experiences of my Anishinaabe heritage, because now I know by heart not only the national anthem, but the ancient song of the loon. I recognize not only the alphabet and the parts of an English sentence, but 10 the intricate language of a beavers teeth and tail.

nR

(A)

The main idea of the passage is that the author

preferred certain academic subjects over others

(B)

succeeded in learning to speak many foreign languages

(C) valued knowledge of the natural world more than book learning

(D) loved both family trips and tribal activities

(E)

learned many important things both in and out of school

ANSWERS

Explanation for Correct Answer E : Choice (E) is correct. As the author explains in the text, "No one questioned the necessity or value of our school education, but . . . it wasn't the only education I would need." References to "deer-hunting weekends" and "powwows" make it clear

eg i

AND EXPLANATIONS

st

er

ed

file://E:\\f4.htm

2006-11-12

The Official SAT Online Course

8/20

that the author received an important cultural education outside the classroom.

Explanation for Incorrect Answer A : Choice (A) is incorrect. The author indicates that education can take on different forms but does not express a preference for any academic subject.

Explanation for Incorrect Answer B : Choice (B) is incorrect. While the author claims to have learned the "song of the loon" and the "intricate language of a beaver's teeth and tail," the only spoken language mentioned in the passage is English.

Explanation for Incorrect Answer C : Choice (C) is incorrect. The author implies that both forms of education described in the passage are important but does not indicate a preference for one over the other.

Explanation for Incorrect Answer D : Choice (D) is incorrect. The passage compares what the author learned both in school and outdoors. "Family trips and tribal activities" account for only the latter.

10

The authors overall tone in this passage is best described as one of

(C) curiosity

(D) appreciation

(E)

uncertainty

ANSWERS

AND EXPLANATIONS

nR

Explanation for Incorrect Answer A : Choice (A) is incorrect. While the author is clearly pleased to have received two complementary forms of education, the tone of the passage never elevates to a level of "jubilation," or rejoicing.

Explanation for Incorrect Answer B : Choice (B) is incorrect. The author expresses "frustration" in neither content nor tone.

Explanation for Incorrect Answer C : Choice (C) is incorrect. While the author seems to enjoy learning, his primary tone is not one of "curiosity."

Explanation for Incorrect Answer E : Choice (E) is incorrect. The author writes with confidence, not "uncertainty."

Daily life is overflowing with mundane mental events. A paper clip gleams amid stacks of documents, a friends face shines like a beacon out of a crowd, the smell of freshly baked bread evokes childhood
file://E:\\f4.htm

eg

Explanation for Correct Answer D : Choice (D) is correct. The author is clearly "thankful" (line 6) for having had the opportunity to supplement a school education with outdoor lessons.

is

te

re

(B)

frustration

(A)

jubilation

2006-11-12

The Official SAT Online Course

9/20

Line memoriesthoughts
5

and perceptions such as these flow by with monotonous ease. So it seems, anyway. Yet given what scientists know about how brains work, even the ability to perceive a paper clip on a messy desk represents an extraordinary 10 and mysterious achievement.
In the first paragraph, the author implicitly likens our experience of sensory impressions to
(A)

11

an emotional roller coaster

(B)

an unobstructed stream

(C) a repeated image

(D) a nostalgic reminiscence

(E)

a diverting daydream

ANSWERS

AND EXPLANATIONS

Explanation for Incorrect Answer A : Choice (A) is incorrect. The author's description of "mundane" and "monotonous" thoughts directly contrasts with the turbulence associated with "an emotional roller coaster."

12

nR

(A)

Explanation for Incorrect Answer C : Choice (C) is incorrect. The author lists several different images in the first paragraph but does not suggest that any of them are repetitive.

Explanation for Incorrect Answer D : Choice (D) is incorrect. While the author asserts that particular perceptions can evoke old memories, it would be inaccurate to claim that all thoughts are linked to nostalgia.

Explanation for Incorrect Answer E : Choice (E) is incorrect. The author implies that most daily perceptions are boring, not "diverting," or amusing.

The primary purpose of the passage is to

celebrate lifes mundane but gratifying pleasures

(B)

convey the overwhelming confusion of every-day life

(C) explore the biological implications of a persons decisions

(D) suggest the complexity of perceptual processes

(E)

present a scientific analysis of an automatic reflex

ANSWERS

Explanation for Correct Answer D : Choice (D) is correct. The passage mainly serves to highlight the "complexity of

eg

AND EXPLANATIONS

is t

er

Explanation for Correct Answer B : Choice (B) is correct. Suggestions that "life is overflowing with mundane mental events" and that "thoughts and perceptions . . . flow by" produce an implicit comparison between people's daily experiences and an "unobstructed stream."

ed

file://E:\\f4.htm

2006-11-12

The Official SAT Online Course


perceptual processes" that might otherwise be taken for granted.

10/20

Explanation for Incorrect Answer A : Choice (A) is incorrect. While the author seems to marvel at the underlying complexity of everyday perceptions, the passage's tone is more analytical than celebratory.

Explanation for Incorrect Answer B : Choice (B) is incorrect. Far from "convey[ing] the overwhelming confusion of everyday life," the passage in fact shows how our perceptions render everyday life easy and "monotonous."

Explanation for Incorrect Answer C : Choice (C) is incorrect. The passage does not discuss people's decisions, only their perceptions.

Explanation for Incorrect Answer E : Choice (E) is incorrect. The passage does allude to the scientific study of perception, but it does not present actual "scientific analysis" of any kind.

Now, what are we to make of this sputtering debate, in which charges of imperialism are met by equally passionate accusations of vandalism, in which each side hates the other, and yet each seems to have its Line share of reason? It occurs to me that perhaps what we 5 have here is one of those debates in which the opposing sides, unbeknownst to themselves, share a myopia that will turn out to be the most interesting and important feature of the whole discussion, a debate, for instance, like that of the Founding Fathers over the nature of the 10 franchise. Think of all the energy and passion spent debating the question of property qualifications, or direct versus legislative elections, while all along, unmentioned and unimagined, was the factto us so centralthat women and slaves 15 were never considered for any kind of vote. While everyone is busy fighting over what should be taught in the classroom, something is being

nR

eg i

st er

ed

The following passage is from a 1991 essay that discusses the debate over which authors should be taught in English classes.

file://E:\\f4.htm

2006-11-12

The Official SAT Online Course

11/20

nR

overlooked. That is the state of reading, and books, and literature in our country, at this time. Why, ask yourself, is everyone so hot under the collar about what to put on the 20 required-reading shelf? It is because, while we have been arguing so fiercely about which books make the best medicine, the patient has been slipping deeper and deeper into a coma. Let us imagine a country in which reading was a popular voluntary activity. There, parents read books 25 for their own edification and pleasure and are seen by their children at this silent and mysterious pastime. These parents also read to their children, give them books for presents, talk to them about books, and underwrite, with their taxes, a public library system that is open all day, every day. 30 In school, the children study certain books together but also have an active reading life of their own. Years later, it may even be hard for them to remember if they read Jane Eyre at

home and Judy Blume in class or the other way around. In college, young people continue to be 35 assigned certain books, but far more important are the books they discover for themselves browsing in the library, in bookstores, on the shelves of friends, one book leading to another, back and forth in history and across languages and cultures. After graduation, they continue to read and in 40 the fullness of time produce a new generation of readers. Oh happy land! I wish we all lived there. In that country of real readers, voluntary, active, selfdetermined readers, a debate like the current

eg

is

te

re

file://E:\\f4.htm

2006-11-12

The Official SAT Online Course

12/20

one over the canon would not be taking place. Or if it did, it 45 would be as a kind of parlor game: What books would you take to a desert island? Everyone would know that the top-ten list was merely a tiny fraction of the books one would read in a lifetime. It would not seem racist or sexist or hopelessly hidebound to put Nathaniel Hawthorne on the 50 list and not

Toni Morrison . It would be more like putting oatmeal and not noodles on the breakfast menua choice partly arbitrary, partly a nod to the national past, and partly, dare one say it, a kind of reverse affirmative action: School might frankly be the place where one reads the 55 books that are a little off-putting, that have gone a little cold, that you might overlook because they do not address, in readerfriendly contemporary fashion, the issues most immediately at stake in modern life but that, with a little study, turn out to have a great deal to say. Being on the list 60 wouldnt mean so much. It might even add to a writers cachet not to be on the list, to be in one way or another too heady, too daring, too exciting to be ground up into institutional fodder for teenagers. Generations of high school kids have been turned

nR

65 off to George Eliot

Silas Marner at a tender age. One can imagine a whole new readership for her if grown-ups were left to approach Middlemarch and Daniel Deronda with open minds, at their leisure.

1 Jane Eyre, by Charlotte Bront, is a nineteenth-century


file://E:\\f4.htm

eg

is t

by being forced to read

er

ed

2006-11-12

The Official SAT Online Course


novel. Judy Blume writes contemporary young adult novels.
2 Hawthorne was a nineteenth-century American writer. Toni Morrison is a contemporary American writer.
3 George Eliot was the pseudonym of a nineteenthcentury female British novelist.
According to the author, too much energy today is spent debating
(A)

13/20

13

how to improve the education system

(B)

how to make literature seem relevant

(C) who the better writers are

(D) what students should read in school

(E)

whether or not to teach classic works

ANSWERS

AND EXPLANATIONS

Explanation for Incorrect Answer B : Choice (B) is incorrect. It is more likely that the author would argue for increased efforts to make literature seem relevant, and therefore appealing, to students.

14

nR

(A)

Explanation for Incorrect Answer C : Choice (C) is incorrect. The author refers to different writers but does not assess the relative quality of their work.

Explanation for Incorrect Answer E : Choice (E) is incorrect. The author mentions several classic novels in the passage but does not imply that the merits of these books are debated.

In the first two paragraphs of the passage (lines 1-23), the author suggests that both sides of the debate

neglect a fundamental issue

(B)

disregard a key piece of evidence

(C) ignore opposing views

(D) lack a historical perspective

(E)

dismiss a valuable tradition

ANSWERS

Explanation for Correct Answer A : Choice (A) is correct. The author asserts that the debate swirling around academic reading lists widely ignores the crucial fact that modern youth have a deteriorating relationship with books.

eg

AND EXPLANATIONS

is

Explanation for Incorrect Answer A : Choice (A) is incorrect. The text suggests that the frenzy regarding English curricula is misplaced, but it does not imply that there is an excessive push for system-wide change.

te

re

Explanation for Correct Answer D : Choice (D) is correct. The author suggests that more time should be spent reviving students' interest in reading, not debating what they should read in school.

file://E:\\f4.htm

2006-11-12

The Official SAT Online Course

14/20

Explanation for Incorrect Answer B : Choice (B) is incorrect. The first two paragraphs allude to various arguments, but they do not indicate that any of the arguments lack evidence.

Explanation for Incorrect Answer C : Choice (C) is incorrect. The author argues that the debate ignores a key issue. At no point, however, does the passage suggest that the debate's participants "ignore" opposing viewpoints.

Explanation for Incorrect Answer D : Choice (D) is incorrect. The passage contains an example drawn from United States history, but the author does not suggest that either side of the debate lacks "historical perspective."

Explanation for Incorrect Answer E : Choice (E) is incorrect. The author imagines a world in which reading is a valuable domestic tradition, passed from one generation to the next, but does not claim that both sides of the debate have dismissed any traditions.

15

The author invokes the Founding Fathers (lines 9-10) chiefly in order to
(A)

appeal to the readers sense of patriotism

(B)

introduce a historical parallel

(D) remind the reader how attitudes change over time

(E)

suggest that progress is compatible with tradition

ANSWERS

AND EXPLANATIONS

16

nR

(A)

Explanation for Incorrect Answer A : Choice (A) is incorrect. A mere mention of the "Founding Fathers" may ignite a spark of "patriotism" in some readers, but the text does not suggest that this is the author's purpose in employing the phrase.

Explanation for Incorrect Answer C : Choice (C) is incorrect. The author argues that the modern debate over reading lists is reminiscent of the early American debate regarding direct versus legislative elections. The role of legislation in the passage, however, does not extend beyond this reference.

Explanation for Incorrect Answer D : Choice (D) is incorrect. Although the passage implies that ideas often change over time, the author's reference to "the Founding Fathers" does not indicate that attitudes regarding the state of reading in the United States will change in the future.

Explanation for Incorrect Answer E : Choice (E) is incorrect. "Progress" may be compatible with certain traditions, but the author does not make such a claim at any point in the passage.

In line 18, state most nearly means

government

(B)

territory

(C) condition

(D) scale

eg

Explanation for Correct Answer B : Choice (B) is correct. Lines 910 set up a "parallel," or comparison, between the modern debate about reading lists and historical debates concerning property rights and electoral procedures. All, the author argues, fail to address crucial issues.

is t

er

ed

(C) examine the history of legislative debate

file://E:\\f4.htm

2006-11-12

The Official SAT Online Course

15/20

(E)

mood

ANSWERS

AND EXPLANATIONS

Explanation for Correct Answer C : Choice (C) is correct. "State" in this context refers to a condition or mode of being. The author suggests throughout the passage that the "state" of Americans' relationships with books is deteriorating.

Explanation for Incorrect Answer A : Choice (A) is incorrect. While the term "state" refers to a mode of government in some cases, this definition is inappropriate in the context of line 18.

Explanation for Incorrect Answer B : Choice (B) is incorrect. "State" can mean a territorial unit, but this meaning is unlikely within the context of the text.

Explanation for Incorrect Answer D : Choice (D) is incorrect. It would be illogical to suggest that the author would refer to the "scale," or unit of measurement, of reading.

(A)

rebellion of students against traditional texts

(B)

lack of enthusiasm for reading in general

(C) scarcity of books on official reading lists

(E)

negative effects of popular media

18

nR
ANSWERS

Explanation for Correct Answer B : Choice (B) is correct. "Comatose" can also mean lethargic or apathetic. In creating a parallel between the "coma" in line 23 and the "state of reading" in line 18, the author is suggesting that Americans' enthusiasm for books is "slipping deeper and deeper."

Explanation for Incorrect Answer A : Choice (A) is incorrect. Debate participants may be "hot under the collar," but the text makes no reference to upset or rebellious students.

Explanation for Incorrect Answer C : Choice (C) is incorrect. There is little evidence in the passage to support the claim that books on academic reading lists are "scarce," or in short supply.

Explanation for Incorrect Answer D : Choice (D) is incorrect. There is no mention of "archaic language" at any point in the text.

Explanation for Incorrect Answer E : Choice (E) is incorrect. Certain books reflect popular culture, but the author does not draw any comparisions between "popular media" and a comatose patient.

In lines 2427 (Let . . . pastime), the country described is noteworthy because

eg

(D) difficulty of understanding archaic language

AND EXPLANATIONS

is te

17

In line 23, the coma represents the

re

Explanation for Incorrect Answer E : Choice (E) is incorrect. "State" can mean a mental or an emotional "condition," but activities such as reading cannot be described as having either.

file://E:\\f4.htm

2006-11-12

The Official SAT Online Course

16/20

(A)

people have allowed new interests to develop from their reading

(B)

parents demonstrate their enjoyment of reading

(C) children learn to read at an early age

(D) children and parents share many activities

(E)

writing is viewed as a valuable skill

ANSWERS

AND EXPLANATIONS

Explanation for Correct Answer B : Choice (B) is correct. Lines 2427 describe an imagined world in which children routinely observe their parents reading for "pleasure."

Explanation for Incorrect Answer A : Choice (A) is incorrect. Reading may expose people to new interests or activities, but the author does not address this possibility.

Explanation for Incorrect Answer C : Choice (C) is incorrect. While children who notice their parents' enjoyment of books may be more likely to read at an early age, the author makes no such claim.

Explanation for Incorrect Answer D : Choice (D) is incorrect. The passage does not describe "many activities" shared between adults and children, only reading.

19

Lines 3039 (In school . . . cultures) present a model of education where students learn to
(A)

value cultural diversity over tradition

(B)

respect the views of both sides of the debate

nR
(E)

(C) reflect critically on the nature of American schooling

(D) differentiate between classic and contemporary works

explore the world through wide-ranging reading

ANSWERS

Explanation for Correct Answer E : Choice (E) is correct. In lines 3039, the author describes a "happy land" where students are encouraged to explore books that represent different eras, languages, and cultures.

Explanation for Incorrect Answer A : Choice (A) is incorrect. The author encourages students to read culturally diverse texts in line 39 but does not imply that diversity is more important than tradition.

Explanation for Incorrect Answer B : Choice (B) is incorrect. The selected lines construct a world in which the canon debate does not exist.

Explanation for Incorrect Answer C : Choice (C) is incorrect. The students mentioned in the quoted passage may reflect critically on literature and culture, but the passage does not describe them analyzing the nature of the American educational system.

Explanation for Incorrect Answer D :

eg is

AND EXPLANATIONS

te r

Explanation for Incorrect Answer E : Choice (E) is incorrect. Reading and writing skills are often linked, but lines 2427 address only the former.

ed

file://E:\\f4.htm

2006-11-12

The Official SAT Online Course

17/20

Choice (D) is incorrect. Although the students mentioned in the quoted passage may be able to differentiate between classic and modern works, this ability is not discussed in the passage.

20

In lines 3334, the author cites Jane Eyre and Judy Blume primarily in order to
(A)

propose that a love of reading might blur a commonly perceived distinction

(B)

show that younger readers cannot distinguish between literature of different eras

(C) argue that most modern novels have no lasting impact on readers

(D) observe that classic literature has great appeal for even reluctant readers

(E)

indicate that certain works are interchangeable

ANSWERS

AND EXPLANATIONS

Explanation for Correct Answer A : Choice (A) is correct. The third paragraph describes an idealized reality "in which reading was a popular voluntary activity." The author suggests that in this world students read books so voraciously that the distinction between books read for school and books read for pleasure becomes blurred.

21

nR

(A)

Explanation for Incorrect Answer D : Choice (D) is incorrect. Certain literary classics such as Jane Eyre may appeal to "even reluctant readers," but lines 3334 are concerned with only "voluntary, active, self-determined" readers.

Explanation for Incorrect Answer E : Choice (E) is incorrect. The author suggests that students may forget the impetus behind their reading of Jane Eyre or of Judy Blume novels but does not imply that Jane Eyre and the works of Judy Blume are interchangeable.

In lines 35-39 (In college . . . cultures), the education illustrated is best described as

elitist

(B)

philanthropic

(C) eclectic

(D) methodical

(E)

rudimentary

ANSWERS

Explanation for Correct Answer C : Choice (C) is correct. "Eclectic" means made up of elements from a variety of sources. An education supplemented by the independent discovery of books "in the library, in bookstores, on the shelves of friends" and beyond would indeed be "eclectic."

eg

AND EXPLANATIONS

is

Explanation for Incorrect Answer C : Choice (C) is incorrect. Jane Eyre's popularity has endured through time, but the author does not refer to this nineteenth-century classic in an assertion that modern novels, such as those written by Judy Blume, have only short-term appeal.

te r

Explanation for Incorrect Answer B : Choice (B) is incorrect. Although Jane Eyre and Judy Blume novels were composed centuries apart, the passage does not mention these two authors to suggest that young readers are incapable of "distinguishing between literature of different eras."

ed

file://E:\\f4.htm

2006-11-12

The Official SAT Online Course

18/20

Explanation for Incorrect Answer A : Choice (A) is incorrect. "Elitist" means snobbish, or privileged, but the education illustrated in these lines is available to the masses.

Explanation for Incorrect Answer B : Choice (B) is incorrect. It is illogical to describe the education illustrated in the passage as "philanthropic," or charitable.

Explanation for Incorrect Answer D : Choice (D) is incorrect. "Methodical" means systematic. The education described in the passage, however, is more loosely structured.

Explanation for Incorrect Answer E : Choice (E) is incorrect. The education mentioned in the quoted passage is hardly "rudimentary," or basic.

22

In lines 5460 (School . . . say), the author describes a world in which schools teach books that are
(A)

interesting

(B)

celebrated

(C) uncontroversial

(D) not obviously relevant

ANSWERS

AND EXPLANATIONS

23

nR

(A)

Explanation for Incorrect Answer A : Choice (A) is incorrect. Some readers may find the books described in lines 5460 interesting, but the author does not imply that they are particularly engaging. In fact, the author says they are off-putting and cold.

Explanation for Incorrect Answer B : Choice (B) is incorrect. The ideal literary canon described in the final paragraph may include celebrated books, but the author does not address their popularity.

Explanation for Incorrect Answer C : Choice (C) is incorrect. While the passage suggests that academic reading lists have caused a good deal of controversy, the author does not discuss whether or not the books on an ideal list would be controversial.

Explanation for Incorrect Answer E : Choice (E) is incorrect. The author does not indicate how inspirational the books on an ideal reading list would be.

Lines 60-64 (Being . . . teenagers) suggest that excluding a book from a reading list might

enhance the reputation of the books author

(B)

encourage students to protest the decision

(C) influence course curricula nationwide

(D) appease conservative parents

(E)

disappoint the books fans

ANSWERS

eg

AND EXPLANATIONS

is

te

Explanation for Correct Answer D : Choice (D) is correct. The reading list described in lines 5460 comprises books that do not address "the issues most immediately at stake in modern life" and that thus are not obviously relevant.

re

(E)

not likely to inspire

file://E:\\f4.htm

2006-11-12

The Official SAT Online Course

19/20

Explanation for Correct Answer A : Choice (A) is correct. Lines 6064 suggest that exclusion from the list would contribute to an author's reputation for being "daring" and "exciting."

Explanation for Incorrect Answer B : Choice (B) is incorrect. Lines 6064 give no indication that students would "protest" the exclusion of any book.

Explanation for Incorrect Answer C : Choice (C) is incorrect. Though the exclusion of a book from the author's ideal canon could affect what students read in school, lines 6064 are more concerned with the influence that such an exclusion would have on a writer's reputation.

Explanation for Incorrect Answer D : Choice (D) is incorrect. While "conservative parents" may support the exclusion of "heady" or "daring" authors from academic reading lists, lines 6064 make no reference to parents at all.

Explanation for Incorrect Answer E : Choice (E) is incorrect. Lines 6064 say nothing about the "fans" of any particular book.

24

The main purpose of the passage is to


(A)

shift the focus of a debate

(C) suggest a practical solution

(E)

promote certain kinds of writing

ANSWERS

nR

Explanation for Correct Answer A : Choice (A) is correct. The main goal of the passage is to shift the reading debate's focus from the content of academic reading lists to ways that Americans' ailing relationships with books might be improved.

Explanation for Incorrect Answer B : Choice (B) is incorrect. Instead of promoting "one side" of an existing debate, the author suggests that both sides are ignoring "the most interesting and important feature of the whole discussion."

Explanation for Incorrect Answer C : Choice (C) is incorrect. The passage provides less of a practical solution to a problem than a new perspective from which to view the problem.

Explanation for Incorrect Answer D : Choice (D) is incorrect. The author makes no attempt at reviving "a discredited idea" at any point in the passage.

Explanation for Incorrect Answer E : Choice (E) is incorrect. The author's focus is on reading rather than on writing.

eg

AND EXPLANATIONS

is t

(D) revive a discredited idea

er

(B)

support one side in a debate

ed

Privacy Policy

Back to Score Report

Copyright 2006 The College Board. All rights reserved.

Terms of Use

Contact Us

file://E:\\f4.htm

2006-11-12

The Official SAT Online Course

20/20

nR

eg

is

te

re

file://E:\\f4.htm

2006-11-12

The Official SAT Online Course

1/21

Help | Profile | My Organizer | My Bookmarks | Logout

Answers and Explanations

Test Sections

Back to Score Report

Section 1

View Answers and Explanations


Online - Practice Test #6

Section 2

Section 4

Section 5

Section 6

Inside famed actor Lily Langtrys private railroad car were a drawing room with a piano, bath fixtures of silver, and there were draperies trimmed with Brussels lace.
(A)

Section 7

there were draperies trimmed with Brussels lace

Section 8

(B)

draperies trimmed with Brussels lace

Section 9

(C) trimmed with Brussels lace were draperies

Section 10

(D) the draperies were trimmed with Brussels lace

(E)

draperies trimmed with Brussels lace were there

ANSWERS

AND EXPLANATIONS

Explanation for Correct Answer B : Choice (B) is correct. It avoids the error of the original by making the third item in the series a noun phrase, as the other two items are.

nR

(A)

Explanation for Incorrect Answer C : Choice (C) involves an error in parallelism. The third item in the series should be a noun phrase like the other two ("a drawing room with a piano," "bath fixtures of silver"), not an independent clause.

Explanation for Incorrect Answer D : Choice (D) involves an error in parallelism. The third item in the series should be a noun phrase like the other two ("a drawing room with a piano," "bath fixtures of silver"), not an independent clause.

Explanation for Incorrect Answer E : Choice (E) involves an error in parallelism. The third item in the series should be a noun phrase like the other two ("a drawing room with a piano," "bath fixtures of silver"), not an independent clause.

Samuel Adams was by no means the first American to espouse the democratic cause, but he has been the first who conceived the party machinery that made it practical.

has been the first who conceived

(B)

had been the first who conceived

(C) was the first having conceived

(D) was the first to conceive

(E)

having been the first to conceive

ANSWERS

Explanation for Correct Answer D : Choice (D) is correct. It avoids the error of the original by using the past tense consistently.

eg

Explanation for Incorrect Answer A : Choice (A) involves an error in parallelism. The third item in the series should be a noun phrase like the other two ("a drawing room with a piano," "bath fixtures of silver"), not an independent clause.

AND EXPLANATIONS

is

te re d

file://E:\\f5.htm

2006-11-12

The Official SAT Online Course

2/21

Explanation for Incorrect Answer A : Choice (A) involves an error in verb tense. It shifts from past tense ("was") to present perfect tense ("has been") for no reason.

Explanation for Incorrect Answer B : Choice (B) involves an error in verb tense. It shifts from past tense ("was") to present perfect tense ("had been") for no reason.

Explanation for Incorrect Answer C : Choice (C) involves an error in verb form. The participle "Having conceived" should be the infinitive form "to conceive."

Explanation for Incorrect Answer E : Choice (E) creates a fragment. There is no subject for the verb "having been."

The plans were made too hastily, without enough thought behind it.
(A)

too hastily, without enough thought behind it

(B)

too hasty, without enough thought behind it

(C) too hastily, without enough thought behind them

Explanation for Correct Answer C : Choice (C) is correct. It avoids the error of the original by using a plural pronoun ("them") to refer to the plural noun "plans."

nR eg

(A)

Explanation for Incorrect Answer A : Choice (A) involves an error in agreement. The singular pronoun "it" cannot correctly refer to the plural noun "plans."

Explanation for Incorrect Answer B : Choice (B) involves improper modification. To modify the verb "made," "hasty" should be "hastily."

Explanation for Incorrect Answer D : Choice (D) involves improper modification. To modify the verb "made," "hasty" should be "hastily."

Explanation for Incorrect Answer E : Choice (E) involves an error in agreement. The singular pronoun "it" cannot correctly refer to the plural noun "plans."

Many psychologists do not use hypnosis in their practices, it is because they know very little about it and are wary of it as a result.

practices, it is because they know very little about it and are wary of it as a result.

(B)

practices because they know very little about it and are therefore wary of it.

(C) practices for the reason that they know very little about it, with resulting wariness

(D) practices because of knowing very little about it and therefore they are wary of it

(E)

practices, their knowledge of it being very little results in wariness of it

is te

ANSWERS

AND EXPLANATIONS

re

(E)

too hastily, and there is not enough thought behind it

(D) too hasty, and there is not enough thought behind them

file://E:\\f5.htm

2006-11-12

The Official SAT Online Course

3/21

ANSWERS

AND EXPLANATIONS

Explanation for Correct Answer B : Choice (B) is correct. It avoids the error of the original by using a second verb phrase ("are therefore wary of it as a result") instead of a second independent clause.

Explanation for Incorrect Answer A : Choice (A) involves improper coordination. Two complete thoughts ("Many...practices" and "it...result") are joined by only a comma.

Explanation for Incorrect Answer C : Choice (C) involves improper phrasing. Whose "wariness" is being referred to is not clear.

Explanation for Incorrect Answer D : Choice (D) involves improper coordination. Two complete thoughts ("Many...it" and" therefore...result") are joined only by a conjuction ("and"), without a comma to proceed it.

Explanation for Incorrect Answer E : Choice (E) involves improper pronoun case. "It" should be "its."

No two of the specimens was sufficiently alike to warrant them being called members of a single species.
(A)

was sufficiently alike to warrant them being called

(B)

was sufficiently alike to warrant the calling of them

(C) was sufficiently alike to warrant their being called

(D) were sufficiently alike to warrant the calling of them

(E)

were sufficiently alike to warrant calling them

ANSWERS

nR

(A)

Explanation for Correct Answer E : Choice (E) is correct. It avoids the error of the original by using a plural verb ("were") for the subject "two."

Explanation for Incorrect Answer A : Choice (A) involves an error in agreement. The subject "two" requires a plural verb ("were").

Explanation for Incorrect Answer B : Choice (B) involves an error in agreement. The subject "two" requires a plural verb ("were").

Explanation for Incorrect Answer C : Choice (C) involves an error in agreement. The subject "two" requires a plural verb ("were").

Explanation for Incorrect Answer D : Choice (D) involves wordiness. The wordy phrase "the calling of them" should simply be "calling them."

My grandson thinks he can cook better than any other person at the fair; and he has the blue ribbons to prove it.

My grandson thinks he can cook better than any other person at the fair; and he

eg i

AND EXPLANATIONS

st

er e

d
file://E:\\f5.htm

2006-11-12

The Official SAT Online Course

4/21

(B)

My grandson thinks he can cook better than any person at the fair, and he

(C) My grandson thinks he can cook better than any person at the fair, consequently he

(D) To think he can cook better than any other person at the fair, my grandson

(E)

Thinking he can cook better than any other person at the fair, my grandson

ANSWERS

AND EXPLANATIONS

Explanation for Correct Answer B : Choice (B) is correct. It avoids the error of the original by using a comma to link two independent clauses joined by the conjunction "and."

Explanation for Incorrect Answer A : Choice (A) involves improper coordination. It uses a semicolon where a comma is necessary.

Explanation for Incorrect Answer C : Choice (C) creates an illogical sentence. It illogically suggests that the grandson's having blue ribbons is a result of his thinking that he is the best cook at the fair.

Differing only slightly from the Greeks were the Roman theaters, which were often freestanding rather than part of a hillside.

nR
(B)

(A)

Differing only slightly from the Greeks were the Roman theaters, which

(C) Differing only in the slightest from the Greeks were the Roman theaters, which

(D) The Greeks differed only slightly from the Romans, they

Differing only slightly from Greek theaters, Roman theaters

(E)

The Greek theaters differed from the Roman theaters only slightly, where they

ANSWERS

Explanation for Correct Answer B : Choice (B) is correct. It avoids the error of the original by correctly comparing "Greek theaters" to "Roman theaters."

Explanation for Incorrect Answer A : Choice (A) involves an illogical comparison. It compares "the Greeks" with "the Roman theaters."

Explanation for Incorrect Answer C : Choice (C) involves an illogical comparison. It compares "the Greeks" with "the Roman theaters."

Explanation for Incorrect Answer D : Choice (D) involves improper coordination. Two complete thoughts ("The Greeks...Romans" and "they...hillside") are joined by only a comma.

Explanation for Incorrect Answer E :

eg

AND EXPLANATIONS

is te

Explanation for Incorrect Answer E : Choice (E) creates an illogical sentence. It does not make sense to say that the blue ribbons that the grandson had somehow prove that he was "thinking he can cook better . . . fair."

re

Explanation for Incorrect Answer D : Choice (D) results in an illogical sentence. There is no relationship between the idea in the first part of the sentence ("To think . . . fair") and the idea in the last part of the sentence ("my grandson . . . to prove it.").

file://E:\\f5.htm

2006-11-12

The Official SAT Online Course

5/21

Choice (E) involves an error in pronoun reference. There is no place to which "where" can refer.

When chronological order is followed too mechanically, they are obscuring rather than clarifying important relationships.
(A)

When chronological order is followed too mechanically, they are obscuring rather than clarifying important relationships.

(B)

When chronological order is followed too mechanically, it obscures rather than clarifying important relationships.

(C) Chronological order, if too mechanically followed, obscures rather than it clarifies important relationships.

(D) Chronological order, if followed too mechanically, obscures rather than clarifies important relationships.

(E)

If you follow a too mechanical chronological order, it obscures rather than clarifies important relationships.

ANSWERS

AND EXPLANATIONS

Explanation for Correct Answer D : Choice (D) is correct. It avoids the error of the original by omitting the plural pronoun ("they"), which did not agree with the singular noun "chronological order."

Explanation for Incorrect Answer B : Choice (B) involves an error in verb form. The verb "clarifying" should be "clarifies."

Explanation for Incorrect Answer C : Choice (C) involves a pronoun error. The pronoun "it" is unnecessary.

Explanation for Incorrect Answer E : Choice (E) involves a pronoun error. It is not clear what the pronoun "it" is meant to refer to.

nR
(A)

Small marine crustaceans known as krill are often fed to farm animals, but there is not much human consumption.

animals, but there is not much human consumption

(B)

animals, but consumption is not done much by people

(C) animals but are rarely eaten by people

(D) animals, but eating them is rarely done by humans

(E)

animals, but among people there is not much consumption

ANSWERS

Explanation for Correct Answer C : Choice (C) is correct. It avoids the error of the original by avoiding unclear pronoun usage.

Explanation for Incorrect Answer A : Choice (A) involves an error in pronoun reference. "There" does not refer to anything in the sentence.

Explanation for Incorrect Answer B : Choice (B) involves wordiness. The wordy phrase "consumption is not done much by people" should be simply "are rarely consumed by people" or "are rarely eaten by people."

eg

AND EXPLANATIONS

is

te

re

Explanation for Incorrect Answer A : Choice (A) involves an error in agreement. The plural pronoun "they" is used incorrectly to refer to the singular noun "chronological order."

file://E:\\f5.htm

2006-11-12

The Official SAT Online Course

6/21

Explanation for Incorrect Answer D : Choice (D) involves an error in pronoun reference. The pronoun "them" incorrectly refers to the closest previous plural noun, "farm animals."

Explanation for Incorrect Answer E : Choice (E) involves an unclear reference. The object of "consumption" is not clear.

10

The educators remarks stressed that well-funded literacy programs are needed if everyone is to gain the skills required for survival in society.
(A)

that well-funded literacy programs are needed if everyone is to gain

(B)

that well-funded literacy programs needed in gaining

(C) there is a need of well-funded literacy programs for everyone will gain

(D) a need for well-funded literacy programs and everyone will gain

(E)

why well-funded literacy programs being necessary for everyone in gaining

ANSWERS

AND EXPLANATIONS

Explanation for Correct Answer A : Choice (A) is correct. It correctly uses a relative clause (introduced by "that") to indicate what the educator was stressing.

11

nR

(A)

Explanation for Incorrect Answer D : Choice (D) involves improper coordination. Two complete thoughts ("The educator's...programs" and "everyone...society") are joined by only a conjunction ("and"), without the comma that should precede it.

Explanation for Incorrect Answer E : Choice (E) creates an illogical sentence. A noun can be "stressed," but an adverb ("why") cannot.

The Portuguese musical tradition known as fado, or fate, has been called the Portuguese blues because of their songs that bemoan someones misfortune, especially the loss of romantic love.

of their songs that bemoan someones

(B)

of their songs bemoaning their

(C) its songs bemoan

(D) the songs that bemoaned

(E)

of how it bemoans their

ANSWERS

Explanation for Correct Answer C : Choice (C) is correct. It avoids the error of the original by using the pronoun "its," which correctly refers to the noun "The Portuguese...fado."

eg

Explanation for Incorrect Answer C : Choice (C) involves improper coordination. Two complete thoughts ("The educator's...programs" and "everyone...society") are joined by only a conjunction ("for"), without the comma that should precede it.

AND EXPLANATIONS

is

te

Explanation for Incorrect Answer B : Choice (B) involves improper verb form. The verb "needed" lacks an auxillary verb ("are").

re d

file://E:\\f5.htm

2006-11-12

The Official SAT Online Course

7/21

Explanation for Incorrect Answer A : Choice (A) involves an error in pronoun reference. The pronoun "their" does not refer to anything that comes before it.

Explanation for Incorrect Answer B : Choice (B) involves an error in pronoun reference. The pronoun "their" does not refer to anything that comes before it.

Explanation for Incorrect Answer D : Choice (D) creates a fragment. The subordinate clause "that...love" is not completed.

Explanation for Incorrect Answer E : Choice (E) involves an error in pronoun reference. The pronoun "their" does not refer to anything that comes before it.

12

Every year, toy manufacturers gather groups of children into playrooms, observing

their choices of toys as predicting which new products will become the

most popular. No error

Corrected Sentence: Every year, toy manufacturers gather groups of children into playrooms, oberving their choices of toys and predicting which new products will become the most popular.

13

nR

Explanation for Incorrect Answer A : There is no error at (A). The verb "gather" appropriately agrees with its subject, "toy manufacturers."

Explanation for Incorrect Answer C : There is no error at (C). The relative pronoun "which" appropriately refers to the noun "new products" in describing the kinds of products that "will become most popular."

Explanation for Incorrect Answer D : There is no error at (D). The adverb "most" properly modifies the adjective "popular."

Explanation for Incorrect Answer E : There is an error in the sentence.

During the last fifty years, we come to take radio communication for granted, but the

mere suggestion that we could communicate in such a fashion must once have

seemed outlandish. No error

eg

Explanation for Correct Answer B : The error in this sentence occurs at (B), where there is an incomplete verb form. The word "as" cannot properly join the verbs "observing" and "predicting" to form the compound predicate. Instead, the conjunction "and" should be used.

is

te r

ANSWERS

AND EXPLANATIONS

ed

file://E:\\f5.htm

2006-11-12

The Official SAT Online Course

8/21

ANSWERS

AND EXPLANATIONS

Corrected Sentence: During the last fifty years, we have come to take radio communication for granted, but the mere suggestion that we could communicate in such a fashion must once have seemed outlandish.

Explanation for Correct Answer B : The error in this sentence occurs at (B), where there is an improper verb tense. The introductory prepositional phrase "During the last fifty years" establishes that the action of the main clause (taking radio communications for granted) takes place over the whole period. Therefore, the present-tense verb, "come" should be in the present perfect tense ("have come").

Explanation for Incorrect Answer A : There is no error at (A). The preposition "During" is an appropriate and necessary part of the prepositional phrase "During the last fifty years."

Explanation for Incorrect Answer C : There is no error at (C). The infinitive "to take" appropriately introduces its object, "radio communications."

Explanation for Incorrect Answer E : There is an error in the sentence.

14

nR eg
ANSWERS

emergency vehicles in traffic jams carry a stern warning to motorists. No error

Corrected Sentence: The uncompromising tone of a recent city call ordinance concerning the blocking of emergency vehicles in traffic jams carries a stern warning to motorists.

AND EXPLANATIONS

Explanation for Correct Answer C : The error in this sentence occurs at (C), where there is subject-verb disagreement. The plural verb "carry" cannot refer to the singular subject, "The uncompromising tone." The singular verb "carries" is needed.

Explanation for Incorrect Answer A : There is no error at (A). The adjective "uncompromising" appropriately modifies the noun "tone."

Explanation for Incorrect Answer B : There is no error at (B). The participle "concerning" appropriately modifies the preceding noun "ordinance."

Explanation for Incorrect Answer D : There is no error at (D). The preposition "to" properly links the noun phrase "a stern warning" to the noun "motorists."

Explanation for Incorrect Answer E : There is an error in the sentence.

is

The uncompromising tone of a recent city hall ordinance concerning the blocking of

te

re

Explanation for Incorrect Answer D : There is no error at (D). The preposition "in" appropriately introduces the prepositional phrase "in such a fashion," and the adjective "such" appropriately modifies "a fashion."

file://E:\\f5.htm

2006-11-12

The Official SAT Online Course

9/21

15

Formed by volcanic eruptions

over

the last five million years, the Hawaiian Islands

containing

an

incredibly wide

variety of speciesmany found

nowhere else on Earth.

No error

ANSWERS

AND EXPLANATIONS

Corrected Sentence: Formed by volcanic eruptions over the last five million years, the Hawaiian Islands contain an incredibly wide variety of species--many found nowhere else on Earth.

Explanation for Correct Answer B : The error in this sentence occurs at (B), where a sentence fragment is created. The use of the participle "containing," instead of the present-tense "contain," leaves the sentence without a main verb.

Explanation for Incorrect Answer A : There is no error at (A). The preposition "over" properly introduces the prepositional phrase "over the last five million years," which establishes when the volcanic eruptions took place.

Explanation for Incorrect Answer C : There is no error at (C). The adverb "incredibly" appropriately modifies the adjective "wide."

Explanation for Incorrect Answer E : There is an error in the sentence.

16

nR

ANSWERS

Because the owl is usually nocturnal plus being virtually noiseless in flight,

it is seldom seen by the casual observer. No error

Corrected Sentence: Because the owl is usually nocturnal and is virtually noiseless in flight, it is seldom seen by the casual observer.

Explanation for Correct Answer B : The error in this sentence occurs at (B), where there is an awkward construction. The phrase "and is" should be used in place of "plus being" to properly join one characteristic (nocturnal behavior) with the other (noiselessness in flight).

Explanation for Incorrect Answer A : There is no error at (A). The singular verb, "is," agrees with its singular subject, "the owl," and the adverb "usually" properly modifies the adjective "nocturnal."

Explanation for Incorrect Answer C : There is no error at (C). The pronoun "it" refers correctly to the noun "the owl."

eg i

Explanation for Incorrect Answer D : There is no error at (D). The phrase "nowhere else" properly indicates that the "wide variety of species" are found mainly in one place.

AND EXPLANATIONS

st er

ed

file://E:\\f5.htm

2006-11-12

The Official SAT Online Course

10/21

Explanation for Incorrect Answer D : There is no error at (D). The singular verb "is" agrees with its singular subject ("the owl"), and the adverb "seldom" appropriately modifies the verb "seen."

Explanation for Incorrect Answer E : There is an error in the sentence.

17

An

economical and efficient

recycling center

is accessible

to the public, responsive to

community needs, and

comply with

current federal regulations

governing waste

disposal.

No error

ANSWERS

AND EXPLANATIONS

Corrected Sentence: An economical and efficient recycling center is accessible to the public, responsive to community needs, and complies with current federal regulations governing waste disposal.

Explanation for Correct Answer C : The error in this sentence occurs at (C), where there is subject-verb disagreement. The plural verb "comply" does not agree with its singular subject ("recycling center").

Explanation for Incorrect Answer A : There is no error at (A). The compound adjective, "economical and efficient," appropriately modifies "recycling center."

18

nR

ANSWERS

Explanation for Incorrect Answer B : There is no error at (B). The singular verb "is" agrees with its singular subject, "recycling center," and the adjective "accessible" appropriately modifies "recycling center" to indicate one aspect (accessibility) of an "economical and efficient" recycling center.

Explanation for Incorrect Answer D : There is no error at (D). The participle "governing" appropriately indicates the kind of regulations being discussed (those governing waste disposal).

Explanation for Incorrect Answer E : There is an error in the sentence.

Jean Toomer was not only the author of Cane, a novel whose publication

has been viewed as marking the beginning of the Harlem Renaissance, but also a

respected advisor among Quakers. No error

Corrected Sentence: The sentence contains no error.

Explanation for Correct Answer E : There is no error in this sentence.

eg

AND EXPLANATIONS

is t

er ed

file://E:\\f5.htm

2006-11-12

The Official SAT Online Course

11/21

Explanation for Incorrect Answer A : There is no error at (A). The phrase "not only" operates appropriately as the first part of the correlative construction "not only . . . but also."

Explanation for Incorrect Answer B : There is no error at (B). The tense of the verb correctly indicates that the action described (the way the novel is viewed) is still going on.

Explanation for Incorrect Answer C : There is no error at (C). The adverb "as" joins with the participle "marking" to produce an appropriate idiom.

Explanation for Incorrect Answer D : There is no error at (D). The preposition "among" joins with the plural noun "Quakers" to correctly describe where Toomer was "a respected advisor."

19

, but the results from New Election results came in from upstate New York quite rapid

. No error York City were known even faster

ANSWERS

AND EXPLANATIONS

Explanation for Correct Answer B : The error in this sentence occurs at (B), where an adjective ("rapid") is incorrectly used to modify the verb "came." The adverb "rapidly" is needed.

20

nR

ANSWERS

Explanation for Incorrect Answer A : There is no error at (A). The preposition "in" appropriately modifies the verb "came" to indicate where the election results came from.

Explanation for Incorrect Answer C : There is no error at (C). The past tense of the verb is consistent with the tense previously established by the past-tense verb "came."

Explanation for Incorrect Answer D : There is no error at (D). The adverb "even" appropriately modifies the adjective "faster" to indicate how quickly the results were known.

Explanation for Incorrect Answer E : There is an error in the sentence.

As we rely more and more on the Internet, your need for effective security planning

and design to safeguard data has increased. No error

Corrected Sentence: As we rely more and more on the Internet, the need for effective security planning and design to safeguard data has increased.

Explanation for Correct Answer B :

eg

AND EXPLANATIONS

is te

Corrected Sentence: Election results came in from upstate New York quite rapidly, but the results from New York City were known even faster.

re

d
file://E:\\f5.htm

2006-11-12

The Official SAT Online Course

12/21

The error in this sentence occurs at (B), where there is an error in pronoun use. There is nothing in the sentence to which the possessive pronoun "your" can logically refer.

Explanation for Incorrect Answer A : There is no error at (A). The subordinating conjuntion "As" appropriately introduces the dependent clause ("As we rely . . . on the Internet").

Explanation for Incorrect Answer C : There is no error at (C). The infinitive "to safeguard" appropriately indicates the purpose of "effective . . . design" (to safeguard data).

Explanation for Incorrect Answer D : There is no error at (D). The tense of the verb "has increased" appropriately indicates that the action described (the increasing need for effective planning) is ongoing.

Explanation for Incorrect Answer E : There is an error in the sentence.

21

The book is essentially a detailed and very well documented record of what happened

to each of the protestors. No error

Corrected Sentence:

22

nR

Explanation for Incorrect Answer A : There is no error at (A). The singular verb "is" agrees with its singular subject "the book," and the adverb "essentially" appropriately modifies the verb "is."

Explanation for Incorrect Answer B : There is no error at (B). The adverbs "very" and "well" combine to appropriately modify the adjective "documented."

Explanation for Incorrect Answer C : There is no error at (C). The preposition "of" combines with the relative pronoun "what" to produce an appropriate idiom indicating what was documented in the book.

Explanation for Incorrect Answer D : There is no error at (D). The pronoun "each" combines with the preposition "of" to form an appropriate idiom linking "what happened" to the group to which it happened.

Experts agree that permanently modifying eating and exercise habits rather than

merely dieting for brief periods are the key to controlling weight. No error

eg

Explanation for Correct Answer E : There is no error in this sentence.

is t

ANSWERS

AND EXPLANATIONS

er

ed

file://E:\\f5.htm

2006-11-12

The Official SAT Online Course

13/21

ANSWERS

AND EXPLANATIONS

Corrected Sentence: Experts agree that permanently modifying eating and exercise habits rather than merely dieting for brief periods is the key to controlling weight.

Explanation for Correct Answer C : The error in this sentence occurs at (C), where there is an incorrect verb form. The plural verb "are" should be the singular "is" to agree with the singular gerund, "modifying eating and exercise habits."

Explanation for Incorrect Answer A : There is no error at (A). The plural verb ("agree") agrees with its subject, "Experts," and the relative pronoun "that" appropriately introduces the phrase that immediately follows.

Explanation for Incorrect Answer B : There is no error at (B). The gerund "dieting" is appropriately modified by the adverb "merely."

Explanation for Incorrect Answer D : There is no error at (D). The preposition "to" properly links the noun "the key" with the gerund "controlling weight."

Explanation for Incorrect Answer E : There is an error in the sentence.

23

, that The ability to control the plots of our dreams is a skill, researchers have shown

we can learn if you want to change recurrent dreams. No error

24

nR

ANSWERS

Corrected Sentence: The ability to control the plots of our dreams is a skill, researchers have shown, that we can learn if we want to change recurrent dreams.

Explanation for Correct Answer D : The error in this sentence occurs at (D), where there is improper prounoun use. The second-person pronoun "you" should be changed to the first-person plural "we" to be consistent with the earlier use of "we."

Explanation for Incorrect Answer A : There is no error at (A). The infinitive "to control" properly modifies the preceding noun, "The ability."

Explanation for Incorrect Answer B : There is no error at (B). The singular verb "is" agrees with its singular subject, "The ability."

Explanation for Incorrect Answer C : There is no error at (C). The verb tense appropriately establishes that the action described (what researchers have determined about the ability to control dreams) has already taken place.

Explanation for Incorrect Answer E : There is an error in the sentence.

eg

AND EXPLANATIONS

is t

er e

file://E:\\f5.htm

2006-11-12

The Official SAT Online Course

14/21

In

swimming

as to

soccer, Evangelina proved time after time to be

an abler

competitor

than

Juanita.

No error

ANSWERS

AND EXPLANATIONS

Corrected Sentence: In swimming as in soccer, Evangelina proved time after time to be an abler competitor than Juanita.

Explanation for Correct Answer B : The error in this sentence occurs at (B), where there is an improper idiom. The preposition "in" should be used with the noun "soccer" to convey the idea that what happens "in swimming" is also what happens "in soccer."

Explanation for Incorrect Answer A : There is no error at (A). The preposition "in" combines with the gerund "swimming" to create an appropriate idiom.

25

The common cold is one of our most indiscriminate diseases; it makes no

nR

ANSWERS

distinction between you and me, millionaires and paupers, or athletes and couch

potatoes. No error

Corrected Sentence:

Explanation for Correct Answer E : There is no error in this sentence.

Explanation for Incorrect Answer A : There is no error at (A). The pronoun "one" is used correctly to refer to "The common cold," and the preposition "of" appropriately introduces the prepositional phrase that modifies "The common cold."

Explanation for Incorrect Answer B : There is no error at (B). The pronoun "it" correctly refers to the noun "The common cold," and the singular verb "makes" agrees with its singular subject, "it."

Explanation for Incorrect Answer C : There is no error at (C). The noun "distinction" links appropriately with the preposition "between" to create an appropriate idiom.

eg

AND EXPLANATIONS

is t

Explanation for Incorrect Answer E : There is an error in the sentence.

er

Explanation for Incorrect Answer D : There is no error at (D). The conjunction "than" is used correctly to introduce the object of the comparison ("Juanita").

ed

Explanation for Incorrect Answer C : There is no error at (C). The comparative form of the adjective "able" is correct and properly modifies the noun "competitor."

file://E:\\f5.htm

2006-11-12

The Official SAT Online Course

15/21

Explanation for Incorrect Answer D : There is no error at (D). Both pronouns ("you" and "me") are properly in the objective case.

26

Like his other

cookbooks, in his new book Chef Louis offers lengthy explanations

of what he considers to be

basic cooking principles.

No error

ANSWERS

AND EXPLANATIONS

Corrected Sentence: As he does in his other cookbooks, in his new book Chef Louis offers lengthy explanations of what he considers to be basic cooking principles.

Explanation for Correct Answer A : The error in this sentence occurs at (A), where there is an illogical comparison. It would be appropriate to compare what is in "his other cookbooks" with what is in "his new book," but instead the sentence illogically compares a thing ("his other cookbooks") with a person ("Chef Louis").

Explanation for Incorrect Answer B : There is no error at (B). The preposition "of" combines with the pronoun "what" to create an appropriate idiom.

Explanation for Incorrect Answer C : There is no error at (C). The pronoun "he" correctly refers to its antecedent "Chef Louis," and the singular verb "considers" agrees with its singular subject, "he."

Explanation for Incorrect Answer D : There is no error at (D). The infinitive "to be" joins with the verb "considers" to appropriately indicate what the "lengthy explanations" are about.

27

nR

ANSWERS

Explanation for Incorrect Answer E : There is an error in the sentence.

Paul Ecke, flower grower and hybridizer, became known as Mr. Poinsettia after

developing new varieties of the flower and by pioneering it as a living symbol of

Christmas. No error

Corrected Sentence: Paul Ecke, flower grower and hybridizer, became known as "Mr. Poinsettia" after developing new varieties of the flower and pioneering it as a living symbol of Christmas.

Explanation for Correct Answer C : The error in this sentence occurs at (C), where there are excess words. The preposition "by" is unnecessary.

Explanation for Incorrect Answer A :

eg i

AND EXPLANATIONS

st

er e

d
file://E:\\f5.htm

2006-11-12

The Official SAT Online Course

16/21

There is no error at (A). The past-tense verb "became" appropriately establishes that Mr. Ecke was known a "Mr. Poinsettia" only after he developed "new varieties of the flower."

Explanation for Incorrect Answer B : There is no error at (B). The adjective "known" appropriately modifies the subject of the sentence "Paul Ecke."

Explanation for Incorrect Answer D : There is no error at (D). The prepositon "as" joins with the noun phrase "a living symbol" to create an appropriate idiom.

Explanation for Incorrect Answer E : There is an error in the sentence.

28

Long

thought of as a quiet, stuffy place where peoplejust borrowed books, libraries

have been changing their images dramatically over the last few years.

No error

ANSWERS

AND EXPLANATIONS

Corrected Sentence: Long thought of as quiet, stuffy places where people just borrowed books, libraries have been changing their images dramatically over the last few years.

29

nR

ANSWERS

Explanation for Incorrect Answer A : There is no error at (A). The verb "thought" combines with the preposition "of" to create an appropriate idiom.

Explanation for Incorrect Answer C : There is no error at (C). The plural noun "people" is logical here because more than one library is being talked about.

Explanation for Incorrect Answer D : There is no error at (D). The present perfect progressive tense of the verb appropriately indicates that the action described may be ongoing.

Explanation for Incorrect Answer E : There is an error in the sentence.

To understand twentieth-century economic practices, we must be sufficiently

familiar with Keynesian theories, whether one agrees with them or not. No error

Corrected Sentence: To understand twentieth-century economic practices, one must be sufficiently familiar with Keynesian theories, whether one agrees with them or not.

Explanation for Correct Answer B : The error in this sentence occurs at (B), where there is incorrect pronoun use. The

eg

AND EXPLANATIONS

is t

Explanation for Correct Answer B : The error in this sentence occurs at (B), where there is noun-noun disagreement. To agree with the plural "libraries," the singular noun "place" should be changed to the plural "places."

er

ed

file://E:\\f5.htm

2006-11-12

The Official SAT Online Course

17/21

first-person plural pronoun "we" is not consistent with the later pronoun "one."

Explanation for Incorrect Answer A : There is no error at (A). The infinitive "To understand" appropriately indicates that a familiarity with Keynesian theories will facilitate one's understanding of "twentiethcentury economic practices."

Explanation for Incorrect Answer C : There is no error at (C). The adjective "familiar" joins with the preposition "with" to form an appropriate idiom.

Explanation for Incorrect Answer D : There is no error at (D). In the prepositional phrase "with them," the object of the preposition, "them," must be in the objective case, as it is here.

Explanation for Incorrect Answer E : There is an error in the sentence.

(11) As ambassador from the Powhatans to the Jamestown settlers, Pocahontas headed off confrontations between mutually suspicious parties. (12) Later, after her marriage to colonist John Rolfe, Pocahontas traveled to England, where her diplomacy played a large part in gaining support for the Virginia Company.

30

nR eg

(E)

(5) But can Smiths account be trusted? (6) Probably it cannot, say several historians interested in dispelling myths about Pocahontas. (7) According to these experts, in his eagerness to find patrons for future expeditions, Smith changed the facts in order to enhance his image. (8) Portraying himself as the object of a royal princess devotion may have merely been a good public relations ploy. (9) Research into Powhatan culture suggests that what Smith described as an execution might have been merely a ritual display of strength. (10) Smith may have been a character in a drama in which even Pocahontas was playing a role.

What is the best way to deal with sentence 1 (reproduced below) ?

Not many children leave elementary school and they have not heard of Pocahontas heroic rescue of John Smith from her own people, the Powhatans.

(A)

Leave it as it is

(B)

Switch its position with that of sentence 2.

(C) Change leave to have left.

(D) Change and they have not heard to without having heard.

Remove the comma and insert known as the.

ANSWERS

AND EXPLANATIONS

Explanation for Correct Answer D : Choice (D) is correct. It properly explains that most children hear the Pocahontas story before they leave elementary school.

Explanation for Incorrect Answer A : Choice (A) is unsatisfactory because the original sentence connects the two main ideas--children leaving school and the Pocahontas story--with only the conjunction "and." The sentence thus offers no clue about the relationship between the two

is t

er

ed

(1) Not many children leave elementary school and they have not heard of Pocahontas heroic rescue of John Smith from her own people, the Powhatans. (2) Generations of Americans have learned the story of a courageous Indian princess who threw herself between the Virginia colonist and the clubs raised to end his life. (3) The captive himself reported the incident. (4) According to that report, Pocahontas held his head in her arms and laid her own upon his to save him from death.

file://E:\\f5.htm

2006-11-12

The Official SAT Online Course

18/21

ideas.

Explanation for Incorrect Answer B : Choice (B) is unsatisfactory because it is logical to give the names of the principal figures in a story or event before telling the story, not after--especially when the names are familiar.

Explanation for Incorrect Answer C : Choice (C) is unsatisfactory because it repeats the error of the original in failing to explain the relationship between the children and the story.

Explanation for Incorrect Answer E : Choice (E) is unsatisfactory because the original correctly refers to Pocahontas's tribe.

31

In context, which of the following is the best way to revise the underlined wording in order to combine sentences 3 and 4 (reproduced below)?

The captive himself reported the incident. According to that report Pocahontas held his head in her arms and laid her own upon his to save him from death.
(A)

The captive himself reported the incident, according to which

(B)

Since then, the captive reported the incident, which said that

(D) It seems that in the captives report of the incident he says that

(E)

According to the captives own report of the incident,

ANSWERS

AND EXPLANATIONS

Explanation for Correct Answer E : Choice (E) is correct. The resulting sentence maintains the sense of the original while eliminating the redundancy.

32

nR

(A)

Explanation for Incorrect Answer A : Choice (A) is unsatisfactory because it contains an unclear referent: "which" seems to refer to the incident itself rather than to the report.

Explanation for Incorrect Answer B : Choice (B) is unsatisfactory because the word "which" appears to refer to the incident, when it can logically refer only to the report of the incident.

Explanation for Incorrect Answer C : Choice (C) is unsatisfactory because the word "consequently" suggests incorrectly that Smith's report is a consequence of the legend.

Explanation for Incorrect Answer D : Choice (D) is unsatisfactory because it uses the unnecessary phrase "it seems" to relate a fact.

Which of the following phrases is the best to insert at the beginning of sentence 10 to link it to sentence 9?

Far from being in mortal danger,

(B)

If what he says is credible,

(C) What grade school history never told you is this:

(D) They were just performing a ritual, and

(E)

But quite to the contrary,

eg

is t

er ed

(C) Consequently, the captive himself reports that

file://E:\\f5.htm

2006-11-12

The Official SAT Online Course

19/21

ANSWERS

AND EXPLANATIONS

Explanation for Correct Answer A : Choice (A) is correct. It links sentence 10 to the rest of the paragraph by explaining the harmlessness of the "ritual display" mentioned in the previous sentence (and thus clarifies the contrast between Smith's account and the probable facts).

Explanation for Incorrect Answer B : Choice (B) is unsatisfactory because sentence 10 outlines a scenario that challenges Smith's "life-or-death" account, implying that Smith is not a credible source.

Explanation for Incorrect Answer C : Choice (C) is unsatisfactory because the inserted phrase unhelpfully interrupts the connection between the "ritual display" introduced in sentence 9 and the explanation of it in sentence 10.

Explanation for Incorrect Answer D : Choice (D) is unsatisfactory because the use of "and" implies that the "ritual" and the "drama" are two different events, whereas the "drama" actually refers to the "ritual display."

33

Which of the following best describes the relationship between sentences 9 and 10?
(A)

(B)

Sentence 10 adds to information reported in sentence 9.

(D) Sentence 10 poses an argument that contradicts the point made in sentence 9.

(E)

nR
ANSWERS

Sentence 10 introduces a new source that confirms the claims made in sentence 9.

Explanation for Correct Answer B : Choice (B) is correct. Sentence 10 elaborates on the information about what may have really happened to Smith presented in sentence 9.

Explanation for Incorrect Answer A : Choice (A) is unsatisfactory because sentence 10 offers only support for the claim made in sentence 9.

Explanation for Incorrect Answer C : Choice (C) is unsatisfactory because the information in sentence 10 is not an "example"; rather, it is a reasoned clarification of what may have happened to Smith.

Explanation for Incorrect Answer D : Choice (D) is unsatisfactory because nothing about sentence 10 contradicts sentence 9.

Explanation for Incorrect Answer E : Choice (E) is unsatisfactory because sentence 10 does not make use of any new sources.

eg i

(C) Sentence 10 provides an example to illustrate an idea presented in sentence 9.

AND EXPLANATIONS

st

Sentence 10 concludes that the theory mentioned in sentence 9 is wrong.

er ed

Explanation for Incorrect Answer E : Choice (E) is unsatisfactory because nothing in sentence 10 is contrary to sentence 9; the latter sentence logically follows the former.

file://E:\\f5.htm

2006-11-12

The Official SAT Online Course

20/21

34

Which of the following would be the best to insert before sentence 11 to introduce the third paragraph?

(A)

It is crucial to consider the political successes as well as the shortcomings of Pocahontas.

(B)

The Pocahontas of legend is the most interesting, but the historical Pocahontas is more believable.

(C) If legend has overemphasized the bravery of Pocahontas, it has underplayed her political talents.

(D) To really know Pocahontas, we must get beyond myth and legend to the real facts about her private life.

(E)

Perhaps we will never really know the real Pocahontas.

ANSWERS

AND EXPLANATIONS

Explanation for Correct Answer C : Choice (C) is correct. The third paragraph gives two detailed examples of Pocahontas's political successes in later life.

Explanation for Incorrect Answer A : Choice (A) is unsatisfactory because the passage does not mention any of Pocahontas's shortcomings.

Explanation for Incorrect Answer B : Choice (B) is unsatisfactory because focusing on the believability of historical facts is odd and unnecessary.

Explanation for Incorrect Answer E : Choice (E) is unsatisfactory because the third paragraph gives detailed information about Pocahontas that is not in dispute.

nR
(A)

35

What information is most logical to add immediately after sentence 12 ?

(B)

How Rolfe and Pocahontas happened to meet and marry

Details about other versions of the legend concerning John Smith

(C) Reasons for the confrontations between the Powhatans and the Jamestown settlers

(D) An account of Rolfes life and work in Virginia

(E)

A brief summary of the other public events in Pocahontas life

ANSWERS

Explanation for Correct Answer E : Choice (E) is correct. This information is consistent with the information presented in the first two sentences of the paragraph.

Explanation for Incorrect Answer A : Choice (A) is unsatisfactory because the first two sentences of the paragraph present events chronologically, and it would be illogical to describe Pocahontas's marriage, which occurred before the events described in sentence 12, at the end of the paragraph

Explanation for Incorrect Answer B : Choice (B) is unsatisfactory because the passage has moved from a discussion of Smith's account to a discussion of Pocahontas's life; to return to Smith at this point would be illogical.

eg is

AND EXPLANATIONS

te

Explanation for Incorrect Answer D : Choice (D) is unsatisfactory because the information in paragraph 3 deals primarily with Pocahontas's public life, not her private life.

re d

file://E:\\f5.htm

2006-11-12

The Official SAT Online Course


Explanation for Incorrect Answer C : Choice (C) is unsatisfactory because this information would need to be presented directly after its introduction in sentence 11.

21/21

Explanation for Incorrect Answer D : Choice (D) is unsatisfactory because the focus of the paragraph is Pocahontas, not her husband.

Back to Score Report

Copyright 2006 The College Board. All rights reserved.

Privacy Policy

Terms of Use

Contact Us

nR

eg

is

te

re d

file://E:\\f5.htm

2006-11-12

The Official SAT Online Course

1/11

Help | Profile | My Organizer | My Bookmarks | Logout

Answers and Explanations

Test Sections

Back to Score Report

Section 1

View Answers and Explanations


Online - Practice Test #6

Section 2

Section 4

Section 5

Section 6

Section 7

Section 8

Section 9

Section 10

The figure above shows five lines. If

which of the following is NOT equal to 90 ?

(B)

(C) t

(D) u

(E)

ANSWERS

AND EXPLANATIONS

Explanation for Correct Answer A : , and since the angle with measure Choice (A) is correct. Since

eg i

st

er

ed

.
are corresponding angles,

(A)

and the

angle with measure

. Therefore,

is not

equal to

nR

Explanation for Incorrect Answer B : Choice (B) is not correct. Since the angle with measure

is supplementary to a

right angle,

Explanation for Incorrect Answer C : , and since the angle with measure Choice (C) is not correct. Since

and

the angle with measure

are corresponding angles,

. Since the angle with

measure

is supplementary to a right angle, it follows that

. So

Explanation for Incorrect Answer D : , the angle with measure Choice (D) is not correct. Since

and a right

angle are corresponding angles. So

Explanation for Incorrect Answer E : Choice (E) is not correct. Since the angle with measure

is supplementary to a

right angle, it follows that

Which of the following is divisible by 3 and by 5 but is not divisible by 10 ?


(A)

30

(B)

35

(C) 40

file://E:\\f6.htm

2006-11-12

The Official SAT Online Course

2/11

(D) 45

(E)

60

ANSWERS

AND EXPLANATIONS
,

Explanation for Correct Answer D :

Choice (D) is correct. Since

is divisible by

and

. However,

is not divisible by

Explanation for Incorrect Answer A : is divisible by , Choice (A) is not correct. Since . is also divisible by , However, since

and

Explanation for Incorrect Answer B : , Choice (B) is not correct. Since

is divisible by

but not divisible

by

Explanation for Incorrect Answer C :

Choice (C) is not correct. Since

is divisible by

by

re

Explanation for Incorrect Answer E :

Choice (E) is not correct. Since

is divisible by

However, since

is also divisible by

but not divisible

d
. Since

and

nR
(A)

The table above gives values of the function f for several values of t. If the graph of f ? is a line, Which of the following defines

(B)

(C)

(D)
(E)

ANSWERS

Explanation for Correct Answer D :

eg is

AND EXPLANATIONS
is a linear function, it has the form

te

Choice (D) is correct. Since

where

and

are constants. From the table, you have

, so

, and thus

, the equation

holds, so

, and thus

. Therefore,

Explanation for Incorrect Answer A :

Choice (A) is not correct. From the table, you have

. If

were defined

, then

. But then

would be equal to

, which is false. So

cannot define

file://E:\\f6.htm

2006-11-12

The Official SAT Online Course

3/11

Explanation for Incorrect Answer B :

Choice (B) is not correct. From the table, you have

. If

were defined as

, then

. But then

would be equal to

, which is false. So

cannot define

Explanation for Incorrect Answer C :

Choice (C) is not correct. From the table, you have

. If

were defined as

, then

. But then

would be equal to

, which is false. So

cannot define

Explanation for Incorrect Answer E :

Choice (E) is not correct. From the table, you have

. If

were defined

as

, then

. But then

would be equal to

, which is

false. So

cannot define

(B)

Segment

(D) Ray

(E)

Line

ANSWERS

Explanation for Correct Answer B : is the set whose members are point Choice (B) is correct. Ray

eg

AND EXPLANATIONS
and all

and

nR

is the set whose members are point on the line. Ray points to the right of on the line. The intersection of these two sets is and all points to the left of and all points on the line between and the set whose members are points . . This set of points is segment and

Explanation for Incorrect Answer A : is the set whose members are points Choice (A) is not correct. Segment

is te

(C) Ray

re

and

(A)

Segment

d
. Ray

In the figure above, the intersection of ray

and ray


is

and all points on the line between

is the set whose

members are point

and all points to the left of

on the line. Any point between

and

on the line is a member of segment

but not a member of ray

Therefore, segment

cannot be the intersection of ray

and ray

Explanation for Incorrect Answer C : is the set whose members are point Choice (C) is not correct. Ray

and all

is the set whose members are point on the line. Ray points to the right of on the line. Any point on the line to the right of and all points to the left of

point

is a member of ray

, but not a member of ray

. Therefore, ray

cannot be the intersection of ray

and ray

Explanation for Incorrect Answer D : is the set whose members are point Choice (D) is not correct. Ray

and all

is the set whose members are point on the line. Ray points to the right of on the line. Any point on the line to the left of and all points to the left of

point

is a member of ray

, but not a member of ray

. Therefore, ray

cannot be the intersection of ray

and ray

Explanation for Incorrect Answer E :

file://E:\\f6.htm

2006-11-12

The Official SAT Online Course

4/11

is the set whose and ray Choice (E) is not correct. The intersection of ray . Any and and all points on the line between and members are points

point on line

to the left of point

or to the right of point

is not in the

intersection of ray

and ray

. Therefore, line

cannot be the

intersection of ray

and ray

According to the graph above, if there are 6,000 registered voters aged 60 and over in Washington County, how many registered voters are under the age of 30?
(A)

1,000

(B)

2,000

(C) 3,000

(D) 4,000

(E)

5,000

Explanation for Correct Answer E : Choice (E) is correct. According to the graph,

eg

gives

ANSWERS

AND EXPLANATIONS

nR

Washington County are age

; this is

is

te r

of
or over. If

ed

is

of the registered voters in

is the total number of registered

voters in Washington County, then

Solving for

of

under the age of

to

. So

. Of all the registered voters,

are of ages

, and

are under age

. Thus,

of the registered voters are

of

, which is

Explanation for Incorrect Answer A : Choice (A) is not correct. From the explanation for the correct response (E), you registered voters, and from the graph, you have that there are a total of

have that

of these registered voters are under the age

. Since

of

is

, the number of registered voters under the age

is

. However, you were to find the number of registered voters under

the age of

Explanation for Incorrect Answer B : Choice (B) is not correct. See the explanation for the correct response (E).

Explanation for Incorrect Answer C : Choice (C) is not correct. See the explanation for the correct response (E).

Explanation for Incorrect Answer D : Choice (D) is not correct. From the explanation for the correct response (E), you registered voters, and from the graph, you have that there are a total of

have that

of these registered voters are of ages

to

. Since

of

file://E:\\f6.htm

2006-11-12

The Official SAT Online Course

5/11

is

, the number of registered voters of ages of

to

is

. However, you were to find the number of registered voters under

the age of

, which includes those under the age of

Based on the graph of the function f above, what are the values of x for which f(x) is positive?
(A)

(B)

or

(C)

or

(D)

(E)

ANSWERS

AND EXPLANATIONS

eg

or
and

Explanation for Correct Answer C : Choice (C) is correct. From the graph of

strictly between

nR

greater than

and less than or equal to

is
, the value of

te

and

re d

is greater than

or

, you can see that for any value of

. For any value of

, the value of

is greater than

For any other value of

between

inclusive, the value of

is less

than or equal to

which

. So the values of

for which

is positive are those for

Explanation for Incorrect Answer A : Choice (A) is not correct. For the values of

such that

, the value of

is positive. However, if

, the value of

is negative.

Explanation for Incorrect Answer B : Choice (B) is not correct. The values of

for which

or

are

the values of

for which

is negative.

Explanation for Incorrect Answer D : Choice (D) is not correct. For some values of

in the interval

the

value of

is positive. However, this is not true for all values of

in that

interval. For example, if

, then

is in the interval

, but you can

see from the graph that

Explanation for Incorrect Answer E : Choice (E) is not correct. For some values of

in the interval

the value

of

is positive. However, this is not true for all values of

in that interval. For

example, if

, then

is in the interval

but you can see from the

graph that

file://E:\\f6.htm

2006-11-12

The Official SAT Online Course

6/11

Bernardo drives to work at an average speed of 50 miles per hour and returns along the same route at an average speed of 25 miles per hour. If his total travel time is 3 hours, what is the total number of miles in the round-trip?
(A)

225

(B)

112.5

(C) 100

(D) 62.5

(E)

50

ANSWERS

AND EXPLANATIONS

Explanation for Correct Answer C : miles long, and if he takes Choice (C) is correct. If Bernardos route to work is miles per hour, then hours to drive to work at an average speed of

. His total travel time going to and from work is

hours, so he takes

hours to drive back. Since he returns along the same route,

Therefore,

. Solving this equation gives

. Since

, the value of

is

. The total number of miles in the round-trip is

thus

Explanation for Incorrect Answer A : Choice (A) is not correct. See the explanation for the correct response (C).

and

hours traveling at

nR eg

lower speed. Therefore, his average speed for the round-trip was less than miles per hour.)

Explanation for Incorrect Answer D : Choice (D) is not correct. See the explanation for the correct response (C).

Explanation for Incorrect Answer E : Choice (E) is not correct. The number of miles Bernardo traveled each way

is

Explanation for Incorrect Answer B : Choice (B) is not correct. See the explanation for the correct response (C). (Keep in mind that Bernardos average speed for the round-trip was not the average of miles per hour hour traveling at miles per hour. Since he spent and

miles per hour, he spent twice as much time at the

te

re

d
the total number of miles in the round-trip was

, but

I.

If x and y are integers such that

and

which of the following could

be true?

II.

III.

(A)

I only

(B)

II only

(C) I and III only

(D) II and III only

(E)

I, II, and III

ANSWERS

AND EXPLANATIONS

Explanation for Correct Answer C :

file://E:\\f6.htm

2006-11-12

The Official SAT Online Course

7/11

Choice (C) is correct. If

is an integer such that

, then

or

. So

could be

, which means

could be true. If

is an integer such

that

, then

must be

, so

cannot be true.

could be true,

because

could be

and

is

, in which case

. So

could be true and

could be true, but

cannot be true.

Explanation for Incorrect Answer A : Choice (A) is not correct. From the explanation for the correct answer, could also be true. true, but

could be

Explanation for Incorrect Answer B : cannot be Choice (B) is not correct. From the explanation for the correct answer, could be true. could be true and . Also, must be true. The value of

Explanation for Incorrect Answer D : Choice (D) is not correct. From the explanation for the correct answer, could be true. true. Also,

cannot be

Explanation for Incorrect Answer E : Choice (E) is not correct. From the explanation for the correct answer, cannot be true. The value of could be true. However, true and .

When a certain number is multiplied by

the result is 60. What is the number?

Your Response:

Explanation:

nR

eg

Correct Response(s): 7.5, 15/2

The correct answer is

multiplied by

is
or

te

. If

re

, then

could be must be

and the product is then multiplied by 32,

d
and the product is then multiplied by

is the number, then when

is

, the result is

. So

. Therefore,

, so the number is

10

What is the greatest integer value of x for which

Your Response:

Correct Response(s): 9

Explanation:

The correct answer is

. If

, and so

. The

greatest integer value of

for which

is

11

An object thrown upward from a height of h feet with an initial velocity of v feet per

second will reach a maximum height of

feet. If the object is thrown upward

from a height of 6 feet with an initial velocity of 32 feet per second, what will be its maximum height, in feet?

file://E:\\f6.htm

2006-11-12

The Official SAT Online Course

8/11

Your Response:

Correct Response(s): 22

Explanation:

The correct answer is

with an initial velocity of

feet, . If the object is thrown upward from a height of in the and feet per second, then

formula

. The formula gives the maximum height, in feet, that the object

will reach, so this height, in feet, is

12

The three angles of a triangle have measures of

and

, where

If x

and y are integers, what is one possible value of

Your Response:

Correct Response(s): 3 or 6 or 9 or 12

Explanation:

The correct answer is

, or

er e
so

. The sum of the measures of the three

angles of a triangle is equal to

then

so

Since

st

are

it follows that

. So the only possible values for

and

d
,

If

are positive integers and

must be a multiple of

, or

13

nR

Explanation:

The incomplete table above is an expense sheet for Carmens business trip. If her hotel expenses were the same each day, what were her total expenses for Friday, in dollars? (Disregard the $ sign when gridding your answer.)

Your Response:

Correct Response(s): 96

eg i

The correct answer is

. If Carmens hotel expenses were

dollars each day,

then her total expenses in dollars for the trip were

. The table

gives her total expenses as

, so

. Solve this

equation for

, so

, and

. Carmens total

expenses for Friday, in dollars, were

14

file://E:\\f6.htm

2006-11-12

The Official SAT Online Course

9/11

In

above,

and

What is the length of

Your Response:

Correct Response(s): 8

Explanation:

is the correct answer. From the information in the figure, you know that

is a right triangle. Since

and

, by the Pythagorean theorem, the

length of

is

. Since

also a right angle, and

is a right triangle. Since

by the Pythagorean theorem, the length of

Your Response:

had

nR

Correct Response(s): 1024

Explanation:

The correct answer is

eg i

st

15

On Monday morning Mr. Smith had a certain amount of money that he planned to spend during the week. On each subsequent morning, he had one fourth the amount of the previous morning. On Saturday morning, 5 days later, he had $1. How many dollars did Mr. Smith originally start with on Monday morning? (Disregard the $ sign when gridding your answer.)

. If Mr. Smith had

er

ed
is

is a right angle,

is

and

dollars on Monday morning and

had one fourth that amount on Tuesday morning, then he had

dollars on

Tuesday morning. On Wednesday morning, he had one fourth the amount he had

on Tuesday morning, or

dollars. Continuing in this way, you see that he

dollars on Saturday morning. If he had

on

Saturday morning, then

. So

16

The median of a list of 99 consecutive integers is 60. What is the greatest integer in the list?

Your Response:

Correct Response(s): 109

Explanation:

file://E:\\f6.htm

2006-11-12

The Official SAT Online Course

10/11

The correct answer is

, then there are

consecutive integers is . If the median of a list of integers and integers in the list that are less than

in the list that are greater than

. Since the integers are consecutive, the

greatest integer in the list is

17

When the positive integer m is divided by 5, the remainder is 3. What is the remainder when 20m is divided by 25?

Your Response:

Correct Response(s): 10

Explanation:

The correct answer is

. When the positive integer

is divided by

, the

remainder is

. So there is a positive integer

such that

. Then

. Therefore, the remainder is

when

is divided by

nR eg
Your Response:

The figure above shows three squares with sides of length 5, 7, and x, respectively. If what is the value of x ? A, B, and C lie on line

Correct Response(s): 49/5, 9.8

Explanation:

The correct answer is

is
or

te
.

. From the figure and the information in the

re

, so

question, you can draw the figure above. Triangle

d
, because they are right triangles with

18

is similar to triangle

. To find

you can set up and solve the proportion

. You have

and

, so

. Therefore,

, and

Back to Score Report

Copyright 2006 The College Board. All rights reserved.

Privacy Policy

Terms of Use

Contact Us

file://E:\\f6.htm

2006-11-12

The Official SAT Online Course

11/11

nR

eg is te r

file://E:\\f6.htm

ed

2006-11-12

The Official SAT Online Course

1/21

Help | Profile | My Organizer | My Bookmarks | Logout

Answers and Explanations

Test Sections

Back to Score Report

Section 1

View Answers and Explanations


Online - Practice Test #6

Section 2

Section 4

Section 5

Section 6

The success of Notes of a Native Son ------- author James Baldwin as one of the most ------- essayists of his time. (A) buoyed . . irrelevant
(B)

Section 7

Section 8

established . . prominent

Section 9

(C) surrendered . . prolific

Section 10

(D) decried . . cynical

(E)

categorized . . mundane

nR

(A)

Explanation for Incorrect Answer A : Choice (A) is incorrect. Buoyed means supported or uplifted. Irrelevant means unrelated or insignificant. If one were to insert these terms into the text, the sentence would read The success of Notes of a Native Son buoyed author James Baldwin as one of the most irrelevant essayists of his time. Success can enhance a writers reputation, but it is illogical to claim that it would cause Baldwin to become insignificant.

Explanation for Incorrect Answer C : Choice (C) is incorrect. Surrendered means abandoned. Prolific means to produce abundant works or results. If one were to insert these terms into the text, the sentence would read The success of Notes of a Native Son surrendered author James Baldwin as one of the most prolific essayists of his time. Many successful authors are prolific, but it does not make sense to say that success abandoned Baldwin as a prolific essayist.

Explanation for Incorrect Answer D : Choice (D) is incorrect. Decried means denounced or belittled. Cynical means negative or pessimistic. If one were to insert these terms into the text, the sentence would read The success of Notes of a Native Son decried author James Baldwin as one of the most cynical essayists of his time. Some critics may have criticized Baldwin for being cynical, but a book cannot denounce its author.

Explanation for Incorrect Answer E : Choice (E) is incorrect. Categorized means classified, and mundane means ordinary. If one were to insert these terms into the text, the sentence would read The success of Notes of a Native Son categorized author James Baldwin as one of the most mundane essayists of his time. It is unlikely that success would classify or label an author as ordinary.

In many parts of the world, people use rice as a central rather than a ------- part of their daily diets.

pivotal

eg is

te r

Explanation for Correct Answer B : Choice (B) is correct. Establish means to cause to be recognized and accepted. Prominent means widely and popularly known. If one were to insert these terms into the text, the sentence would read The success of Notes of a Native Son established author James Baldwin as one of the most prominent essayists of his time. The success of Notes of a Native Son gave Baldwin recognition and acceptance within literary circles.

ed

ANSWERS

AND EXPLANATIONS

file://E:\\f7.htm

2006-11-12

The Official SAT Online Course

2/21

(B)

ritualistic

(C) salient

(D) supplementary

(E)

solemn

ANSWERS

AND EXPLANATIONS

Explanation for Correct Answer D : Choice (D) is correct. "Supplementary" means something added or in addition to. If one were to insert this term into the text, the sentence would read "In many parts of the world, people use rice as a central rather than a supplementary part of their daily diets." The phrase "rather than" indicates that the blanked word will mean the opposite of the word "central." In this context, "supplementary" does indeed have the opposite meaning of "central."

Explanation for Incorrect Answer A : Choice (A) is incorrect. "Pivotal" means being of vital or significant importance. If one were to insert this term into the text, the sentence would read "In many parts of the world, people use rice as a central rather than a pivotal part of their daily diets." "Pivotal" and "central" are synonymous, thus it makes little sense to claim that people use rice as a central rather than a "pivotal" part of their diet.

nR eg

(A)

Explanation for Incorrect Answer C : Choice (C) is incorrect. "Salient" means prominent. If one were to insert this term into the text, the sentence would read "In many parts of the world, people use rice as a central rather than a salient part of their daily diets." A "salient" part of a diet would be an important part. It is illogical to contrast the use of rice as a "central" part of the diet with the use of rice as an important part of the diet.

Explanation for Incorrect Answer E : Choice (E) is incorrect. "Solemn" means somberly or gravely impressive. If one were to insert this term into the text, the sentence would read "In many parts of the world, people use rice as a central rather than a solemn part of their daily diets." It makes little sense to describe rice as a "solemn," or somber, part of a diet.

Victor gained a reputation for being a ------- because he constantly bullied other children.

bungler

(B)

ruffian

(C) stickler

(D) daredevil

(E)

naysayer

ANSWERS

AND EXPLANATIONS

Explanation for Correct Answer B : Choice (B) is correct. "Ruffian" describes a person who is a hoodlum or bully. If one were to insert this term into the text, the sentence would read "Victor gained a reputation for being a ruffian because he constantly bullied other children." Bullying other children is precisely the behavior that would give Victor a reputation as a ruffian.

is

te r

Explanation for Incorrect Answer B : Choice (B) is incorrect. "Ritualistic" means advocating or practicing ritual. If one were to insert this term into the text, the sentence would read "In many parts of the world, people use rice as a central rather than a ritualistic part of their daily diets." A "ritualistic," or ceremonial, use of rice does not necessarily mean that the rice is not a "central" part of the diet.

ed

file://E:\\f7.htm

2006-11-12

The Official SAT Online Course

3/21

Explanation for Incorrect Answer A : Choice (A) is incorrect. "Bungler" describes a person who is clumsy or inept in behavior. If one were to insert this term into the text, the sentence would read "Victor gained a reputation for being a bungler because he constantly bullied other children." The claim that Victor had a reputation for being a "bungler" does not fit with the claim that "he bullied other children," as a bully is not necessarily clumsy.

Explanation for Incorrect Answer C : Choice (C) is incorrect. "Stickler" describes a person who insists on something unyieldingly. If one were to insert this term into the text, the sentence would read "Victor gained a reputation for being a stickler because he constantly bullied other children." Although Victor may be a "stickler," or perfectionist, his reputation is based on his tendency to bully other children.

Explanation for Incorrect Answer D : Choice (D) is incorrect. "Daredevil" describes a person who is recklessly bold. If one were to insert this term into the text, the sentence would read "Victor gained a reputation for being a daredevil because he constantly bullied other children." While Victor may be a "daredevil," or risktaker, the sentence offers no evidence of this.

Explanation for Incorrect Answer E : Choice (E) is incorrect. "Naysayer" describes a person who opposes in order to take a pessimistic view. If one were to insert this term into the text, the sentence would read "Victor gained a reputation for being a naysayer because he constantly bullied other children." The sentence offers no evidence that Victor is a "naysayer," or contrarian.

Paradoxically, the senator was both a ------- and -------: she publicly defended the rights and wisdom of the people, but she often spoke with a disdainful air of superiority.
(A)

demagogue . . a maverick

(B)

conservative . . an anarchist

(C) populist . . an elitist

(E)

partisan . . a snob

nR

ANSWERS

Explanation for Correct Answer C : Choice (C) is correct. A "populist" is an advocate for the interests and rights of the common people. An "elitist" believes that certain persons deserve favored treatment by virtue of their perceived superiority. If one were to insert these terms into the text, the sentence would read "Paradoxically, the senator was both a populist and an elitist: she publicly defended the rights and wisdom of the people, but she often spoke with a disdainful air of superiority." The term "paradoxically" indicates that the two missing terms will be contradictory. The senator's behavior is inconsistent because she supports common people publically, while believing in the superiority of certain classes of people.

Explanation for Incorrect Answer A : Choice (A) is incorrect. A "demagogue" is a leader who obtains power by means of impassioned appeals to the emotions and prejudices of the populace. A "maverick" is one that resists adherence to a group. If one were to insert these terms into the text, the sentence would read "Paradoxically, the senator was both a demagogue and a maverick: she publicly defended the rights and wisdom of the people, but she often spoke with a disdainful air of superiority." A "demagogue," a person who stirs up emotion, can also be a "maverick" or independent politician. The two are not necessarily opposed to one another, and so the behavior of a person who exhibits both of these characteristics would not logically be described as paradoxical.

Explanation for Incorrect Answer B : Choice (B) is incorrect. "Conservative" is one favoring traditional values and views. "Anarchist" is an advocate of political disorder and confusion. If one were to insert these terms into the text, the sentence would read "Paradoxically, the senator was both a conservative and an anarchist: she publicly defended the rights and wisdom of the people, but she often spoke with a disdainful air of superiority." A

eg

(D) moderate . . a reactionary

AND EXPLANATIONS

is

te

re d

file://E:\\f7.htm

2006-11-12

The Official SAT Online Course

4/21

"conservative" politician does not necessarily defend the rights of the common people. An "anarchist" opposes the idea of government and is unlikely to hold a government office.

Explanation for Incorrect Answer D : Choice (D) is incorrect. "Moderate" is one who holds average views or opinions. "Reactionary" means an opponent of progress or liberalism. If one were to insert these terms into the text, the sentence would read "Paradoxically, the senator was both a moderate and a reactionary: she publicly defended the rights and wisdom of the people, but she often spoke with a disdainful air of superiority." "Moderate," or middle-ground, and "reactionary" are not the polar opposites implied by the term "paradoxically."

Explanation for Incorrect Answer E : Choice (E) is incorrect. "Partisan" is a supporter or proponent of a party, cause, faction, person, or idea. A "snob" looks down on people he or she regards as socially inferior. If one were to insert these terms into the text, the sentence would read "Paradoxically, the senator was both a partisan and snob: she publicly defended the rights and wisdom of the people, but she often spoke with a disdainful air of superiority." While a "snob" would speak with an air of superiority, it makes little sense to contrast a "snob" with a "partisan," or someone favoring a particular political party. Furthermore, a partisan would not necessarily defend the rights of the common people.

(A)

inundated

(B)

situated

(C) rejuvenated

(D) supplanted

(E)

excavated

nR

Explanation for Correct Answer A : Choice (A) is correct. "Inundated" means to cover completely as in a flood. If one were to insert this term into the text, the sentence would read "The geologist speculated that eons ago, before the area was inundated, the present-day island was actually a hilltop in a vast forest." The words "before" and "actually" in this sentence suggest that something happened to transform the hilltop into an island. An inundation of water would be the most logical cause of such a transformation.

Explanation for Incorrect Answer B : Choice (B) is incorrect. "Situated" means having a location. If one were to insert this term into the text, the sentence would read "The geologist speculated that eons ago, before the area was situated, the present-day island was actually a hilltop in a vast forest." The location, or where the island is "situated," has not changed.

Explanation for Incorrect Answer C : Choice (C) is incorrect. "Rejuvenated" means restored to an original or new condition. If one were to insert this term into the text, the sentence would read "The geologist speculated that eons ago, before the area was rejuvenated, the present-day island was actually a hilltop in a vast forest." While parts of the area may have been rejuvenated, or restored, this would not have turned the area from a hilltop into an island.

Explanation for Incorrect Answer D : Choice (D) is incorrect. "Supplanted" means displaced and substituted for. If one were to insert this term into the text, the sentence would read "The geologist speculated that eons ago, before the area was supplanted, the present-day island was actually a hilltop in a vast forest." The information in the sentence does not support the idea that the area was supplanted, or displaced.

Explanation for Incorrect Answer E : Choice (E) is incorrect. "Excavated" means removed or exposed by digging. If one were to insert this term into the text, the sentence would read "The geologist speculated that eons ago, before the area was excavated, the present-day island

eg

ANSWERS

AND EXPLANATIONS

is

te re

The geologist speculated that eons ago, before the area was -------, the present-day island was actually a hilltop in a vast forest.

file://E:\\f7.htm

2006-11-12

The Official SAT Online Course


was actually a hilltop in a vast forest." Although the area may have been excavated, or exhumed, this does not explain how the hilltop became an island.

5/21

Passage 1

Passage 2

Estimates of the number of humans that Earth can sustain have ranged in recent decades from fewer than a billion to more than a trillion. Such elasticity 15 is probably unavoidable, since carrying capacity is essentially a subjective term. It makes little sense to talk about carrying capacity in relationship to humans, who are capable of adapting and altering both their culture and their physical environment, and can thus defy any formula 20 that might settle the matter. The number of people that Earth can support depends on how we on Earth want to live, on what we want to consume, and on what we regard as

nR

eg is

Any wildlife biologist can tell you how many deer a given area can supporthow much browse there is for the deer to eat before they begin to suppress the reproduction of trees, before they begin to Line starve in the winter. Any biologist can calculate how 5 many wolves a given area can support too, in part by counting the number of deer. And so on, up and down the food chain. Its not an exact science, but it comes pretty closeat least compared to figuring out the carrying capacity of Earth for human 10 beings, which is an art so dark that anyone with any sense stays away from it.

te r

ed

file://E:\\f7.htm

2006-11-12

The Official SAT Online Course

6/21

a crowd.
6

Both passages support which of the following conclusions about Earths carrying capacity for humans?
(A)

It is routinely underestimated by biologists.

(B)

It cannot be easily determined, given numerous variables and unknowns.

(C) It has only recently become the subject of considerable scientific debate.

(D) It is a valuable concept despite its apparent shortcomings.

(E)

It has increased as a result of recent technological innovations.

ANSWERS

AND EXPLANATIONS

Explanation for Correct Answer B : Choice (B) is correct. Both passages discuss the difficulty of accurately estimating how many people the Earth can support. Passage 1 asserts that estimating the Earth's "carrying capacity" is a "dark art," and Passage 2 explains why such estimates are problematic.

nR

(A)

Explanation for Incorrect Answer E : Choice (E) is incorrect. The first passage does not discuss recent "technological innovations" at all, and the second passage supports this conclusion indirectly at best.

The author of Passage 1 refers to Any wildlife biologist in line 1 and Any biologist in line 5 to emphasize the point that

a particular type of calculation can be made with great confidence

(B)

scientific findings often meet with resistance from the general public

(C) certain beliefs are rarely questioned by scientists

(D) most biologists are concerned with issues related to wildlife mortality

(E)

all biologists must be skilled at applying mathematical formulas

ANSWERS

Explanation for Correct Answer A : Choice (A) is correct. The author of Passage 1 contrasts the ease of calculating how many animals an area can support with the difficulty of making such a calculation for people. The use of the word "any" conveys the author's belief that predicting an animal population's "carrying capacity" is a relatively simple task for biologists.

Explanation for Incorrect Answer B : Choice (B) is incorrect. Passage 1 does not include any reference to the "general public."

eg

Explanation for Incorrect Answer D : Choice (D) is incorrect. Both passages clearly question the value of estimating the Earth's "carrying capacity" for humans. Passage 1 declares that "anyone with any sense stays away" from the concept, and Passage 2 says that "it makes little sense to talk about carrying capacity in relationship to humans."

AND EXPLANATIONS

is te

Explanation for Incorrect Answer C : Choice (C) is incorrect. Nothing in these passages suggests that the debate regarding human "carrying capacity" is a new one.

re

Explanation for Incorrect Answer A : Choice (A) is incorrect. Passage 2 asserts that biologists' estimations of the Earth's "carrying capacity" for humans cover a broad range, but neither author claims that these existing "estimates" are too low.

file://E:\\f7.htm

2006-11-12

The Official SAT Online Course

7/21

Explanation for Incorrect Answer C : Choice (C) is incorrect. The author questions biologists' ability to accurately estimate the Earth's "carrying capacity" for humans but makes no reference to "beliefs" of any kind.

Explanation for Incorrect Answer D : Choice (D) is incorrect. It may be true that the majority of biologists care about "wildlife mortality," but the author of Passage 1 makes no such claim.

Explanation for Incorrect Answer E : Choice (E) is incorrect. While one might infer that the biologists mentioned in Passage 1 need to be skilled in mathematics, this is certainly not a point of emphasis in the passage.

Both authors would agree that the Estimates (Passage 2, line 13) are
(A)

overly generous

(B)

largely undocumented

(C) often misunderstood

(D) politically motivated

(E)

essentially unreliable

ANSWERS

AND EXPLANATIONS

nR

(A)

Explanation for Incorrect Answer B : Choice (B) is incorrect. Neither author claims that the "estimates" in Passage 2 are generally "undocumented."

Explanation for Incorrect Answer C : Choice (C) is incorrect. While the authors question the accuracy of certain carrying capacity "estimates," there is no suggestion that these figures have been "misunderstood."

Explanation for Incorrect Answer D : Choice (D) is incorrect. Neither author ascribes a political motivation to the scientists responsible for the "estimates" mentioned in Passage 2.

Which of the following best describes the relationship between the two passages?

Passage 1 offers a hypothesis that is explicitly refuted in Passage 2.

(B)

Passage 1 describes a popular misconception that is exemplified by Passage 2.

(C) Passage 2 presents an argument that elaborates on a point made in Passage 1.

(D) Passage 2 defends a position that is attacked in Passage 1.

(E)

Passage 2 provides an anecdote that confirms the theory advanced in Passage 1.

ANSWERS

Explanation for Correct Answer C :

eg

Explanation for Incorrect Answer A : Choice (A) is incorrect. Passage 2 asserts that biologists' estimations of the Earth's "carrying capacity" for humans cover a broad range, but neither author claims that these existing estimates are too high.

AND EXPLANATIONS

is

te

Explanation for Correct Answer E : Choice (E) is correct. Passage 2 highlights the "elasticity" of scientists' "estimates," suggesting that such figures are fundamentally "unreliable." The author of Passage 1 echoes this assertion in describing the estimation of the Earth's "carrying capacity" for humans as an inexact science.

re

file://E:\\f7.htm

2006-11-12

The Official SAT Online Course


Choice (C) is correct. Passage 1 claims that estimating the Earth's "carrying capacity" for humans is "not an exact science." Passage 2 elaborates on this point by explaining how "carrying capacity" is a subjective concept when applied to humans.

8/21

Explanation for Incorrect Answer A : Choice (A) is incorrect. Passage 2 supports, not refutes, the claim made in Passage 1 that scientists should stay away from "carrying capacity" estimates.

Explanation for Incorrect Answer B : Choice (B) is incorrect. While the notion that human sustainability can be easily estimated may be a popular misunderstanding, Passage 2 counters, rather than exemplifies, such a "misconception."

Explanation for Incorrect Answer D : Choice (D) is incorrect. Passage 1 criticizes scientists who attempt to calculate the Earth's "carrying capacity" for humans, but the author of Passage 2 agrees with this criticism.

Explanation for Incorrect Answer E : Choice (E) is incorrect. Passage 2 contains nothing that resembles an "anecdote," or retelling of an interesting incident.

The passage below is excerpted from the introduction to a collection of essays published in 1994.

My entry into Black womens history was serendipitous. In the preface to Black Women in America: An Historical Encyclopedia, I recount the story of exactly how Shirley Herd (who, in addition to teaching in the local Line school system, was also president of the Indianapolis 5 chapter of the National Council of Negro Women) successfully provoked me into changing my research and writing focus. Although I dedicate this volume to her and to her best friend, fellow club woman and retired primary school teacher Virtea Downey, I still blush at the fact that I went to 10 graduate school to become a historian in order to contribute to the Black Struggle for social justice and yet met her request to write a history of Black women in Indiana with condescension. I had never even thought about Black women as historical subjects with their own relations to a 15 states history,

nR

eg

is t

er

ed

file://E:\\f7.htm

2006-11-12

The Official SAT Online Course

9/21

and I thought her invitation and phone call extraordinarily intrusive. Only later did I concede how straightforward and reasonable had been her request to redress a historical omission. Black women were conspicuous by their absence. None of the social studies texts or state 20 histories that Herd and Downey had used to teach their students made mention of the contributions of Black women. Since historians had left them out, Herd reasoned, only a real historian could

30

35

40

nR

eg

is

te re

25

put them in, and since I was the only tenured Black woman historian in the state of Indiana at that time, the task was mine. Herd rejected my reservations and completely ignored my admonitions that she could not call up a historian and order a book the way you drive up to a fastfood restaurant and order a hamburger. In spite of my assertions of ignorance about the history of Black women in Indiana and my confession of having never studied the subject in any history course or examined any manuscript sources pertaining to their lives, Herd persevered. Black women, as historical subjects and agents, were as invisible to me as they had been to school textbook writers. Undaunted by my response, Herd demanded that I connect (thankfully without perfect symmetry) my biology and autobiography, my race and gender, my being a Black woman, to my skill as a historian, and write for her and for the local chapter members of the National Council a history of Black women in Indiana. I relented and wrote the book,

file://E:\\f7.htm

2006-11-12

The Official SAT Online Course

10/21

10

nR
(A)

The primary purpose of the passage is to show how the author

(B)

discovered Black womens history when she was in graduate school

became a historian to help Black people in America achieve social justice

(C) developed her research skills by undertaking a challenging project

(D) became a more renowned scholar due to the influence of two interesting individuals

(E)

came to view Black women as a worthy subject for historical analysis

ANSWERS

Explanation for Correct Answer E : Choice (E) is correct. The passage mainly discusses the process by which the author comes to realize that Black women are a worthy subject of historical study.

Explanation for Incorrect Answer A : Choice (A) is incorrect. The author explains that she did not discover her interest in Black women's history until she was already a tenured historian. It was only after her discussions with Herd and Downey that the author became interested in the subject.

Explanation for Incorrect Answer B : Choice (B) is incorrect. Although the author does mention that she "went to graduate school to become a historian in order to contribute to the Black Struggle for social justice," the passage was not written to support this claim.

eg

* tenure: a permanent position, often granted to a teacher after a specified number of years of demonstrated competence

AND EXPLANATIONS

is t

When the Truth Is Told: Black Womens Culture and Community in Indiana, 1875-1950, as requested. In the process, I was both humbled and astounded by 45 the array of rich primary source materials Herd, Downey, and the other club women had spent two years collecting. There were diaries, club notes, church souvenir booklets, photographs, club minutes, birth, death, and marriage certificates, letters, and handwritten county and local histories. 50 Collectively this material revealed a universe I never knew existed in spite of having lived with Black women all of my life . . . and being one myself. Or perhaps more accurately, I knew a universe of Black women existed. I simply had not envi55 sioned its historical meaning.

er e

file://E:\\f7.htm

2006-11-12

The Official SAT Online Course

11/21

Explanation for Incorrect Answer C : Choice (C) is incorrect. The author may have honed her research skills while preparing for her book, but the primary purpose of the passage is to show the development of her beliefs as a historian.

Explanation for Incorrect Answer D : Choice (D) is incorrect. Although the author does mention the impact of Herd and Downey on her project, there is no indication that her collaboration with them enhanced her status as a historian.

11

The first sentence indicates that the authors entry (line 1) was
(A)

troublesome but worthwhile

(B)

challenging but rewarding

(C) fortunate and inevitable

(D) unexpected but agreeable

(E)

startling and provocative

ANSWERS

AND EXPLANATIONS

Explanation for Incorrect Answer A : Choice (A) is incorrect. The author would almost certainly agree that her entry into Black women's history was "worthwhile," but the term "serendipitous" does not suggest that it was particularly "troublesome."

Explanation for Incorrect Answer B : Choice (B) is incorrect. The first sentence alone does not support the claim that the author's initiation into Black women's history was "challenging."

12

nR

(A)

Explanation for Incorrect Answer C : Choice (C) is incorrect. "Serendipitous" means the opposite of "inevitable," or unavoidable.

Explanation for Incorrect Answer E : Choice (E) is incorrect. The author may have been startled to find so much value in a topic that she had previously ignored, but "serendipitous" does not mean "provocative," or stimulating.

The author initially responded to Herds request with condescension (lines 1314) because the author

knew that Herd had not been to graduate school

(B)

believed that historians should avoid controversial projects

(C) had too many other projects requiring her attention

(D) rejected Herds contention that such a history would address the Black struggle for social justice

(E)

viewed Herds request as irrelevant and presumptuous

ANSWERS

Explanation for Correct Answer E : Choice (E) is correct. The author clearly considered Herd's request "presumptuous" and "intrusive," but she also believed it to be irrelevant because she "had never even thought about Black women as historical subjects."

eg

AND EXPLANATIONS

is te r

ed

Explanation for Correct Answer D : Choice (D) is correct. The author describes her introduction to Black women's history as "serendipitous," indicating that she regarded her "entry" into the field as both unexpected and fortunate.

file://E:\\f7.htm

2006-11-12

The Official SAT Online Course

12/21

Explanation for Incorrect Answer A : Choice (A) is incorrect. The author does not mention Herd's educational credentials.

Explanation for Incorrect Answer B : Choice (B) is incorrect. The author gives no indication that she thinks historians should avoid controversy.

Explanation for Incorrect Answer C : Choice (C) is incorrect. The author does not specifically mention whether she had any other time-consuming commitments.

Explanation for Incorrect Answer D : Choice (D) is incorrect. The author does not indicate that Herd explicitly discussed a "struggle for social justice."

13

The comparison in lines 27-30 (Herd . . . hamburger) primarily demonstrates the authors belief that historians
(A)

do not usually accept pay for their work

(B)

are frequently unassuming about their profession

(C) do not generally undertake projects on request

(E)

do not generally interact with members of the public

ANSWERS

AND EXPLANATIONS

14

nR

(A)

Explanation for Incorrect Answer A : Choice (A) is incorrect. The author does not mention the issue of payment at any point in the passage.

Explanation for Incorrect Answer B : Choice (B) is incorrect. The author does not imply that historians are modest about their work. On the contrary, these lines reflect her initial arrogance regarding Herd's proposed project.

Explanation for Incorrect Answer D : Choice (D) is incorrect. While historians may spend many years working on one project, the hamburger comparison does not address this idea.

Explanation for Incorrect Answer E : Choice (E) is incorrect. Although it may be rare for a historian to accept a project idea from a member of the public, there is no evidence to support the claim that historians do not "interact with members of the public" in general.

Lines 30-34 (In spite . . . persevered) suggest that the author believed that

her lack of scholarly training on this topic was a reason to be embarrassed

(B)

primary source materials on this subject would be difficult to find

(C) historians should conduct research in the areas in which they have expertise

(D) the lives of Black women in Indiana were historically interesting and complex

(E)

Herd wanted her to conduct research on a topic of general interest

ANSWERS

eg

AND EXPLANATIONS

is

Explanation for Correct Answer C : Choice (C) is correct. The author's admonishment indicates that she was not accustomed to undertaking academic projects upon request.

te

re d

(D) spend a comparatively long time on their projects

file://E:\\f7.htm

2006-11-12

The Official SAT Online Course

13/21

Explanation for Correct Answer C : Choice (C) is correct. The author tried to excuse herself from fulfilling Herd's request by professing her ignorance of Black women's history. Inherent in this action is the assumption that historians should work within their areas of expertise.

Explanation for Incorrect Answer A : Choice (A) is incorrect. The author does not imply that her lack of scholarly training in this previously neglected subject is a source of embarrassment.

Explanation for Incorrect Answer B : Choice (B) is incorrect. While the author had not studied any primary sources before accepting Herd's proposal, she does not imply that she had assumed they would be difficult to find.

Explanation for Incorrect Answer D : Choice (D) is incorrect. The author clearly states that she had never before "thought about Black women as historical subjects."

Explanation for Incorrect Answer E : Choice (E) is incorrect. Nothing in this sentence or in the passage as a whole suggests that Black women's history was considered to be a mainstream subject.

15

(A)

knew that Black women contributed to society, but she did not understand the significance of their contributions

(B)

believed that the diversity of Black womens experiences would make them difficult to write about

(C) assumed that because Black women are not frequently studied by historians, they would not be an acceptable topic for a book

(E)

was aware of the diversity of Black womens lives, but was not willing to write about them

This passage, from a short story published in 1978, describes a visit to a planetarium,

nR

ANSWERS

Explanation for Correct Answer A : Choice (A) is correct. The final two sentences of the passage clarify the author's original misconception of Black women's role in history. She writes that she had been aware of "a universe of Black women" without comprehending its historical significance.

Explanation for Incorrect Answer B : Choice (B) is incorrect. The author does not indicate that the book was particularly challenging to write.

Explanation for Incorrect Answer C : Choice (C) is incorrect. The passage asserts that Black women's history has been widely neglected by academics, but ultimately argues that it is a worthy book topic.

Explanation for Incorrect Answer D : Choice (D) is incorrect. The author does not discuss the history of Black women's political power in these sentences or elsewhere in the passage.

Explanation for Incorrect Answer E : Choice (E) is incorrect. The last two sentences are less about diversity than they are about Black women's historical significance.

eg i

(D) believed that Black women wield political power only in contemporary times

AND EXPLANATIONS

st

er e

The last two sentences (Or perhaps . . . meaning) primarily indicate that the author

file://E:\\f7.htm

2006-11-12

The Official SAT Online Course

14/21

a building in which images of stars, planets, and other astronomical phenomena are projected onto a domed ceiling.

Inside, we sat on wonderfully comfortable seats that were tilted back so that you lay in a sort of a hammock, attention directed to the bowl of the ceiling, which soon turned dark blue, with a faint rim of light Line around the edge. There was some splendid, commanding music. 5 The adults all around were shushing the children, trying to make them stop crackling their potato chip bags. Then a mans voice, an eloquent professional voice, began to speak slowly, out of the walls. The voice reminded me a little of the way radio announcers used to introduce a piece of 10 classical music or describe the progress of the Royal Family to Westminster Abbey on one of their royal occasions. There was a faint echo-chamber effect. The dark ceiling was filled with stars. They came out not all at once but one after another, the way stars 15 really do come out at night, though more quickly. The Milky Way galaxy appeared, was moving closer; stars swam into brilliance and kept on going, disappearing beyond the edges of the sky-screen or behind my head. While the flow of light continued, the voice presented the 20 stunning facts. From a few light-years away, it announced, the Sun appears as a bright star, and the planets are not visible. From a few dozen light-years away, the Sun is not visible, either, to the naked eye. And that distancea few dozen light-yearsis only about a thousandth part of 25 the distance from the Sun to the center of our galaxy, one

nR

eg

is

te re

file://E:\\f7.htm

2006-11-12

The Official SAT Online Course

15/21

30

35

40

45

50

galaxy, which itself contains about two hundred billion stars. And is, in turn, one of millions, perhaps billions, of galaxies. Innumerable repetitions, innumerable variations. All this rolled past my head, too, like balls of lightning. Now realism was abandoned, for familiar artifice. A model of the solar system was spinning away in its elegant style. A bright bug took off from the Earth, heading for Jupiter. I set my dodging and shrinking mind sternly to recording facts. The mass of Jupiter two and a half times that of all the other planets put together. The Great Red Spot. The thirteen moons. Past Jupiter, a glance at the eccentric orbit of Pluto, the icy rings of Saturn. Back to Earth and moving in to hot and dazzling Venus. Atmospheric pressure ninety times ours. Moonless Mercury rotating three times while circling the Sun twice; an odd arrangement, not as satisfying as what they used to tell us that it rotated once as it circled the Sun. No perpetual darkness after all. Why did they give out such confident information, only to announce later that it was quite wrong? Finally, the picture already familiar from magazines: the red soil of Mars, the blooming pink sky. When the show was over I sat in my seat while children clambered over me, making no comments on anything they had just seen or heard. They were pestering their keepers for eatables and further entertainments. An effort had been made to get their attention, to take it away from canned

nR

drinks and potato chips and fix it on various


file://E:\\f7.htm

eg is te

re d

2006-11-12

The Official SAT Online Course

16/21

16

Which best describes the overall structure of the passage?


(A)

Narrative description followed by commentary

(B)

Reminiscence followed by present-day application

(C) An account of a problem followed by a suggested solution

(D) A generalization followed by specific examples

(E)

A discussion of opposing viewpoints followed by an attempt to reconcile them

nR
ANSWERS

Explanation for Correct Answer A : Choice (A) is correct. The bulk of the passage recreates the narrator's visit to a planetarium, but the text switches from "description" to "commentary" in the final paragraph.

Explanation for Incorrect Answer B : Choice (B) is incorrect. The passage seems to be a "reminiscence," or a recollection, of past events, but there is no indication that the narrator jumps from the past to the present in either verb tense or content.

Explanation for Incorrect Answer C : Choice (C) is incorrect. The narrator suggests that the pursuit of "awe" is fundamentally problematic but does not offer a "solution" of any kind.

Explanation for Incorrect Answer D : Choice (D) is incorrect. The narrator uses a myriad of details, not generalizations, to depict the sensory experience created inside of the planetarium.

Explanation for Incorrect Answer E : Choice (E) is incorrect. The only viewpoint presented in the passage is that of the narrator.

eg

AND EXPLANATIONS

is te

re

knowns and unknowns and horrible immensities, and it seemed to have failed. A good thing, too, I thought. Children 55 have a natural immunity, most of them, and it shouldnt be tampered with. As for the adults who would deplore it, the ones who promoted this show, werent they immune themselves to the extent that they could put in the echo-chamber effects, the music, the solemnity, simulating the awe 60 that they supposed they ought to feel? Awewhat was that supposed to be? A fit of the shivers when you looked out the window? Once you knew what it was, you wouldnt be courting it.

file://E:\\f7.htm

2006-11-12

The Official SAT Online Course

17/21

17

Lines 5-7 (The adults . . . bags) primarily illustrate the childrens feelings of
(A)

helplessness

(B)

restlessness

(C) awe

(D) anticipation

(E)

irritation

ANSWERS

AND EXPLANATIONS

Explanation for Correct Answer B : Choice (B) is correct. "Restless" means antsy or excitable. The image of "adults all around . . . shushing the children" suggests that the children are too restless to remain quiet in the darkened room.

Explanation for Incorrect Answer A : Choice (A) is incorrect. The adults may feel helpless in their efforts to quiet the noisy children, but nothing in these lines indicates that the children themselves are helpless.

Explanation for Incorrect Answer D : Choice (D) is incorrect. While some of the children in the planetarium audience may look forward to the show, their general noisiness indicates inattention more than "anticipation."

Explanation for Incorrect Answer E : Choice (E) is incorrect. Some of the children may act out of mild boredom, but the narrator does not suggest that they are largely irritated, or annoyed.

18

In line 11, progress most nearly means

nR
(A)

evolution

(B)

(C) prosperity

(D) promotion

improvement

(E)

advance

ANSWERS

Explanation for Correct Answer E : Choice (E) is correct. The word "progress" in this context means a ceremonial journey. Line 11 refers to members of British royalty making such a trip to a famous London cathedral.

Explanation for Incorrect Answer A : Choice (A) is incorrect. The term "progress" is sometimes used to refer to "evolution," or gradual development, but it is illogical to describe the "evolution" of a group of people to a place.

Explanation for Incorrect Answer B : Choice (B) is incorrect. "Progress" sometimes indicates a positive change, but it makes little sense to describe the Royal Family's procession to Westminster Abbey as an "improvement."

Explanation for Incorrect Answer C : Choice (C) is incorrect. While it may be said that one who prospers is progressing

eg

AND EXPLANATIONS

is t

er ed

Explanation for Incorrect Answer C : Choice (C) is incorrect. The children's agitation contrasts with the feelings of "awe" that the planetarium show is presumably meant to inspire.

file://E:\\f7.htm

2006-11-12

The Official SAT Online Course

18/21

financially or personally, this use of the term "progress" does not fit in the context of line 11.

Explanation for Incorrect Answer D : Choice (D) is incorrect. The term "promotion" typically refers to a professional progression, which has nothing to do with the Royal Family's ceremonial journey.

19

The first paragraph of the passage establishes a mood of


(A)

jaded dismissal

(B)

nervous apprehension

(C) dramatic anticipation

(D) initial concern

(E)

mundane routine

ANSWERS

AND EXPLANATIONS

Explanation for Incorrect Answer B : Choice (B) is incorrect. The first paragraph is peppered with adjectives such as "comfortable," "splendid," and "eloquent." These terms create a mood that is neither nervous nor apprehensive.

20

nR

(A)

Explanation for Incorrect Answer D : Choice (D) is incorrect. The passage does not suggest that the narrator has any concerns about the upcoming presentation.

Explanation for Incorrect Answer E : Choice (E) is incorrect. It is unlikely that an author would describe a mundane, routine event or feeling in such colorful detail.

The words dodging and shrinking (line 34) primarily suggest that the author was

somewhat bothered by the children in the audience

(B)

initially overwhelmed by the information being presented

(C) unable to admit to some troubling feelings about astronomy

(D) refusing to acknowledge the implications of space travel

(E)

unwilling to believe the studies being discussed

ANSWERS

Explanation for Correct Answer B : Choice (B) is correct. "Stunning facts" illustrating the immensity of the universe "rolled past" the narrator's head in the second paragraph, creating a dizzying web of information. The narrator's mind seems to dodge and shrink away from the deluge of figures and concepts. Only when the facts become less abstract and more familiar is the narrator capable of mechanically recording information.

eg is te

Explanation for Incorrect Answer A : Choice (A) is incorrect. At no point does the narrator appear to be "jaded," or tired, of the planetarium experience.

AND EXPLANATIONS

re

Explanation for Correct Answer C : Choice (C) is correct. The first paragraph attempts to recreate the tension that exists in a darkened planetarium as audience members settle into their chairs and wait for the show to begin. The author's evocative description of the domed screen coming to life as "commanding music" swells under an "eloquent," disembodied voice contributes to the feeling of "dramatic anticipation."

file://E:\\f7.htm

2006-11-12

The Official SAT Online Course

19/21

Explanation for Incorrect Answer A : Choice (A) is incorrect. The reference to the narrator's "dodging and shrinking mind" appears in line 34. It is unlikely that the children who are briefly mentioned in line 6 are responsible for the narrator's mental state almost 30 lines later.

Explanation for Incorrect Answer C : Choice (C) is incorrect. The narrator readily admits that the astronomical facts are "troubling" in their complexity.

Explanation for Incorrect Answer D : Choice (D) is incorrect. No significant mention of "space travel" appears at any point in the passage.

Explanation for Incorrect Answer E : Choice (E) is incorrect. Although the narrator acknowledges in lines 4445 that "confident" scientific information is often revised, nothing in the passage suggests that the narrator's "dodging and shrinking mind" is related to any doubts regarding the show's accuracy.

21

In lines 40-43 (Moonless . . . Sun), the narrators comment about the arrangement demonstrates a preference for
(A)

irony

(B)

inventiveness

(D) ornamentation

(E)

ambiguity

ANSWERS

AND EXPLANATIONS

22

nR

(A)

Explanation for Incorrect Answer A : Choice (A) is incorrect. Lines 4043 do not demonstrate the narrator's sense of "irony."

Explanation for Incorrect Answer B : Choice (B) is incorrect. The narrator's reaction to Mercury's "odd arrangement" may be unusual, but it does not particularly demonstrate "inventiveness," or creativity.

Explanation for Incorrect Answer D : Choice (D) is incorrect. The narrator includes numerous details in the passage but presents them simply. It is somewhat inaccurate, then, to claim that the narrator's style as it appears in lines 4043 is ornamental, or embellished.

Explanation for Incorrect Answer E : Choice (E) is incorrect. The narrator's feelings regarding Mercury's complicated orbit suggest a discomfort with "ambiguity," or confusion, not a preference for it.

In line 53, fix most nearly means

focus

(B)

prepare

(C) repair

(D) decide

influence

eg is

Explanation for Correct Answer C : Choice (C) is correct. The narrator is disappointed to learn from the planetarium show that Mercury's orbit is more complicated than once thought. Apparently the moonless planet rotates three times per every two trips around the sun, "an odd arrangement" compared to the outdated theory that Mercury rotates just once per solar orbit. The narrator's preference for the simpler "arrangement" implies a preference for "symmetry," or balance.

te re

(C) symmetry

file://E:\\f7.htm

2006-11-12

The Official SAT Online Course

20/21

(E)

ANSWERS

AND EXPLANATIONS

Explanation for Correct Answer A : Choice (A) is correct. "Fix" in this context means to direct one's attention. Lines 5155 specifically refer to the adults' effort to catch the children's attention and direct it to the planetarium show.

Explanation for Incorrect Answer B : Choice (B) is incorrect. "Fix" sometimes means to "prepare," but this definition is illogical within the context of line 53.

Explanation for Incorrect Answer C : Choice (C) is incorrect. A common definition of "fix" is to "repair," but it is illogical to claim that adults tried to "repair" children's attention to something.

Explanation for Incorrect Answer D : Choice (D) is incorrect. The term "fix" is occasionally used to reflect a decision, but a child's attention cannot be decided on something else.

23

The phrase horrible immensities (line 54) primarily indicates


(A)

exaggerated information

(B)

unforeseen events

(C) historical monstrosities

(D) controversial debates

(E)

incomprehensible realities

nR
ANSWERS

Explanation for Correct Answer E : Choice (E) is correct. The astronomical "realities" portrayed in the planetarium show suggest a universe so immense that its sheer size is "incomprehensible" and horribly intimidating to the narrator.

Explanation for Incorrect Answer A : Choice (A) is incorrect. Facts that indicate the vastness of the universe may seem "exaggerated," but there is nothing to suggest that the information presented at the planetarium is overstated.

Explanation for Incorrect Answer B : Choice (B) is incorrect. The presentation does not focus on "unforeseen," or unexpected, events.

Explanation for Incorrect Answer C : Choice (C) is incorrect. The presentation does not address a single monstrosity, historical or otherwise.

Explanation for Incorrect Answer D : Choice (D) is incorrect. Nowhere in the passage does the narrator directly refer to "controversial debates."

eg is

AND EXPLANATIONS

te

re

Explanation for Incorrect Answer E : Choice (E) is incorrect. An event or outcome that has been improperly influenced is sometimes described as "fixed," but this sense of the word does not fit the context of line 53.

file://E:\\f7.htm

2006-11-12

The Official SAT Online Course

21/21

24

The author suggests that the echo-chamber effects, the music, the solemnity (lines 59-60) are evidence that
(A)

most adults have feelings of great appreciation for the universe

(B)

most adults would rather not attend planetarium shows

(C) contemporary scientists have an inflated view of the importance of their work

(D) the shows promoters do not fully appreciate the true nature of the universe

(E)

the shows promoters understand that children are entranced by special effects

ANSWERS

AND EXPLANATIONS

Explanation for Correct Answer D : Choice (D) is correct. The narrator criticizes the show's promoters by suggesting that they used special effects such as "echo-chamber effects" to create a mere simulation of "the awe that they.. . ought to feel" at the immensity of the universe.

Explanation for Incorrect Answer B : Choice (B) is incorrect. The narrator does not speculate about the popularity of planetarium shows among adults.

Explanation for Incorrect Answer C : Choice (C) is incorrect. Contemporary scientists likely contributed to the planetarium presentation, but the narrator does not discuss how these scientists feel about their work.

Copyright 2006 The College Board. All rights reserved.

nR

Explanation for Incorrect Answer E : Choice (E) is incorrect. The show's creators may have included special effects to appeal to children, but the narrator does not mention such a theory. The only implication is that such effects illustrate the promoters' immunity to the awesome nature of the universe.

eg

is te

re

Explanation for Incorrect Answer A : Choice (A) is incorrect. The narrator does not imply that the majority of adults appreciate the universe's "horrible immensities," but instead suggests that the adults responsible for infusing the show with special effects are "immune themselves" to the awesome realities of the universe.

Back to Score Report

Privacy Policy

Terms of Use

Contact Us

file://E:\\f7.htm

2006-11-12

The Official SAT Online Course

1/13

Help | Profile | My Organizer | My Bookmarks | Logout

Answers and Explanations

Test Sections

Back to Score Report

Section 1

View Answers and Explanations


Online - Practice Test #6

Section 2

Section 4

Section 5

If notebooks cost $2 each and backpacks cost $32 each, which of the following represents the cost, in dollars, of n notebooks and b backpacks?
(A)
(B)
(C)
(D)
(E)

Section 6

Section 7

Section 8

Section 9

Section 10

ANSWERS

AND EXPLANATIONS

Explanation for Correct Answer D :

te re d
each, then the cost of

Choice (D) is correct. If notebooks cost

each, then the cost of

dollars. If backpacks cost

dollars. So, the cost, in dollars, of

notebooks and

Explanation for Incorrect Answer A : Choice (A) is not correct. See the explanation for the correct response (D).

Explanation for Incorrect Answer B : Choice (B) is not correct. See the explanation for the correct response (D).

nR

Explanation for Incorrect Answer C :

eg is

notebooks is

backpacks is

notebooks is

Choice (C) is not correct. Since

dollars, of

backpack cost

this would be the cost, in

notebooks and

backpacks if each notebook cost

and each

Explanation for Incorrect Answer E :

Choice (E) is not correct. Since

this would be the cost, in

dollars, of

notebooks and

backpacks if each notebook cost

and each

backpack cost

The average (arithmetic mean) of 6, 19, and x is 19. What is the value of x?
(A)

19

(B)

25

(C) 31

(D) 32

(E)

57

ANSWERS

AND EXPLANATIONS

Explanation for Correct Answer D :

Choice (D) is correct. If the average of

and

, then

file://E:\\f8.htm

2006-11-12

The Official SAT Online Course

2/13

So

, and

Explanation for Incorrect Answer A : Choice (A) is not correct. Since the average of

and

is

and

must be greater than

. See the explanation for the correct response (D).

Explanation for Incorrect Answer B : Choice (B) is not correct. See the explanation for the correct response (D).

Explanation for Incorrect Answer C : Choice (C) is not correct. See the explanation for the correct response (D).

Explanation for Incorrect Answer E : Choice (E) is not correct. The sum of

and

is equal to

, but

is not

equal to

. See the explanation for the correct response (D).

Ali, Ben, and Carla made a total of 20 sandwiches. Ben made 3 times as many as Ali, and Carla made twice as many as Ben. How many sandwiches did Ali make?
(A)

Two

(C) Five

(D) Six

eg i

. Solving for

Explanation for Correct Answer A : sandwiches, and Ben made three times as Choice (A) is correct. If Ali made sandwiches. Carla made twice as many many as Ali, then Ben made

sandwiches as Ben, so Carla made

sandwiches they made was

st

gives

ANSWERS

AND EXPLANATIONS

er

(E)

Ten

ed

. This is also equal to

(B)

Four

sandwiches. The total number of

, so

, so Ali made

sandwiches.

U
4

nR

(A)

Explanation for Incorrect Answer B : Choice (B) is not correct. See the explanation for the correct response (A).

Explanation for Incorrect Answer C : Choice (C) is not correct. See the explanation for the correct response (A).

Explanation for Incorrect Answer D : Choice (D) is not correct. See the explanation for the correct response (A). Ben is sandwiches, so times as many sandwiches as Ali, and Ali made made the number of sandwiches Ben made. However, the question asked how many sandwiches Ali made.

Explanation for Incorrect Answer E : Choice (E) is not correct. See the explanation for the correct response (A).

If 0.03 percent of n is 3, what is 3 percent of n?

900

(B)

600

(C) 300

(D) 0.006

(E)

0.003

file://E:\\f8.htm

2006-11-12

The Official SAT Online Course

3/13

ANSWERS

AND EXPLANATIONS

Explanation for Correct Answer C :

Choice (C) is correct. If

percent of

is

, then

. So

, and

. Thus,

percent of

is

Explanation for Incorrect Answer A : Choice (A) is not correct. From the explanation for the correct response (C), you . However, the question asked for percent of . So is have . percent of

Explanation for Incorrect Answer B : Choice (B) is not correct. From the explanation for the correct response (C), you . However, the question asked for percent of is . So is have

percent of

asked for

percent of

nR

(A)

What is an equation of line

(B)

(C)

(D)
(E)

ANSWERS

Explanation for Correct Answer B :

eg i

of

AND EXPLANATIONS
is

st

asked for

percent of

in the figure above?

er

Explanation for Incorrect Answer E : Choice (E) is not correct. From the explanation for the correct response (C), you . However, the question percent of is . So is have

ed

Explanation for Incorrect Answer D : Choice (D) is not correct. From the explanation for the correct response (C), you . However, the question percent of is . So is have

Choice (B) is correct. An equation of line

, where

is the slope

and

is the

-intercept of

. From the figure, you can see that the points

file://E:\\f8.htm

2006-11-12

The Official SAT Online Course

4/13

and

are on

, so the slope is

. Since

intersects the

axis at the point

, the

-intercept of

is

. So

and

, and

has the equation

Explanation for Incorrect Answer A :

Choice (A) is not correct. You can see from the figure that the point

is on line

. But

. So

cannot be an equation

of

Explanation for Incorrect Answer C :

Choice (C) is not correct. You can see from the figure that the point

is on line

. But

Explanation for Incorrect Answer D :

Choice (D) is not correct. You can see from the figure that the point

. But

Explanation for Incorrect Answer E :

re d

is

is on line

Choice (E) is not correct. You can see from the figure that the point

is on line

. But

nR

A
(A)

If the tick marks on the number line above are equally spaced, which of the lettered

eg

(B)

points A through E is between

(C) C

(D) D

(E)

ANSWERS

AND EXPLANATIONS
and

Explanation for Correct Answer B : Choice (B) is correct. If the portion of the number line between into eighths, the result is

is te
.

and

is divided

The only lettered point between

and

file://E:\\f8.htm

2006-11-12

The Official SAT Online Course

5/13

Explanation for Incorrect Answer A : Choice (A) is not correct. If the portion of the number line between divided into eighths, as in the figure above, you can see that point

is and is less than

Explanation for Incorrect Answer C : Choice (C) is not correct. If the portion of the number line between

and

is

divided into eighths, as in the figure above, you can see that point

is greater

than

Explanation for Incorrect Answer D : Choice (D) is not correct. If the portion of the number line between divided into eighths, as in the figure above, you can see that point

is and is greater

than

Explanation for Incorrect Answer E : Choice (E) is not correct. If the portion of the number line between divided into eighths, as in the figure above, you can see that point

is and is greater

than

(A)
(B)

(C)
(D) x

(E)

nR
ANSWERS

Explanation for Correct Answer D :

Choice (D) is correct. If

eg

AND EXPLANATIONS
, then

is t

er

If

what does h equal in terms of x ?

ed

, so

Explanation for Incorrect Answer A : Choice (A) is not correct. See the explanation for the correct response (D).

Explanation for Incorrect Answer B : Choice (B) is not correct. See the explanation for the correct response (D).

Explanation for Incorrect Answer C : Choice (C) is not correct. See the explanation for the correct response (D).

Explanation for Incorrect Answer E : Choice (E) is not correct. See the explanation for the correct response (D).

file://E:\\f8.htm

2006-11-12

The Official SAT Online Course

6/13

In the figure above, what is the value of y?


(A)

40

(B)

45

(C) 50

(D) 60

(E)

72

ANSWERS

AND EXPLANATIONS
, it follows that

Explanation for Correct Answer D : Choice (D) is correct. Since

and

. Vertical angles are equal, so

. Thus,

Explanation for Incorrect Answer A : Choice (A) is not correct. See the explanation for the correct response (D).

Explanation for Incorrect Answer C : Choice (C) is not correct. See the explanation for the correct response (D).

Explanation for Incorrect Answer E : Choice (E) is not correct. See the explanation for the correct response (D).

If
(A)
(B)

nR
(C) 20

10

(D) 30

(E)

870

ANSWERS

Explanation for Correct Answer C :

eg is

, and either

which of the following is a possible value of

AND EXPLANATIONS
, then

te
or

re

. If

Choice (C) is correct. If

Explanation for Incorrect Answer A :

d
. If

Explanation for Incorrect Answer B : Choice (B) is not correct. See the explanation for the correct response (D).

. So

, then

Choice (A) is not correct. If

, then

. So

, and either

or

, then

. If

, then

. So

cannot equal

Explanation for Incorrect Answer B :

Choice (B) is not correct. If

, then

. So

, and either

or

. If

, then

. If

, then

. So

file://E:\\f8.htm

2006-11-12

The Official SAT Online Course

7/13

cannot equal

Explanation for Incorrect Answer D :

Choice (D) is not correct. If

, then

. So

, and either

or

. If

, then

. If

, then

. So

cannot equal

Explanation for Incorrect Answer E :

Choice (E) is not correct. If

, then

. So

, and either

or

. If

, then

. If

, then

. So

cannot equal

10

Mark began a 4-mile bicycle trip by riding slowly uphill for 1 mile. He rested for 10 minutes and then rode quickly downhill for the rest of the trip. Which of the following graphs could correctly represent his trip?

(A)

(B)

nR
(C)

(D)

(E)

ANSWERS

Explanation for Correct Answer B : Choice (B) is correct. This graph could show Marks distance traveled as a function of time. The first portion of the graph indicates that Mark rode at a constant rate of mile. The miles per hour) and covered a distance of minutes ( mile in

eg is

AND EXPLANATIONS

te re

file://E:\\f8.htm

2006-11-12

The Official SAT Online Course

8/13

next portion of the graph is horizontal, which indicates that he rested for minutes and did not cover any distance during that period. The graph indicates that miles in minutes Mark rode at a constant rate of during the final miles per hour, which is faster than he rode in the first minutes, or minutes.

Explanation for Incorrect Answer A : Choice (A) is not correct. This graph could not represent Marks trip. One reason is miles per hour for the first that it indicates that Mark rode at a rate of minutes of his trip. If miles per hour during the last minutes and at a rate of miles per hour cannot miles per hour is described as riding slowly, then be described as riding quickly.

Explanation for Incorrect Answer C : Choice (C) is not correct. This graph could not represent Marks trip. One reason is miles miles, but the trip was that it shows the total distance of the trip to be long.

Explanation for Incorrect Answer D : Choice (D) is not correct. This graph could not represent Marks trip. One reason is -minute period during that it does not have a horizontal section to indicate the which Mark covered no distance.

11

There are 6 red, 6 brown, 6 yellow, and 6 gray scarves packaged in 24 identical, unmarked boxes, 1 scarf per box. What is the least number of boxes that must be selected in order to be sure that among the boxes selected 3 or more contain scarves of the same color?
(A)

(B)

(C) 7

nR
(D) 8

(E)

ANSWERS

Explanation for Correct Answer E : of boxes are selected, there could be red scarves in Choice (E) is correct. If of them, and gray scarves of them, yellow scarves in them, brown scarves in or more scarves of the same color of them; in this case, there would not be in

among these

eg is te r

AND EXPLANATIONS

boxes. However, if one more box is selected, the color of the scarf

in that box must be the same as the color of the scarves in

boxes, and there will then be

of the same color. So,

boxes that must be selected in order to be sure that among the boxes selected or more contain scarves of the same color.

Explanation for Incorrect Answer A : boxes are selected, there could be a scarf of a Choice (A) is not correct. If different color in each box.

Explanation for Incorrect Answer B : boxes are selected, there could be red scarves in Choice (B) is not correct. If of them. of them, and yellow scarves in of them, brown scarves in

Explanation for Incorrect Answer C : boxes are selected, there could be red scarves in Choice (C) is not correct. If of them, and a gray of them, yellow scarves in of them, brown scarves in

ed

Explanation for Incorrect Answer E : Choice (E) is not correct. This could not represent Marks trip. One reason is that it minutes of the trip. This is indicates a negative distance traveled during the last not possible.

of the original

is the least number of

file://E:\\f8.htm

2006-11-12

The Official SAT Online Course

9/13

scarf in

of them.

Explanation for Incorrect Answer D : boxes are selected, there could be red scarves in Choice (D) is not correct. If of them, and gray of them, yellow scarves in of them, brown scarves in of them. scarves in

12

In the figure above, ABCDEF is a regular hexagon, and its center is point O. What is the value of x ?
(A)

80

(B)

60

(C) 40

(D) 30

ANSWERS

AND EXPLANATIONS

nR

From the symmetry of the regular hexagon, it follows that

eg

is

. Drawing some additional lines and labeling some angles yields additional information from the figure:

te
, so

Explanation for Correct Answer B : is the center of the hexagon, it follows that Choice (B) is correct. Since

re d

. Since

(E)

20

. Since

the sum of these angles is

, each of these angles has a measure of

. So

. It is also true that

, and

, it follows that

. Therefore,

, and

Explanation for Incorrect Answer A : Choice (A) is not correct. See the explanation for the correct response (B).

Explanation for Incorrect Answer C : Choice (C) is not correct. See the explanation for the correct response (B).

Explanation for Incorrect Answer D : Choice (D) is not correct. See the explanation for the correct response (B).

Explanation for Incorrect Answer E : Choice (E) is not correct. See the explanation for the correct response (B).

13

Let the function f be defined by

for all numbers x. Which of the following is

equivalent to

file://E:\\f8.htm

2006-11-12

The Official SAT Online Course

10/13

(A)
(B)
(C)
(D)
(E)

25pr

ANSWERS

AND EXPLANATIONS
for all numbers

Explanation for Correct Answer C :

Choice (C) is correct. Since

, you have

Explanation for Incorrect Answer A :

Choice (A) is not correct. Since

for all numbers

, you have

. This expression is not equivalent to

example, if

, then

, but

Explanation for Incorrect Answer B :

Choice (B) is not correct. Since

er

, but

ed
for all numbers

. This expression is not equivalent to

example, if

, then

, but

. For

, you have

. For

Explanation for Incorrect Answer D :

is t
, then

Choice (D) is not correct. Since

eg

For example, if

Explanation for Incorrect Answer E :

nR

Choice (E) is not correct. Since

example, if

14

, then

for all numbers

, you have

. This expression is not equivalent to

for all numbers

, you have

. This expression is not equivalent to

. For

, but

and is divided into 8 congruent regions. What is The circle above has an area of the perimeter of one of these regions?

(A)
(B)

(C)

(D)

file://E:\\f8.htm

2006-11-12

The Official SAT Online Course

11/13

(E)

ANSWERS

AND EXPLANATIONS
congruent regions is

Explanation for Correct Answer C : Choice (C) is correct. The perimeter of each of the

, where

is the radius of the circle and

is the circumference of the

circle. The area of the circle is

, so

. The circumference of the

circle is

, so

. The perimeter of one of the regions, then, is

Explanation for Incorrect Answer A : Choice (A) is not correct. See the explanation for the correct response (C).

Explanation for Incorrect Answer B : Choice (B) is not correct. See the explanation for the correct response (C).

15

and

nR
(A)
(B)

For which of the following values of k will the system of equations above have no solution?

(C) 0

10

(D) 5

(E)

ANSWERS

Explanation for Correct Answer A :

eg

AND EXPLANATIONS
, then the system of equations is

Choice (A) is correct. If

is te

re

can equal both

Explanation for Incorrect Answer E : Choice (E) is not correct. See the explanation for the correct response (C).

d
and

Explanation for Incorrect Answer D : Choice (D) is not correct. See the explanation for the correct response (C).

. The first equation is equivalent to

. There are

no values

and

for which

and

. So the

system of equations has no solution if

Explanation for Incorrect Answer B :

Choice (B) is not correct. If

, then the system of equations is

and

. The solution to this system is

Explanation for Incorrect Answer C :

Choice (C) is not correct. If

, then the system of equations is

file://E:\\f8.htm

2006-11-12

The Official SAT Online Course

12/13

and

. The solution to this system is

and

Explanation for Incorrect Answer D :

Choice (D) is not correct. If

, then the system of equations is

and

. The solution to this system is

and

Explanation for Incorrect Answer E :

Choice (E) is not correct. If

, then the system of equations is

and

. The solution to this system is

and

16

RESULTS OF BEANBAG GAME

II.

For all of the throws attempted, more hit the target than missed the target.

(A)

I only

nR
(B)

II only

(C) I and III only

(D) II and III only

(E)

I, II, and III

ANSWERS

Explanation for Correct Answer E : Choice (E) is correct. To determine the answer to the question, you must decide which of the three statements are true.

eg

III.

No one hit the target 5 times.

AND EXPLANATIONS

is

I.

More than half the people hit the target on their first throw.

te

In a certain game, each person threw a beanbag at a target until the person missed the target. The table shows the results for the 25 people who played the game. For example, 4 people hit the target on their first 3 throws and missed on their 4th throw. Based on the information in the table, which of the following must be true?

re

d
Statement

: From the table, you can see that only

of the

people

, hit , which is more than half of missed the target on their first throw. So is true. the target on their first throw. : The total number of throws attempted was Statement . The total

number of throws that missed the target was

. So

throws hit the target, which is more than the number of throws that is true. missed the target. missed before their fifth throw and people, : Of the Statement

missed their fifth throw. So no one hit the target

times.

is true.

Explanation for Incorrect Answer A : Choice (A) is not correct. From the explanation for the correct answer, is not the only true statement. are all true. So

, and

file://E:\\f8.htm

2006-11-12

The Official SAT Online Course


Explanation for Incorrect Answer B : Choice (B) is not correct. From the explanation for the correct answer, is not the only true statement. are all true. So

13/13

, and

Explanation for Incorrect Answer C : Choice (C) is not correct. From the explanation for the correct answer, are not the only true statements. and are all true. So

, and

Explanation for Incorrect Answer D : Choice (D) is not correct. From the explanation for the correct answer, are not the only true statements. and are all true. So

, and

Back to Score Report

Copyright 2006 The College Board. All rights reserved.

Privacy Policy

Terms of Use

Contact Us

nR

eg is t

er

file://E:\\f8.htm

ed

2006-11-12

The Official SAT Online Course

1/16

Help | Profile | My Organizer | My Bookmarks | Logout

Answers and Explanations

Test Sections

Back to Score Report

Section 1

View Answers and Explanations


Online - Practice Test #6

Section 2

Section 4

Section 5

The new vaccine is ------- preventing certain forms of pneumonia and should, therefore, be more widely ------- in order to prevent outbreaks of the disease.
(A)

Section 6

Section 7

required for . . constrained

Section 8

(B)

unsuccessful in . . distributed

Section 9

(C) instrumental in . . reconstituted

Section 10

(D) effective in . . administered

(E)

unverified for . . disseminated

ANSWERS

AND EXPLANATIONS

nR

Explanation for Incorrect Answer A : Choice (A) is incorrect. "Required" means needed or essential. "Constrained" means to keep within close bounds. If one were to insert these terms into the text, the sentence would read "The new vaccine is required for preventing certain forms of pneumonia and should, therefore, be more widely constrained in order to prevent outbreaks of the disease." It makes no sense to say that a vaccine that is required for preventing illness should be "constrained," or restricted.

Explanation for Incorrect Answer B : Choice (B) is incorrect. "Unsuccessful" means having an unfavorable outcome. "Distributed" means delivered or handed out. If one were to insert these terms into the text, the sentence would read "The new vaccine is unsuccessful in preventing certain forms of pneumonia and should, therefore, be more widely distributed in order to prevent outbreaks of the disease." It is illogical to argue that an unsuccessful vaccine should be distributed or passed out more widely.

Explanation for Incorrect Answer C : Choice (C) is incorrect. "Instrumental" means serving as a means or agency. "Reconstituted" means provided with a new structure. If one were to insert these terms into the text, the sentence would read "The new vaccine is instrumental in preventing certain forms of pneumonia and should, therefore, be more widely reconstituted in order to prevent outbreaks of the disease." An instrumental or helpful vaccine does not need to be reconstituted, or remade, because it is already effective.

Explanation for Incorrect Answer E : Choice (E) is incorrect. "Unverified" means lacking proof or substantiation. "Disseminated" means dispersed throughout. If one were to insert these terms into the text, the sentence would read "The new vaccine is unverified for preventing certain forms of pneumonia and should, therefore, be more widely disseminated in order to prevent outbreaks of the disease." It makes no sense to argue that an unverified or unproven vaccine should be disseminated or dispensed more widely.

eg

is t

Explanation for Correct Answer D : Choice (D) is correct. Something that is "effective" produces a desired effect. "Administered" means dispensed. If one were to insert these terms into the text, the sentence would read "The new vaccine is effective in preventing certain forms of pneumonia and should, therefore, be more widely administered in order to prevent outbreaks of the disease." The word "therefore" indicates that the second part of the sentence should logically occur, based on the claim made in the first part. It makes sense to argue that a successful vaccine should be widely administered.

er

ed

file://E:\\f9.htm

2006-11-12

The Official SAT Online Course

2/16

In an effort to ------- the ------- theater, the troupe members contributed thousands of dollars to keep the playhouse operating.
(A)

qualify . . obsolete

(B)

salvage . . floundering

(C) exacerbate . . defunct

(D) revitalize . . prosperous

(E)

commandeer . . lucrative

ANSWERS

AND EXPLANATIONS

Explanation for Correct Answer B : Choice (B) is correct. "Salvage" means to save from ruin, and "floundering" means struggling. If one were to insert these terms into the text, the sentence would read "In an effort to salvage the floundering theater, the troupe members contributed thousands of dollars to keep the playhouse operating." The fact that the theater required thousands of dollors to maintain operations implies that it was floundering and needed to be salvaged.

nR

(A)

Explanation for Incorrect Answer D : Choice (D) is incorrect. "Revitalize" means to give new life to, and "prosperous" means marked by economic well-being. If one were to insert these terms into the text, the sentence would read "In an effort to revitalize the prosperous theater, the troupe members contributed thousands of dollars to keep the playhouse operating." A "prosperous," or wealthy, theater would not need to be revitalized through financial donations.

Explanation for Incorrect Answer E : Choice (E) is incorrect. "Commandeer" means to take by force, and "lucrative" means profitable. If one were to insert these terms into the text, the sentence would read "In an effort to commandeer the lucrative theater, the troupe members contributed thousands of dollars to keep the playhouse operating." It is unlikely that actors would attempt to forcibly take over a playhouse or that a "lucrative" theater would be in need of financial donations.

In her writings about language, the poet Gloria Anzalda celebrates the ------- of English and Spanish dialects spoken by Mexican Americans, arguing that such ------lends an empowering flexibility to expression.

multiplicity . . variety

(B)

proliferation . . moderation

(C) ambivalence . . focus

(D) dearth . . depletion

(E)

abridgment . . imitation

ANSWERS

Explanation for Correct Answer A :

eg is

Explanation for Incorrect Answer C : Choice (C) is incorrect. "Exacerbate" means to make worse, and something that is "defunct" has ceased to exist. If one were to insert these terms into the text, the sentence would read "In an effort to exacerbate the defunct theater, the troupe members contributed thousands of dollars to keep the playhouse operating." It is illogical to assert that actors would donate money to a struggling theater in an effort to make its problems worse.

AND EXPLANATIONS

te

re d

Explanation for Incorrect Answer A : Choice (A) is incorrect. "Qualify" means to describe or to modify. "Obsolete" means no longer in use. If one were to insert these terms into the text, the sentence would read "In an effort to qualify the obsolete theater, the troupe members contributed thousands of dollars to keep the playhouse operating." The second half of the sentence indicates that the theater is failing, but not yet obsolete. Furthermore, the verb "qualify" does not make sense in this context.

file://E:\\f9.htm

2006-11-12

The Official SAT Online Course

3/16

Choice (A) is correct. "Multiplicity" means a great number. "Variety" means something varying from others of the same general kind. If one were to insert these terms into text, the sentence would read "In her writings about language, the poet Gloria Anzalda celebrates the multiplicity of English and Spanish dialects spoken by Mexican Americans, arguing that such variety lends an empowering flexibility to expression." The word "such" suggests that the two blanked words are synonyms, both describing a celebrated feature of Mexican American speech. "Multiplicity" and "variety" describe Mexican American language in the same way. Multiple and various forms of speech would also give Mexican Americans the "empowering flexibility of expression" Anzalda refers to.

Explanation for Incorrect Answer B : Choice (B) is incorrect. Proliferation means to increase at a rapid rate. Moderation means being within reasonable limits. If one were to insert these terms into the text, the sentence would read In her writings about language, the poet Gloria Anzalda celebrates the proliferation of English and Spanish dialects spoken by Mexican Americans, arguing that such moderation lends an empowering flexibility to expression. A proliferation, or rapid increase, of dialects would not be described as a moderation. It is also unlikely that moderation could give someone flexibility of expression.

Explanation for Incorrect Answer E : Choice (E) is incorrect. "Abridgment" means the act of reducing or shortening. "Imitation" means something derived or copied from an original. If one were to insert these terms into text, the sentence would read "In her writings about language, the poet Gloria Anzalda celebrates the abridgement of English and Spanish dialects spoken by Mexican Americans, arguing that such imitation lends an empowering flexibility to expression."An "abridgment" is a reduction or shortening of something, a concept that has little to do with either "imitation" or "flexibility of expression."

nR

(A)

The mountain road was distinctly -------: it twisted back and forth along the contours of the hillside.

panoramic

(B)

precipitous

(C) serpentine

(D) circumscribed

(E)

retrograde

ANSWERS

Explanation for Correct Answer C : Choice (C) is correct. "Serpentine" means winding. If one were to insert this term into the text, the sentence would read "The mountain road was distinctly serpentine: it twisted back and forth along the contours of the hillside." The colon indicates that the second part of the sentence will illustrate the meaning of the missing term. "Serpentine" fits the sentence because a winding road does indeed twist "back and forth."

eg

Explanation for Incorrect Answer D : Choice (D) is incorrect. "Dearth" means a lack of or scarce supply. "Depletion" means the act of depleting. If one were to insert these terms into text, the sentence would read "In her writings about language, the poet Gloria Anzalda celebrates the dearth of English and Spanish dialects spoken by Mexican Americans, arguing that such depletion lends an empowering flexibility to expression." While "dearth" and "depletion" both refer to a lack of something, it is illogical to state that a lack of ways of speaking would lend "flexibility to expression."

AND EXPLANATIONS

is

te

re

Explanation for Incorrect Answer C : Choice (C) is incorrect. "Ambivalence" means uncertainty or indecisiveness. "Focus" means a center of interest or activity. If one were to insert these terms into text, the sentence would read "In her writings about language, the poet Gloria Anzalda celebrates the ambivalence of English and Spanish dialects spoken by Mexican Americans, arguing that such focus lends an empowering flexibility to expression." "Ambivalence," or conflicting feelings, would tend to take away from a person's focus. Furthermore, the two words are not synonyms.

file://E:\\f9.htm

2006-11-12

The Official SAT Online Course

4/16

Explanation for Incorrect Answer A : Choice (A) is incorrect. "Panoramic" refers to a complete view of a surrounding area. If one were to insert this term into the text, the sentence would read "The mountain road was distinctly panoramic: it twisted back and forth along the contours of the hillside." A winding road will not necessarily offer "panoramic" views.

Explanation for Incorrect Answer B : Choice (B) is incorrect. "Precipitous" means extremely steep. If one were to insert this term into the text, the sentence would read "The mountain road was distinctly precipitous: it twisted back and forth along the contours of the hillside." There is no indication that the winding road in the sentence is particularly steep.

Explanation for Incorrect Answer D : Choice (D) is incorrect. "Circumscribed" refers to something that is constricted or surrounded by a boundary. If one were to insert this term into the text, the sentence would read "The mountain road was distinctly circumscribed: it twisted back and forth along the contours of the hillside." The statement after the colon does not indicate that the road is "circumscribed" in any way.

Explanation for Incorrect Answer E : Choice (E) is incorrect. "Retrograde" means moving backward. If one were to insert this term into the text, the sentence would read "The mountain road was distinctly retrograde: it twisted back and forth along the contours of the hillside." It does not make sense to describe the road as moving in a backward direction.

At the family reunion Hiroko found her cousin charming and gentle, the ------- of his formerly rude and overbearing self.
(A)

remnant

(B)

antithesis

(C) consequence

(E)

mainstay

nR
ANSWERS

Explanation for Correct Answer B : Choice (B) is correct. "Antithesis" means exact opposite. If one were to insert this term into the text, the sentence would read "At the family reunion Hiroko found her cousin charming and gentle, the antithesis of his formerly rude and overbearing self." The sentence indicates that Hiroko's cousin has changed dramatically. He was once "rude and overbearing," and is now "charming and gentle." "Antithesis" connotes this dramatic change perfectly.

Explanation for Incorrect Answer A : Choice (A) is incorrect. "Remnant" means a remainder. If one were to insert this term into the text, the sentence would read "At the family reunion Hiroko found her cousin charming and gentle, the remainder of his formerly rude and overbearing self." It makes no sense to say that "charming and gentle" qualitites would be the "remnant," or remainder, of a "formerly rude and overbearing self."

Explanation for Incorrect Answer C : Choice (C) is incorrect. "Consequence" means the relation of a result to its cause. If one were to insert this term into the text, the sentence would read "At the family reunion Hiroko found her cousin "charming and gentle," the consequence of his formerly rude and overbearing self." It is illogical to claim that charming and gentle behavior could be a consequence of "rude and overbearing" behavior. This sentence suggests that Hiroko's cousin's behavior is different from what it once was, not that it is the result of what it once was.

Explanation for Incorrect Answer D : Choice (D) is incorrect. "Extremity" means the farthest point. If one were to insert this term into the text, the sentence would read "At the family reunion Hiroko found

eg

(D) extremity

AND EXPLANATIONS

is te re

d
file://E:\\f9.htm

2006-11-12

The Official SAT Online Course

5/16

her cousin charming and gentle, the extremity of his formerly rude and overbearing self." Hiroko's cousin's new behavior is not an extreme expression of her cousin's former self, it is completely different.

Explanation for Incorrect Answer E : Choice (E) is incorrect. "Mainstay" means a chief support or foundational influence or characteristic. If one were to insert this term into the text, the sentence would read "At the family reunion Hiroko found her cousin charming and gentle, the mainstay of his formerly rude and overbearing self." Hiroko's cousin was exhibiting characteristics that seemed new, not foundational, to how his cousin previously knew him.

His conduct at the state dinner was a cavalcade of blunders, one ------- following another until the evening ended.
(A)

query

(B)

gibe

(C) gaffe

(D) tryst

(E)

tribute

ANSWERS

AND EXPLANATIONS

Explanation for Correct Answer C : Choice (C) is correct. "Gaffe" means a blunder. If one were to insert this term into the text, the sentence would read "His conduct at the state dinner was a cavalcade of blunders, one gaffe following another until the evening ended." The comma indicates that the missing term is synonymous with blunder. Both "blunder" and "gaffe" can be used to refer to a social error.

Both passages discuss the issue of the intelligence of dogs. Passage 1 was adapted from a 2001 book on animal intelligence. Passage 2 was written in 2001 by a dog trainer and writer.

Passage 1

nR

Explanation for Incorrect Answer B : Choice (B) is incorrect. "Gibe" means a derisive remark. If one were to insert this term into the text, the sentence would read "His conduct at the state dinner was a cavalcade of blunders, one gibe following another until the evening ended." The use of gibes at a formal dinner could certainly be considered to be a social faux pas, but a blunder does not always appear in the form of a taunt.

Explanation for Incorrect Answer D : Choice (D) is incorrect. "Tryst" typically refers to a romantic meeting. If one were to insert this term into the text, the sentence would read "His conduct at the state dinner was a cavalcade of blunders, one tryst following another until the evening ended." The term "tryst" is not a synonym of "blunder."

Explanation for Incorrect Answer E : Choice (E) is incorrect. A "tribute" is an acknowledgment of gratitude, respect, or admiration. If one were to insert this term into the text, the sentence would read "His conduct at the state dinner was a cavalcade of blunders, one tribute following another until the evening ended." It's not likely that a tribute at a state dinner would be described as a social error.

It was no accident that nineteenth-century


file://E:\\f9.htm

eg

Explanation for Incorrect Answer A : Choice (A) is incorrect. A "query" is a question or inquiry. If one were to insert this term into the text, the sentence would read "His conduct at the state dinner was a cavalcade of blunders, one query following another until the evening ended." It is inaccurate to equate a blunder, or mistake, with a query.

is t

er ed

2006-11-12

The Official SAT Online Course

6/16

naturalist Charles Darwin strove to connect the mentality and emotionality of people with that of dogs, rather than, say, doves or horses. Neither his theory of evolution Line nor any general understanding of biology demanded 5 that he preferentially underline our similarity to dogs over other species. But politically and emotionally, the choice was inevitable for an English gentleman who had set himself the task of making the idea of evolutionary continuity palatable. Darwin wrote that dogs possess 10 something very similar to a conscience. They certainly possess some power of self-command. . . . Dogs have long been accepted as the very type of fidelity and obedience. Darwin was not alone in his beliefs that dogs possess human virtues. The characteristics of loyalty 15 and obedience, coupled with an expressive face and body, can account for why dogs are such popular and valued pets in many cultures. Depending on the breed and the individual, dogs can be noble, charming, affectionate, and reliable. But while all dog owners should rightly appreciate 20 these and other endearing traits in their pets, nothing says that the cleverness of a highly intelligent primate such as a chimpanzee is part of the package. Scientists generally believe the reasoning abilities of chimps to be considerably greater than that of dogs. But many people nonetheless 25 believe that dogs are smarter than chimps precisely because of our

nR eg

familiarity and emotional ties with the dogs that


file://E:\\f9.htm

is t

er

ed

2006-11-12

The Official SAT Online Course

7/16

Every dog trainer that I know had the same childhood, a childhood filled with the brilliant, heroic dogs of 40 literature. We read about dogs who regularly traveled thousands of miles to be reunited with owners who somehow misplaced them, repeatedly saved people from certain death, and continually exhibited a better grasp of strategic problemsolving than the average Ph.D. In the preface 45 to one of his many dog stories, S. P. Meek a bit shamefacedly remarked that in writing of dogs I endeavored to hold these heroes down to the level of canine intelligence, and to make them, above all, believable. If at times I seem to have made them show supercanine intelligence, it is because my 50 enthusiasm has run away with me. We forgave him, of course. It was something of a shock, therefore, to discover

nR

eg i

st e

Passage 2

re

we love. We apply the same secret rules to our fellow humans: the old in-group, out-group story. People in your ingroup are those who are similar to you, either because 30 they belong to the same organizations as you, or enjoy the same activities, or, and this is the kicker, because they are simply around more often. Dogs, because of their proximity to their owners, are definitely in. The intensity of our relationship with dogs causes us, quite 35 naturally, to imbue them with high-level mental abilities, whether they have earned those extra intelligence points or not. We like them, so we think well of them.

file://E:\\f9.htm

2006-11-12

The Official SAT Online Course

8/16

55

60

65

70

75

how the learning theory experts believed dogs think and learn. I was told that dogs, unlike chimpanzees, have no real reasoning ability. Dogs dont think: rather, they learn to avoid the unpleasant (negative reinforcement), seek the pleasant (positive reinforcement), or some combination of the two. To contend otherwise was to be guilty of the sin of anthropomorphizing, the attribution to an animal of motivations and consciousness that only a human being could possess. Yet as a dog trainer, I find myself siding more with the Meeks than I do with the learning theorists: nobody could believe dispassionately in the totality of positive and negative reinforcement after seeing the pure intelligence shining in the face of a border collie intent upon helping a shepherd herd sheep. Dogs do think and reason. Granted, a dog might not be able to run a maze as quickly as a chimp. But a dog outshines any other animal that I know in the ability to work willingly with a human being, to communicate with a puzzling creature who often makes incomprehensible demands. Researchers have increasingly come to view intelligence as a complex collection of mental abilities that cannot be fully captured in any simple way. Dogs are geniuses at being useful, and it is this usefulness that we admire when we praise their intelligence. As Jonica Newby, a specialist in animal-human interaction, writes, In some ways intelligence is a matter of match-

nR

ing behavior to environment. To compare


file://E:\\f9.htm

eg is

te

re

2006-11-12

The Official SAT Online Course

9/16

intelligence in creatures that have evolved differently is a 80 bit like deciding which has hit upon the best mode of travel: the dolphin or the horse. And it is dogs, not chimps, who possess the most helpful mode of travel for human beings.
7

Unlike the author of Passage 1, the author of Passage 2 develops an argument by relying on
(A)

personal experience

(B)

scientific observation

(C) historical contextualization

(D) statistical evidence

(E)

direct quotation

ANSWERS

AND EXPLANATIONS

nR

(A)

Explanation for Incorrect Answer C : Choice (C) is incorrect. Unlike the author of Passage 1, the author of Passage 2 provides little "historical" background information.

Explanation for Incorrect Answer D : Choice (D) is incorrect. The author of Passage 2 does not provide "statistical evidence."

Explanation for Incorrect Answer E : Choice (E) is incorrect. Both authors use direct quotations to develop their arguments.

The phrase It was no accident (line 1) implies that the author of Passage 1 believes that Darwin

knew that the resemblance between dogs and humans could not be accounted for by his theory

(B)

exploited the sympathies of his audience to gain support for his theory

(C) considered intelligence to be largely a matter of luck

(D) believed that the way previous authors had written about dogs was inaccurate

(E)

wished to convince the public to celebrate the virtues of dogs

ANSWERS

Explanation for Correct Answer B : Choice (B) is correct. The author suggests that Darwin's example of animal intelligence was intentional. His readers' tendency to hold dogs in high regard made them more likely to accept his theory of evolutionary continuity.

eg

Explanation for Incorrect Answer B : Choice (B) is incorrect. Passage 1 relies heavily on "scientific" evidence. Passage 2 does not.

AND EXPLANATIONS

is te re

Explanation for Correct Answer A : Choice (A) is correct. Passage 1 examines the issue of dog intelligence from a primarily objective perspective, while Passage 2 is infused with the author's firsthand experiences.

file://E:\\f9.htm

2006-11-12

The Official SAT Online Course

10/16

Explanation for Incorrect Answer A : Choice (A) is incorrect. Darwin's theory does in fact account for the similarities between dogs and humans.

Explanation for Incorrect Answer C : Choice (C) is incorrect. The phrase "[i]t was no accident" has no connection to the question of whether or not intelligence is a matter of luck.

Explanation for Incorrect Answer D : Choice (D) is incorrect. The author does not mention Darwin's opinions regarding previous depictions of dogs.

Explanation for Incorrect Answer E : Choice (E) is incorrect. Darwin's focus is on supporting his argument, not on celebrating animals.

In line 13, type most nearly means


(A)

category

(B)

model

(C) designation

(D) role

10

nR

(A)

Explanation for Incorrect Answer A : Choice (A) is incorrect. Dogs may embody "fidelity and obedience," but it is illogical to claim that they are a "category," or division, of specific behavorial traits.

Explanation for Incorrect Answer C : Choice (C) is incorrect. While Darwin implies that dogs exemplify certain traits, it would be illogical to claim that they are a "designation," or name, of "fidelity and obedience."

Explanation for Incorrect Answer D : Choice (D) is incorrect. Dogs may have played a "role" in Darwin's theories regarding animal intelligence, but it would make little sense to say that they are the "very role" of certain behavioral traits.

Explanation for Incorrect Answer E : Choice (E) is incorrect. The term "figure" is not typically used in this context.

The italics in line 25 most directly emphasize

a misguided idea that is becoming obsolete

(B)

a negative view that scientists adopt toward lay people

(C) a common criticism of dog owners

(D) the controversial nature of an alternative theory

(E)

the intensity of a conviction based on sentiment

ANSWERS

eg

AND EXPLANATIONS

is

Explanation for Correct Answer B : Choice (B) is correct. Darwin describes dogs as "the very type of fidelity and obedience," implying that they are a "model," or ideal example, of these qualities.

te

ANSWERS

AND EXPLANATIONS

re

(E)

figure

file://E:\\f9.htm

2006-11-12

The Official SAT Online Course

11/16

Explanation for Correct Answer E : Choice (E) is correct. Italics draw attention to the word "believe," emphasizing people's willingness to credit the dogs they love with a depth of intelligence that they may not possess.

Explanation for Incorrect Answer A : Choice (A) is incorrect. Passage 1 suggests that the belief expressed in line 25 is not scientifically valid, or misguided. The italics in this sentence do not, however, indicate that public opinion on canine intelligence is changing or going away.

Explanation for Incorrect Answer B : Choice (B) is incorrect. The author does not discuss the scientists' attitudes regarding the general public at any point in the passage.

Explanation for Incorrect Answer C : Choice (C) is incorrect. The author alludes to dog owners' opinions but does not criticize them by placing the word "believe" in italics.

Explanation for Incorrect Answer D : Choice (D) is incorrect. The use of italics does not indicate whether or not dog owners' beliefs are controversial.

11

In line 29, the author of Passage 1 uses the word old to suggest that the story is
(A)

familiar

(B)

historic

(C) fictitious

(D) tiresome

(E)

outdated

ANSWERS

12

nR

(A)

Explanation for Correct Answer A : Choice (A) is correct. The author uses the word "old" in the phrase "the old ingroup, out-group story" to suggest that this explanation is a familiar one.

Explanation for Incorrect Answer B : Choice (B) is incorrect. The word "old" can certainly mean "historic" in certain contexts, but the author of passage 1 does not indicate that this "story" has historical significance of any kind.

Explanation for Incorrect Answer C : Choice (C) is incorrect. This story is hardly "fictitious," or false; on the contrary, the author suggests that it accurately describes the bond between humans and dogs.

Explanation for Incorrect Answer D : Choice (D) is incorrect. The author does not suggest in any way that the "story" referred to is "tiresome."

Explanation for Incorrect Answer E : Choice (E) is incorrect. Something that is "outdated" is no longer current, but the author suggests that "the old in-group, out-group story" is just as useful in the present as it was in the past.

Darwin (lines 1-13, Passage 1) and Meek (lines 45-51, Passage 2) serve as examples of

writers who had the courage to voice unpopular viewpoints

(B)

researchers who conducted important studies on animal behavior

eg

AND EXPLANATIONS

is

te

re

file://E:\\f9.htm

2006-11-12

The Official SAT Online Course

12/16

(C) people who maliciously publicized misleading information about dogs

(D) individuals whose writings reflect an idealized view of dog behavior

(E)

scientists who were authorities on the issue of animal intelligence

ANSWERS

AND EXPLANATIONS

Explanation for Correct Answer D : Choice (D) is correct. Darwin's descriptions of dogs' supreme "fidelity and obedience" and Meek's tendency to bestow a "'supercanine intelligence'" on the dogs in his stories support the claim that these writers idealize dog behavior.

Explanation for Incorrect Answer A : Choice (A) is incorrect. Darwin's efforts to make "the idea of evolutionary continuity palatable" indicate that some of his theories may have been "unpopular." There is no indication, however, that Meek's viewpoints were not well received.

Explanation for Incorrect Answer B : Choice (B) is incorrect. Although Darwin was a researcher, Passage 2 only indicates that Meek was a writer of "many dog stories."

13

In line 53, the author of Passage 2 uses quotation marks to


(A)

express anger about a prevailing belief

nR
(E)

(B)

demonstrate respect for a certain group of scientists

(C) indicate uncertainty about the precise usage of a word

(D) cite a term used in an unusual context

cast doubt on the aptness of a description

ANSWERS

Explanation for Correct Answer E : Choice (E) is correct. Writers sometimes place quotation marks around a word or phrase to cast doubt on its accuracy. The learning theory experts' "shocking" assessment of canine intelligence prompts the author of passage 2 to diminish the credibility of the learning theorists by placing the word "experts" in quotation marks.

Explanation for Incorrect Answer A : Choice (A) is incorrect. It can be argued that the author's use of quotation marks expresses disbelief or even annoyance, but there is no conveyance of an extreme emotion such as anger.

Explanation for Incorrect Answer B : Choice (B) is incorrect. The use of quotation marks around the word "experts" is a sign of skepticism, not "respect."

Explanation for Incorrect Answer C : Choice (C) is incorrect. The author may view the experts' theories with uncertainty, but there is no reason to question the straightforward use of the word "experts" in this context.

eg

AND EXPLANATIONS

is te

Explanation for Incorrect Answer E : Choice (E) is incorrect. Darwin might qualify as an authority on animal intelligence, but Meek seems to be a fiction writer.

re

Explanation for Incorrect Answer C : Choice (C) is incorrect. There is no indication that either Darwin or Meek deliberately misled the public about canine intelligence, and nothing in these passages suggests that the men were malicious.

file://E:\\f9.htm

2006-11-12

The Official SAT Online Course

13/16

Explanation for Incorrect Answer D : Choice (D) is incorrect. It is not surprising to encounter the term "experts" within a discussion of a particular scientific theory.

14

The experts (line 53) would most likely argue that which of the following is guilty of the sin mentioned in line 58 ?
(A)

A veterinarian who is unwilling to treat a sick animal

(B)

A cat owner who believes his cat misses its siblings

(C) A dog owner who is unwilling to punish her dog for misbehaving

(D) A zoologist who places the interests of people before those of animals

(E)

A horse trainer who fails to recognize that his horse is hungry

ANSWERS

AND EXPLANATIONS

Explanation for Correct Answer B : Choice (B) is correct. This is a prime example of "anthropomorphizing," or attributing human feelings to animals.

Explanation for Incorrect Answer C : Choice (C) is incorrect. There is no indication that the dog owner is unwilling to punish her dog because she believes her dog has human feelings.

15

nR

(A)

Explanation for Incorrect Answer E : Choice (E) is incorrect. A horse trainer's inability to recognize that his horse is hungry has nothing to do with attributing human feelings to animals.

Both the author of Passage 1 and the experts mentioned in line 53 of Passage 2 directly support the idea that

writers of dog stories intentionally distort the truth for dramatic purposes

(B)

comparing the intelligence of dogs to that of chimps is a pointless enterprise

(C) many people have an excessive emotional attachment to their dogs

(D) dogs are less intelligent than many people believe

(E)

few people are familiar with learning theory as it applies to dogs

ANSWERS

Explanation for Correct Answer D : Choice (D) is correct. The author of Passage 1 asserts that many people like to believe that dogs have "high-level mental abilities, whether they have earned those extra intelligence points or not." The "experts" mentioned in Passage 2 contribute to the argument that people exaggerate canine intelligence by claiming that dogs "have no real reasoning ability."

Explanation for Incorrect Answer A : Choice (A) is incorrect. Passage 2 implies that Meek tried to keep his canine characters "believable," though he may not have always succeeded, and the author

eg

Explanation for Incorrect Answer D : Choice (D) is incorrect. "A zoologist who places the interests of people before those of animals" would not be guilty of the sin mentioned in line 58 ("anthropomorphizing").

AND EXPLANATIONS

is

te re

Explanation for Incorrect Answer A : Choice (A) is incorrect. The veterinarian's motivation for refusing to treat the animal would need to be specified to suggest anthropomorphization.

file://E:\\f9.htm

2006-11-12

The Official SAT Online Course

14/16

of Passage 1 does not mention fiction writers at all.

Explanation for Incorrect Answer B : Choice (B) is incorrect. Both the author of Passage 1 and the "experts" cited in Passage 2 seem to find the comparison between "the intelligence of dogs" and "that of chimps" to be worthwhile.

Explanation for Incorrect Answer C : Choice (C) is incorrect. Unlike the author of Passage 1, the "experts" do not mention the "emotional attachment" of people to dogs.

Explanation for Incorrect Answer E : Choice (E) is incorrect. Although the "experts" cited in Passage 2 specialize in "learning theory," there is no mention of how familiar people are with their field of study.

16

Based on lines 63-67 (nobody . . . sheep), the author of Passage 2 would most likely appear to the author of Passage 1 as
(A)

a neutral observer of animal behavior

(B)

well informed concerning research into animal intelligence

(C) having a deep fondness for border collies and therefore overestimating them

(D) having little respect for traditional scientific research

(E)

having a narrow understanding of what constitutes intelligence

ANSWERS

AND EXPLANATIONS

17

nR

(A)

Explanation for Incorrect Answer A : Choice (A) is incorrect. The author of Passage 1 would most likely consider the author of Passage 2 to be emotionally attached to dogs and therefore incapable of assessing their intelligence objectively.

Explanation for Incorrect Answer B : Choice (B) is incorrect. Lines 6367 reflect a subjective perspective on canine intelligence. They do not suggest that the author of Passage 2 is particularly knowledgeable about research on the topic.

Explanation for Incorrect Answer D : Choice (D) is incorrect. The author of Passage 2 seems to eschew the results of scientific studies in favor of personal experience, but lines 6367 do not reflect a disrespect for "traditional" research.

Explanation for Incorrect Answer E : Choice (E) is incorrect. The two writers may have differing views regarding the extent to which dogs are capable of reasoning, but it would be extreme to claim that the author of Passage 2 has a limited comprehension "of what constitutes intelligence."

In Passage 2, lines 67-68 (Granted . . . chimp) principally serve to

acknowledge a flaw in a prevalent theory

(B)

digress from a primary claim

(C) evoke an air of mystery

(D) dismiss a scientific hypothesis as unfounded

eg

is

Explanation for Correct Answer C : Choice (C) is correct. The author of Passage 1 concedes that "dogs can be noble, charming, affectionate, and reliable" but suggests that they may not have "earned those extra intelligence points." In comparison, the description of "pure intelligence shining in the face of a border collie" in lines 6367 may be interpreted to be an overestimation based on the writer's fondness for dogs.

te

re d

file://E:\\f9.htm

2006-11-12

The Official SAT Online Course

15/16

(E)

anticipate a potential objection to an argument

ANSWERS

AND EXPLANATIONS

Explanation for Correct Answer E : Choice (E) is correct. In admitting that chimps are superior to canines in some traditional measures of intelligence, the author anticipates a possible objection to the argument that dogs possess an unmatched ability to work and communicate with people.

Explanation for Incorrect Answer A : Choice (A) is incorrect. A chimp's ability to navigate a maze faster than a dog can does not contradict any theories mentioned in the passage.

Explanation for Incorrect Answer B : Choice (B) is incorrect. A "digression" is a departure from the main argument in a text. Lines 6768, however, are very much related to the author's claim that dogs possess a keen intelligence.

Explanation for Incorrect Answer C : Choice (C) is incorrect. On the contrary, the author's use of concession in Passage 2 is meant to clarify the argument made in favor of canine intelligence.

18

The authors of both passages mention chimpanzees in order to


(A)

suggest that the public has a distorted view of chimpanzee intelligence

(C) justify the beliefs of the public regarding the intelligence of certain animals

(D) criticize an eccentric scientific claim about animal intelligence

nR
ANSWERS

(E)

provide an example of an animal considered to be highly intelligent

Explanation for Correct Answer E : Choice (E) is correct. The author of Passage 1 suggests that chimpanzees are more intelligent than dogs, while the author of Passage 2 argues that the two animals exhibit different types of advanced intelligence.

Explanation for Incorrect Answer A : Choice (A) is incorrect. The author of Passage 2 does not discuss how "the public" views chimpanzee intelligence.

Explanation for Incorrect Answer B : Choice (B) is incorrect. Both authors refer to chimpanzees' reasoning abilities, but neither discusses the emotions of primates.

Explanation for Incorrect Answer C : Choice (C) is incorrect. The author of Passage 1 asserts that people have a generally flawed understanding of animal intelligence, and the author of Passage 2 doesn't discuss public opinion at all.

Explanation for Incorrect Answer D : Choice (D) is incorrect. Neither author mentions chimpanzees in reference to "an eccentric scientific claim."

eg

(B)

compare the emotions of primates to those of dogs

AND EXPLANATIONS

is te

re

Explanation for Incorrect Answer D : Choice (D) is incorrect. Lines 6768 make no attempt to discredit a scientific theory. Instead, they attempt to place the results of a specific scientific finding in context.

file://E:\\f9.htm

2006-11-12

The Official SAT Online Course

16/16

19

How do the authors of the two passages differ in their assumptions about animal intelligence?
(A)

The author of Passage 1 implies that intelligence is a single entity, whereas the author of Passage 2 suggests that intelligence can be demonstrated in many distinct ways.

(B)

The author of Passage 1 believes that no animal can be considered truly intelligent, whereas the author of Passage 2 celebrates the reasoning power of all animals.

The author of Passage 1 believes that intelligence can be measured, (C) whereas the author of Passage 2 believes that such quantification would be unethical.
(D) The author of Passage 1 suggests that intelligence is innate, whereas the author of Passage 2 argues that it is acquired.

(E)

The author of Passage 1 considers intelligence to be developed over time, whereas the author of Passage 2 shows that it is largely static.

ANSWERS

AND EXPLANATIONS

Explanation for Correct Answer A : Choice (A) is correct. The author of Passage 1 seems to recognize a single form of intelligence by which all animals may be evaluated. The author of Passage 2, however, argues that dogs exhibit an intelligence that is different from that of chimpanzees, but no less useful.

Explanation for Incorrect Answer D : Choice (D) is incorrect. Neither author directly discusses assumptions about whether intelligence is "innate" or "acquired."

nR

Explanation for Incorrect Answer E : Choice (E) is incorrect. Neither author argues that intelligence is a static or dynamic trait in animals.

eg

is

Explanation for Incorrect Answer C : Choice (C) is incorrect. At no point does the author of Passage 2 delve into the ethics of measuring animal intelligence.

te

Explanation for Incorrect Answer B : Choice (B) is incorrect. Both authors clearly believe that animals can possess intelligence. The author of Passage 1 alludes to the "cleverness of a highly intelligent primate."

re

d
Copyright 2006 The College Board. All rights reserved.

Back to Score Report

Privacy Policy

Terms of Use

Contact Us

file://E:\\f9.htm

2006-11-12

The Official SAT Online Course

1/8

Help | Profile | My Organizer | My Bookmarks | Logout

Answers and Explanations

Test Sections

Back to Score Report

Section 1

View Answers and Explanations


Online - Practice Test #6

Section 2

Section 4

Section 5

The sales assistant arranged the gems on the counter, he proceeded to tell us about the origins of each stone.
(A)

Section 6

Section 7

The sales assistant arranged the gems on the counter, he

Section 8

(B)

The gems, which were arranged on the counter by the sales assistant, who

Section 9

(C) The gems were first arranged on the counter by the sales assistant, then

Section 10

(D) After arranging the gems on the counter, the sales assistant

(E)

The sales assistant, having arranged the gems on the counter, he

ANSWERS

AND EXPLANATIONS

Explanation for Incorrect Answer A : Choice (A) involves improper coordination. Two complete thoughts ("The sales...counter and "he...stone") are joined by only a comma.

Explanation for Incorrect Answer B : Choice (B) creates a fragment. There is no main verb in the sentence.

nR

(A)

Explanation for Incorrect Answer C : Choice (C) creates an illogical sentence. It is illogical that "gems" "proceeded to tell us" something.

Explanation for Incorrect Answer E : Choice (E) involves an error in pronoun use. The pronoun "he" is unnecessary.

A whistle-blower is when an employee reports fraud or mismanagement in a company.

when an employee reports fraud or mismanagement

(B)

an employee who reports fraud or mismanagement

(C) reporting by an employee of fraud or mismanagement

(D) if an employee reports fraud or mismanagement

(E)

fraud or mismanagement being reported by an employee

ANSWERS

Explanation for Correct Answer B : Choice (B) is correct. It avoids the error of the original by correctly referring to a whistle-blower as "an employee."

Explanation for Incorrect Answer A : Choice (A) creates an illogical sentence. A whistle-blower is a what, not a when.

eg

AND EXPLANATIONS

is te re

Explanation for Correct Answer D : Choice (D) is correct. It avoids the error of the original by reducing the first complete thought to a phrase.

file://E:\\f10.htm

2006-11-12

The Official SAT Online Course

2/8

Explanation for Incorrect Answer C : Choice (C) creates an illogical sentence. A whistle-blower is a person, not "reporting."

Explanation for Incorrect Answer D : Choice (D) creates an illogical sentence. A whistle-blower is a what, not an "if."

Explanation for Incorrect Answer E : Choice (E) creates an illogical sentence. It calls a whistle-blower "fraud or mismanagement."

After Eliza, the heroine of Shaws Pygmalion, is transformed from a flower girl into a gentlewoman, she realizes that ones social class matters less than your character.
(A)

she realizes that ones social class matters less than your

(B)

she realizes that ones social class matters less than ones

(C) then realizing that ones social class matters less than their

(D) having realized how social class matters less than

(E)

there is her realization about how social class matters less than

Explanation for Incorrect Answer C : Choice (C) creates a fragment. There is no main verb in the sentence.

nR

(A)

Explanation for Incorrect Answer D : Choice (D) creates a fragment. There is no main verb in the sentence.

Explanation for Incorrect Answer E : Choice (E) involves an error in pronoun reference. "There" does not stand for anything in the sentence.

Knowing the roots of words that are hard to spell helps students to become a better speller.

helps students to become a better speller

(B)

is helpful to students who want to be a better speller

(C) helps students to become better spellers

(D) is helpful to students in becoming a better speller

(E)

helps a student be better spellers

ANSWERS

Explanation for Correct Answer C : Choice (C) is correct. It avoids the error of the original by using plural nouns consistently ("students," "spellers").

Explanation for Incorrect Answer A :

eg

AND EXPLANATIONS

is

Explanation for Incorrect Answer A : Choice (A) involves a pronoun shift. The pronoun changes from "one" to secondperson "your" for no reason.

te

re

Explanation for Correct Answer B : Choice (B) is correct. It avoids the error of the original by using the pronoun "one" consistently.

ANSWERS

AND EXPLANATIONS

file://E:\\f10.htm

2006-11-12

The Official SAT Online Course

3/8

Choice (A) involves an error in agreement. The singular noun "a better speller" cannot correctly refer to the plural noun "students."

Explanation for Incorrect Answer B : Choice (B) involves an error agreement. The singular noun "a better speller" cannot correctly refer to the plural noun "students."

Explanation for Incorrect Answer D : Choice (D) involves an error in agreement. "Students" cannot logically become "a better speller."

Explanation for Incorrect Answer E : Choice (E) involves an error in agreement. The plural noun "better spellers" cannot correctly refer to the singular noun "a student."

Most experts believe that young childrens not being given physical affection, this interferes with their normal development.
(A)

young childrens not being given physical affection, this interferes

(B)

for young children who have had physical affection withheld from them, it interferes

(C) the failure at giving young children physical affection would interfere

(D) when withholding physical affection from young children, it interferes

ANSWERS

AND EXPLANATIONS

Explanation for Incorrect Answer A : Choice (A) involves incorrect punctuation. The comma unnecessarily separates the subject ("young...affection") from its verb ("interferes").

nR

(A)

Explanation for Incorrect Answer B : Choice (B) involves unclear pronoun reference. What the pronoun "it" refers to is not clear.

Explanation for Incorrect Answer C : Choice (C) involves improper use of a definite article. The phrase "the failure" suggests that the failure has been referred to previously, but it has not.

Explanation for Incorrect Answer D : Choice (D) involves improper phrasing. The verb "withholding" has no subject.

Electronic bulletin boards, combining the convenience of a telephone with the massive information storage capacity of a computer, present messages on diverse subjects as astronomy, artificial intelligence, and skydiving.

diverse subjects as

(B)

diverse subjects that are

(C) subjects of such diversity as

(D) subjects as diverse as

(E)

a subject as diverse as

ANSWERS

Explanation for Correct Answer D : Choice (D) is correct. It avoids the error of the original by using the idiomatic "as . . . as" construction to indicate the variety of subjects addressed on electronic bulletin

eg

AND EXPLANATIONS

is

te

Explanation for Correct Answer E : Choice (E) is correct. It avoids the error of the original by not separating the subject ("the withholding...children") from its verb ("interferes").

re

(E)

the withholding of physical affection from young children interferes

file://E:\\f10.htm

2006-11-12

The Official SAT Online Course

4/8

boards.

Explanation for Incorrect Answer A : Choice (A) involves improper phrasing. "As" requires "such" before it.

Explanation for Incorrect Answer B : Choice (B) involves improper phrasing. "That are" should be "such as."

Explanation for Incorrect Answer C : Choice (C) involves an improper idiom. "Such diversity as" should be "diversity such as."

Explanation for Incorrect Answer E : Choice (E) creates an error in agreement (in number). The phrase "a subject" cannot refer to the multiple subjects named.

Free from British rule after the American Revolution, a strong central government was an idea that many of the representatives attending the Constitutional Convention were wary of.
(A)

a strong central government was an idea that many of the representatives attending the Constitutional Convention were wary of

(B)

(C) many of the representatives attending the Constitutional Convention were wary of a strong central government

(E)

many representatives at the Constitutional Convention, wary of a strong central government

ANSWERS

AND EXPLANATIONS

Explanation for Correct Answer C : Choice (C) is correct. It avoids the error of the original by making clear that it was "the representatives" who were "free from British rule."

nR

(A)

Explanation for Incorrect Answer A : Choice (A) creates an illogical sentence. "A strong central government" is not what had become "free from British rule."

Explanation for Incorrect Answer B : Choice (B) creates an illogical sentence. "The idea of a strong central government" is not what had become "free from British rule."

Explanation for Incorrect Answer D : Choice (D) involves an improper preposition. One is wary of something, not toward something.

Explanation for Incorrect Answer E : Choice (E) creates a fragment. There is no main verb in the sentence.

Being cleaner and longer-burning compared with bituminous coal, anthracite was the first coal widely used in the United States for both domestic and industrial purposes.

Being cleaner and longer-burning compared with

(B)

Both cleaner and more longer-burning compared to

(C) Cleaner and longer-burning than

(D) By burning longer and more clean than

(E)

Cleaner as well as longer-burning, unlike

eg

is

te

(D) many representatives at the Constitutional Convention felt wary toward a strong central government

re d

the idea of a strong central government made wary many of the representatives attending the Constitutional Convention

file://E:\\f10.htm

2006-11-12

The Official SAT Online Course

5/8

ANSWERS

AND EXPLANATIONS

Explanation for Correct Answer C : Choice (C) is correct. It avoids the error of the original by correctly completing the comparison with the word "than."

Explanation for Incorrect Answer A : Choice (A) involves improper comparison. "Compared with" should be "than."

Explanation for Incorrect Answer B : Choice (B) involves improper comparison. "Compared to" should be "than."

Explanation for Incorrect Answer D : Choice (D) involves improper modification. To modify the verb "burning," "clean" would have to be "cleanly."

Explanation for Incorrect Answer E : Choice (E) involves improper comparison. It lacks "than" and adds the unnecessary word "unlike."

(A)

had prepared us equally well for the challenges of working and further study

(B)

had prepared us equally well for the challenges of work and of further study

(C) has supplied the preparation for challenging work along with further study

(D) leaves us prepared for the challenges of work and further study both

(E)

were the preparation for making the challenges of work or further study easier

ANSWERS

10

nR

(A)

Explanation for Correct Answer B : Choice (B) is correct. It avoids the error of the original by making the two items joined by "and" parallel. "Work" and "further study" are both nouns.

Explanation for Incorrect Answer A : Choice (A) involves an error in parallelism. The two items joined by "and" should be parallel, so "working" should be the noun "work."

Explanation for Incorrect Answer C : Choice (C) involves an error in agreement. The singular verb "has supplied" does not agree with its plural subject, "our many courses in the liberal arts."

Explanation for Incorrect Answer D : Choice (D) involves an error in agreement. The singular verb "leaves" does not agree with its plural subject, "our many courses in the liberal arts."

Explanation for Incorrect Answer E : Choice (E) involves awkward phrasing. "Were the preparation for making" should simply be "prepared us for."

Modern bluegrass songs, telling of love and despair and celebrating mountain beauty, reflect the genres rural origins.

Modern bluegrass songs, telling of love and despair and celebrating mountain beauty,

Modern bluegrass songs through their telling of love and despair and

eg

AND EXPLANATIONS

is t

er

ed

At graduation, the speaker assured us that our many courses in the liberal arts had prepared us equally well for the challenges of working and further study.

file://E:\\f10.htm

2006-11-12

The Official SAT Online Course

6/8

(B)

celebrating mountain beauty,

(C) Because modern bluegrass songs tell of love and despair and also celebrating mountain beauty, they

(D) With modern bluegrass songs that tell of love and despair and celebrate mountain beauty, they

(E)

Telling of love and despair, modern bluegrass songs celebrating mountain beauty, and they also

ANSWERS

AND EXPLANATIONS

Explanation for Correct Answer A : Choice (A) is correct. It uses correct verb forms and punctuation.

Explanation for Incorrect Answer B : Choice (B) involves an error in punctuation. The phrase "through...beauty" should be enclosed in commas.

Explanation for Incorrect Answer C : Choice (C) creates an illogical sentence. It suggests that "because modern bluegrass songs tell of love," the songs are therefore rural.

Explanation for Incorrect Answer E : Choice (E) uses an improper verb form. "Celebrating" should be "celebrate."

(B)

mechanisms, since it reproduces rapidly,

(C) mechanisms, since, with its rapid reproduction,

nR
(E)

(D) mechanisms; because it reproduces rapidly,

ANSWERS

mechanisms; then rapid reproduction allows

Explanation for Correct Answer D : Choice (D) is correct. It avoids the error of the original by correctly joining two complete thoughts ("The fruit fly...mechanisms" and "because...generations") with a semicolon.

Explanation for Incorrect Answer A : Choice (A) involves improper coordination. Two complete thoughts ("The fruit fly...mechanisms" and "because...generations") are joined by only a comma.

Explanation for Incorrect Answer B : Choice (B) involves improper coordination. Two complete thoughts ("The fruit fly...mechanisms" and "since...generations") are joined by only a comma.

Explanation for Incorrect Answer C : Choice (C) involves incorrect punctuation. The subordinating work "since" should not be preceded by a comma.

Explanation for Incorrect Answer E : Choice (E) involves improper phrasing. "Allows" should be followed by "to," not by "can."

eg

(A)

mechanisms, because it reproduces rapidly

AND EXPLANATIONS

is

11

The fruit fly is often used to study genetic mechanisms, because it reproduces rapidly scientists can observe the effects of experiments on several generations.

te

re

Explanation for Incorrect Answer D : Choice (D) involves unclear pronoun reference. What the pronoun "they" refers to is not clear.

file://E:\\f10.htm

2006-11-12

The Official SAT Online Course

7/8

12

Benin was the first sub-Saharan African country to experience a civilian coup: they were a regime that was dominated by the armed forces and obliged by citizens to implement democratic reforms.
(A)

they were a regime that was dominated by the armed forces and obliged by

(B)

they had been a regime that was dominated by the armed forces, when they were obliged to

(C) it had a regime, armed forces dominating, but then were obliged to

(D) armed forces dominated them until this regime were obliged by

(E)

a regime, dominated by the armed forces, was obliged by

ANSWERS

AND EXPLANATIONS

Explanation for Correct Answer E : Choice (E) is correct. It avoids the error of the original by using the singular verb "was" to agree with the singular noun "regime."

Explanation for Incorrect Answer B : Choice (B) involves an error in agreement. The plural pronoun "they" cannot correctly refer to the singular noun "Benin."

13

nR
(A)

This legend about Admiral Nelson, like other naval heroes, are based only partially on fact.

(B)

like other naval heroes, are

like those of other naval heroes, are

(C) like other naval heroes, is

(D) like legends about other naval heroes, are

(E)

like legends about other naval heroes, is

ANSWERS

Explanation for Correct Answer E : Choice (E) is correct. It avoids the error of the original by comparing "this legend" to "legends about other naval heroes."

Explanation for Incorrect Answer A : Choice (A) involves an illogical comparison. It compares "this legend" to "other naval heroes."

Explanation for Incorrect Answer B : Choice (B) involves an error in agreement. It uses the plural verb "are" for the singular subject "this legend."

Explanation for Incorrect Answer C : Choice (C) involves an illogical comparison. It compares "this legend" to "other

eg

Explanation for Incorrect Answer D : Choice (D) involves an error in agreement. The plural pronoun "them" cannot correctly refer to the singular noun "Benin."

AND EXPLANATIONS

is t

Explanation for Incorrect Answer C : Choice (C) involves an error in agreement. The plural verb "were" cannot agree with the singular subject "Benin."

er ed

Explanation for Incorrect Answer A : Choice (A) involves an error in agreement. The plural pronoun "they" cannot correctly refer to the singular noun "Benin."

file://E:\\f10.htm

2006-11-12

The Official SAT Online Course


naval heroes."

8/8

Explanation for Incorrect Answer D : Choice (D) involves an error in agreement. It uses the plural verb "are" for the singular subject "this legend."

14

Bats and mosquitoes come out at twilight, and the bats would look for mosquitoes and the mosquitoes would look for people.
(A)

and the bats would look for mosquitoes and the mosquitoes would look

(B)

and the bats come to look for mosquitoes while the mosquitoes look

(C) the bats look for mosquitoes and the mosquitoes are looking

(D) the bats looking for mosquitoes while mosquitoes would look

(E)

the bats to look for mosquitoes and the mosquitoes to look

ANSWERS

AND EXPLANATIONS

Explanation for Correct Answer E : Choice (E) is correct. It avoids the error of the original by using consistent verb forms ("to look").

Explanation for Incorrect Answer A : Choice (A) involves an error in verb tense. It shifts from present tense ("come") to past tense ("would look") for no reason.

Explanation for Incorrect Answer B : Choice (B) involves an error in verb form. It shifts from "come to look" to "look" for no reason.

nR

Explanation for Incorrect Answer D : Choice (D) involves an error in verb form. The participle "looking" is an incomplete verb.

eg

Explanation for Incorrect Answer C : Choice (C) involves an error in verb form. It shifts from "look" to "are looking" for no reason.

is te

re d

Privacy Policy

Back to Score Report

Copyright 2006 The College Board. All rights reserved.

Terms of Use

Contact Us

file://E:\\f10.htm

2006-11-12

Anda mungkin juga menyukai